Wikipedia:Reference desk/Archives/Science/September 2005

From Wikipedia, the free encyclopedia

uwuwiiw

Do partially broken human teeth grow?[edit]

If an adult human breaks a part of his/her tooth, does the tooth grow or always remains partially broken? If the tooth is pulled out, does a new tooth grow in adults? --anon

As someone who has spent entirely too much money in lifetime paying Dentist bills, and now 100% false teeth, it seems to me the battle was to save what was left of eroded teeth until I was willing to part with the last of them. In children there is something called wisdom teeth, in which something grows, comes out, then is replaced with another, but in my adult experience, once you got a damaged tooth, or a cavety, the dentist can patch it up, but there is no "natural" cure. AlMac|(talk) 07:18, 30 August 2005 (UTC)[reply]
I'm not sure what you mean about the wisdom teeth? --Phroziac (talk) 16:58, September 3, 2005 (UTC)
  • Children will exchange their milk teeth, but after that your teeth won't grow anymore. I speak from experience as two people in my family had broken front teeth. The dentist patched it up with fake teeth or fake partial teeth, but once damaged, adult teeth can not recover. In fact, once you've exchanged your teeth as a kid, your new teeth won't grow anymore. They're already adult size. - Mgm|(talk) 08:37, August 30, 2005 (UTC)
  • Humans only get two sets of teeth - their milk teeth and the adult set, though the wisdom teeth at the back may not appear until you're in your twenties and (in my case) have to come out straight away. If you damage your adult teeth, that's it, you're stuck weith them - they don't grow any more. -- Arwel 12:24, 31 August 2005 (UTC)[reply]
  • Which I can, unfortunatly, confirm from my own exprerience. A small piece of my lateral incisor broke of after I fell flat on my face. More than a decade later it still hasn't grown back, although it has become smoother. --R.Koot 13:10, 31 August 2005 (UTC)[reply]
  • By the way, why are they called milk teeth? --Phroziac (talk) 16:58, September 3, 2005 (UTC)

mathematics[edit]

who discovered rational numbers?

The ancient Egyptians knew quite a lot about them, though the general idea may have crossed somebody else's mind before that. David Sneek 07:21, 30 August 2005 (UTC)[reply]
See rational numbers. --R.Koot 11:45, 30 August 2005 (UTC)[reply]


My sister's computer is becoming a real pain. While it worked before, suddenly items she tries to download won't show in the traffic screen after clicking download. According to the program she's connected (searches work fine and there's a disconnected option to click in the menu) so what's going on? - Mgm|(talk) 08:40, August 30, 2005 (UTC)

Are other people able to download files from your sister while she's connected to the network? It might be that her ISP is blocking the Kazaa service on their end, especially if she's on a college campus or something similar. Garrett Albright 16:33, 30 August 2005 (UTC)[reply]
  • Haven't been able to check, because I don't know how to make sure someone gets on the same network. Anyway, I doubt our ISP is blocking the service, because she used the same ISP the day before when it worked fine and it's at home, not a campus of any kind. - Mgm|(talk) 21:21, August 30, 2005 (UTC)

Have you installed any new software, a firewall perhaps? If all else fails and you're sure that your internet connection is working fine, you may want to try reinstalling it. If you do this, be careful that it doesn't delete all of your shared files during the reinstallation pro

Medical Information[edit]

What do doctors add in the medicine that contains antibodies in a syringe?

I might have misunderstood your question... but if you mean excipients (agents that are added to improve stability of the active ingredient or to maintain it in solution) then you can find quite a lot of information on the electronic Medicines compendium if you know the name of the vaccine. The website is http://emc.medicines.org.uk and once you've typed in the name of the vaccine, choose the link with the letters SPC (which stand for Summary of Product Characteristics) next to it, and browse the excipients section (I think it is always section 6.1). This works for UK medicines, I don't know what other databases there are, but Google might have the answer. Hope that helps, Jo Brodie, 16:49 BST, August 30, 2005.

This is a peculiar question and I suspect represents a misunderstanding or a non-Western practice. It is extremely rare for American or western European doctors to give injections of antibodies in a syringe. Antibodies in the form of gamma globulin are given by iv infusion for many problems but this doesn't involve a syringe. I cannot off the top of my head think of any examples except old fashioned passive immunity treatments for rabies, tetanus, and snakebite antivenin. There are a number of newer monoclonal antibodies used as treatments for specific uncommon conditions, but most of these are still experimental research protocols and most are given by infusion rather than injection. There were some quack cancer treatments involving injections in the past which claimed to induce a patient to make antibodies against the tumor, but these were pretty uncommon. Can you give us more context? What disease is being treated, and in what country, and is the doctor a mainstream MD or alternative practitioner? alteripse 16:19, 30 August 2005 (UTC)[reply]

The questioner really needs to give us more to go on. Another possibility is that they are referring to the use of radiolabelled antibodies in nuclear medicine. - Nunh-huh 20:41, 30 August 2005 (UTC)[reply]

Assuming the antibodies is a mistake, and concentrating on the "What doctors add" bit: I believe one thing added to (all?) injections of medicine is a buffer, to prevent blood pH from changing. -- DrBob 20:49, 30 August 2005 (UTC)[reply]
To the extent that any diluent is a buffer, it's so to avoid discomfort from the injection of highly basic or acidic solutions, which are locally irritating. It's a rare injection that would change blood pH....50 cc of sodium bicarbonate, usually in multiples, gets the job done for a while, but not your everyday run-of-the-mill injection. - Nunh-huh 22:45, 30 August 2005 (UTC) Absolutely right. alteripse 01:17, 31 August 2005 (UTC)[reply]
Huh, I guess my A-level chemistry lied to me, then. -- DrBob 03:17, 31 August 2005 (UTC) Disillusionment can be tough. Sorry if it was your first time. alteripse 10:47, 31 August 2005 (UTC)[reply]

universe as animation frames[edit]

In The Dilbert Future, author Scott Adams says, "Some physicists theorize that reality is like frames of an animated movie, with infinite universes existing at once." Any idea which physicists they are or what their theory is called. Multiverse article didn't help me. Jay 18:42, 30 August 2005 (UTC)[reply]

Probably not the physicist he was refering to, but if you're looking for that type of metaphore, I suggest you try Michio Kaku, he's pretty good for a pop culture physicist--152.163.101.12 21:28, 30 August 2005 (UTC)[reply]

Ring around the Earth[edit]

One thing which I've always wondered. If you were to build a steel ring with a diameter of, say, 100 metres or 1 kilometre greater than that of the Earth, and somehow place it around the entire Earth, would it more or less hover in mid-air with no support of any kind? JIP | Talk 19:50, 30 August 2005 (UTC)[reply]

Someone's been reading a bit too much Arthur C Clarke, personally i thought 3001 was a bit of a let down, but I guess you liked it well enough--152.163.101.12 21:25, 30 August 2005 (UTC)[reply]
Not really. The Earth is pretty rough at that scale - you have mountains, seas, that sort of thing - which are very inhomogenous; it'd be like dealing with a lot of weird mascons. (Consider the gravitational effects of "ring next to a mountain" vs. "ring over central Pacific). It'd probably "crash to the ground" somewhere along the length - or, indeed, buckle under its own weight and do so all over the place. Shimgray 19:55, 30 August 2005 (UTC)[reply]
It wouldn't be very stable unless you spun it, but you could make it a geosynchronous orbit. You might want to look at Ringworld and Dyson sphere. Bovlb 19:59:37, 2005-08-30 (UTC)
At geosynch you might manage stable, but not at a kilometer high - local mass differences are just too significant, then. Shimgray 20:33, 30 August 2005 (UTC)[reply]
You might also find an interest in the space elevator concept. How is that supposed to stay where it is supposed to be? AlMac|(talk) 06:08, 31 August 2005 (UTC)[reply]
The plan for that is to be above geosynchronous orbit (much much more than 1 km above the surface). Technically it would work at geosynchronous orbit, but the designers want the elevator structure to be under some tension. The ring structure would "hover" at geosynchronous orbit, except that our geoid isn't perfect and we've dumped lots of satellites up there, so I don't think it'd be particularly stable, but — as mentioned above — spinning it up would help. — Laura Scudder | Talk 15:49, 31 August 2005 (UTC)[reply]
I believe the general concept is to counterweight an elevator so that the center of gravity is at the geosynch point. As Laurascudder noted, though, we'd also have to do some major orbital cleanup. IIRC, Kim Stanley Robinson discusses a lot of this in his book Red Mars — Lomn | Talk / RfC 15:54:06, 2005-08-31 (UTC)

"Luminous toxin"[edit]

Just curious, but is "luminous toxin" (the poison that slowly killed the character in the 1950 movie D.O.A.) a real poison? According to the movie, the poison is real (from the movie: "``The medical facts in this motion picture are authentic. Luminous toxin is a descriptive term for an actual poison.'' — Technical Adviser, Edward F. Dunne, M.D.") However, after I have searched the web and Wikipedia for "luminous toxin" and only found references to it in articles related to the film (no scientific/medical articles), I am starting to wonder whether "luminous toxin" is real or not. Thanks. 61.94.149.174 06:33, 31 August 2005 (UTC)[reply]

A "descriptive term for an actual poison" means thay are describing the toxin rather than giving you its name. Medical articles will refer to it by its actual name. I could describe milk as "a white liquid complete baby food" but everyone else will still call it milk. Theresa Knott (a tenth stroke) 19:30, 31 August 2005 (UTC)[reply]
Expect a 40's film to be very loose with scientific merit. Very roughly, you could describe phosphosous as a luminous toxin. White phosphorous, specifically, glows and is a systemic poison, although mostly known for bone damage. This was probably close enough for the purposes of a sensational movie script. However, having several days of relative health before dying (despite the best medical intervention) is possibly more charactistic of overdose from paracetamol or deadly Amanita mushrooms, both of which cause liver failure that results in death after an initial recovery 131.172.99.15 (talk) 12:44, 23 December 2008 (UTC)snaxalotl[reply]
Without a reliable citation, phosphorous or one of its compounds is just a guess. I have marked the phosphorous passage 'citation needed'. I doubt that either of the authors of D.O.A. knew of a specific chemical with the properties described in the film. It's probably literary invention. Without inside information, we cannot even be sure there really was a medical consultant on the film. David Spector (talk) 16:50, 27 January 2011 (UTC)[reply]

The "luminous toxin" was iridium. Watch DOA! - that's not correct. You misunderstood the plot line.

Probability: The “Oldest Son or Daughter Phenomenon”[edit]

Explain why in a group of 35 children at a school, a randomly selected individual has a high probability of being the oldest son or daughter in his or her family.

--anonym

Maybe because there are a lot of one / two child famillies nowadays? --βjweþþ (talk) 07:54, 31 August 2005 (UTC)[reply]

Presumably this is a math assignment. If the average number of children per family are less than 2 (as is true in most of the western world), any randomly chosen child is more likely to be an eldest than a non-eldest.

Average children per family (CperF) can be expressed as N1+N2+N3+N4+N5+N6...+N20 divided by the NF, where N1 is total number of eldest children, N2 is total number of second children, N3 is total number of third children, etc, and NF is the total number of families. There is a value of CperF below which more than half of any group of children are eldest (N1). Is this what you are supposed to compute? It would be somewhat above 2 because preschoolers (age under 5 years) would be selectively excluded from your classroom sample.

Unless King Herod or the Tenth Plague of Egypt has visited the community lately, there will always be more eldest children than second, more second children than third, etc (i.e., N1>N2>N3...N20), so that even in a society in which there are on average many children per family (high CperF), any randomly chosen child is more likely to be an eldest than a second, third, fourth, etc. alteripse 11:16, 31 August 2005 (UTC)[reply]

Also if the scholl is a kindergarten, the youngest son/daughter might not have been born yet. --R.Koot 12:22, 31 August 2005 (UTC)[reply]
Actually, this is not relevant. Whether a younger sibling was already born or not, it won't change the fact that the elder is the elder (even if s/he is the only child). Ornil 16:53, 31 August 2005 (UTC)[reply]
I think it is relevant, since having a child in kindergarten correlates with having young children in general (i.e. having a young family) and therefore correlates with not having had all of your children yet; i.e. the distribution will be skewed toward having one (or few) child MORE THAN the average distribution131.172.99.15 (talk) 12:51, 23 December 2008 (UTC)snaxalotl[reply]

The Expected Value of a Sweepstakes Contest[edit]

A magazine clearinghouse holds a sweepstakes contest to sell subscriptions. If you return the winning number, you win $1,000,000 (USD). You have a 1-in-20-million chance of winning, but your only cost to enter the contest is a first-class stamp to mail the entry. Use the current price of a first-class stamp to calculate the expected net winnings if you enter this contest. Is it worth entering the sweepstakes?

--anonym

Yes. No. Maybe. Depends whose stamp it is. I think it's probably better to be doing your homework than entering the sweepstakes. Notinasnaid 07:58, 31 August 2005 (UTC)[reply]


  • If the price of a first-class stamp is $1.29, then you can expect to lose .

Power and torque output (of WRC cars)[edit]

I want to ask a question about power and torque, more specifically about rally car engines. Reportedly, Peugeot 206 WRC's 2-liter turbocharged engine generates 635 Nm of torque at 4000 rpm. if i'm not mistaken, this makes 667 hp at the same rpm. but the car's maximum power output is reported to be 300 HP at 5250 rpm (and 300 HP is the maximum legal power output in the championship). How can the engine develop such a high amount of torque considering that the power output is not that high? Isn't there an inconsistency between the figures? Am I just miscalculating or how is this possible? Thanks, --Ozkaplan

According to the article on torque, if is power in kilowatts, is torque in Newton metres, and R = rotational speed in revolutions per minute, if you manipulate the constants:
If you plug in T = 635 and R = 4000 into that I get approximately 266 kilowatts, which is about 357 horsepower. Considerably over the limit, still. As to how it's done, let's just say that creative interpretations of physics, mathematics, and indeed logic pertaining to rulebooks have been a stock in trade of automobile racing for a very long time. In the 1980's, the maximum level of turbo boost in Formula One was supposed to be restricted by the use of a standard popoff valve, which opens up to bleed off pressurised air if the pressure gets too high. Teams apparently got around this by fitting huge turbos to their qualifying motors which pumped so much air the popoff valve couldn't get rid of it all...--Robert Merkel 12:18, 31 August 2005 (UTC)[reply]

computers[edit]

what's the speed of processor for a computer system?

Speed is measured in Hertz, or cycles per second. Modern systems run in the Gigahertz range, though one cycle does not necessarily equal one instruction. See Central processing unit and clock rate for more info. — Lomn | Talk / RfC 15:36:50, 2005-08-31 (UTC)

Analysing the performance of a computer system is a very, very complex topic, if you want to tackle it in all its glory. However, these days, for most purposes, for most people, the speed of the processor in your PC doesn't really matter - you will barely be able to tell the difference in CPU speed between the slowest and fastest x86-compatible PC Intel and AMD make. The thing that causes the most perceived delay in my use of a computer is almost always my Internet connection. --Robert Merkel 22:45, 31 August 2005 (UTC)[reply]

Review of Manufacturing topic / categories[edit]

I checked a minor manufacturing topic; it got me looking into Manufacturing as a whole. It seems to me that it is a very big topic that is fragmented and difficult to navigate. I am not sure how to start reviewing it without being a vandal! The categories lists also seem in need of review. I might start by creating a "Manufacturing overview" page similar to this temporary page. Joe1011010 19:41, 31 August 2005 (UTC)[reply]

I think you should go ahead with a rewrite if you think you can improve the article(s). BUT, in a high level topic normally we would not just say "The beginings of manufacturing is covered in the Industrial Revolution." Rather, it is better to have a condensed overview of the industrial revolution in the manufacuring article, along with an italisized statement indicating that more detail can be found in the article on the main article on the industrial revolution. Happy editing ike9898 17:22, September 6, 2005 (UTC)out, does a new tooth grow in adults? --anon

As someone who has spent entirely too much money in lifetime paying Dentist bills, and now 100% false teeth, it seems to me the battle was to save what was left of eroded teeth until I was willing to part with the last of them. In children there is something called wisdom teeth, in which something grows, comes out, then is replaced with another, but in my adult experience, once you got a damaged tooth, or a cavety, the dentist can patch it up, but there is no "natural" cure. AlMac|(talk) 07:18, 30 August 2005 (UTC) I'm not sure what you mean about the wisdom teeth? --Phroziac (talk) 16:58, September 3, 2005 (UTC) Children will exchange their milk teeth, but after that your teeth won't grow anymore. I speak from experience as two people in my family had broken front teeth. The dentist patched it up with fake teeth or fake partial teeth, but once damaged, adult teeth can not recover. In fact, once you've exchanged your teeth as a kid, your new teeth won't grow anymore. They're already adult size. - Mgm|(talk) 08:37, August 30, 2005 (UTC) Humans only get two sets of teeth - their milk teeth and the adult set, though the wisdom teeth at the back may not appear until you're in your twenties and (in my case) have to come out straight away. If you damage your adult teeth, that's it, you're stuck weith them - they don't grow any more. -- Arwel 12:24, 31 August 2005 (UTC) Which I can, unfortunatly, confirm from my own exprerience. A small piece of my lateral incisor broke of after I fell flat on my face. More than a decade later it still hasn't grown back, although it has become smoother. --R.Koot 13:10, 31 August 2005 (UTC) By the way, why are they called milk teeth? --Phroziac (talk) 16:58, September 3, 2005 (UTC) mathematics Edit

who discovered rational numbers?

The ancient Egyptians knew quite a lot about them, though the general idea may have crossed somebody else's mind before that. David Sneek 07:21, 30 August 2005 (UTC) See rational numbers. --R.Koot 11:45, 30 August 2005 (UTC)

My sister's computer is becoming a real pain. While it worked before, suddenly items she tries to download won't show in the traffic screen after clicking download. According to the program she's connected (searches work fine and there's a disconnected option to click in the menu) so what's going on? - Mgm|(talk) 08:40, August 30, 2005 (UTC)

Are other people able to download files from your sister while she's connected to the network? It might be that her ISP is blocking the Kazaa service on their end, especially if she's on a college campus or something similar. Garrett Albright 16:33, 30 August 2005 (UTC) Haven't been able to check, because I don't know how to make sure someone gets on the same network. Anyway, I doubt our ISP is blocking the service, because she used the same ISP the day before when it worked fine and it's at home, not a campus of any kind. - Mgm|(talk) 21:21, August 30, 2005 (UTC) Have you installed any new software, a firewall perhaps? If all else fails and you're sure that your internet connection is working fine, you may want to try reinstalling it. If you do this, be careful that it doesn't delete all of your shared files during the reinstallation pro

Medical Information Edit

What do doctors add in the medicine that contains antibodies in a syringe?

I might have misunderstood your question... but if you mean excipients (agents that are added to improve stability of the active ingredient or to maintain it in solution) then you can find quite a lot of information on the electronic Medicines compendium if you know the name of the vaccine. The website is http://emc.medicines.org.uk and once you've typed in the name of the vaccine, choose the link with the letters SPC (which stand for Summary of Product Characteristics) next to it, and browse the excipients section (I think it is always section 6.1). This works for UK medicines, I don't know what other databases there are, but Google might have the answer. Hope that helps, Jo Brodie, 16:49 BST, August 30, 2005.

This is a peculiar question and I suspect represents a misunderstanding or a non-Western practice. It is extremely rare for American or western European doctors to give injections of antibodies in a syringe. Antibodies in the form of gamma globulin are given by iv infusion for many problems but this doesn't involve a syringe. I cannot off the top of my head think of any examples except old fashioned passive immunity treatments for rabies, tetanus, and snakebite antivenin. There are a number of newer monoclonal antibodies used as treatments for specific uncommon conditions, but most of these are still experimental research protocols and most are given by infusion rather than injection. There were some quack cancer treatments involving injections in the past which claimed to induce a patient to make antibodies against the tumor, but these were pretty uncommon. Can you give us more context? What disease is being treated, and in what country, and is the doctor a mainstream MD or alternative practitioner? alteripse 16:19, 30 August 2005 (UTC)

The questioner really needs to give us more to go on. Another possibility is that they are referring to the use of radiolabelled antibodies in nuclear medicine. - Nunh-huh 20:41, 30 August 2005 (UTC)

Assuming the antibodies is a mistake, and concentrating on the "What doctors add" bit: I believe one thing added to (all?) injections of medicine is a buffer, to prevent blood pH from changing. -- DrBob 20:49, 30 August 2005 (UTC) To the extent that any diluent is a buffer, it's so to avoid discomfort from the injection of highly basic or acidic solutions, which are locally irritating. It's a rare injection that would change blood pH....50 cc of sodium bicarbonate, usually in multiples, gets the job done for a while, but not your everyday run-of-the-mill injection. - Nunh-huh 22:45, 30 August 2005 (UTC) Absolutely right. alteripse 01:17, 31 August 2005 (UTC) Huh, I guess my A-level chemistry lied to me, then. -- DrBob 03:17, 31 August 2005 (UTC) Disillusionment can be tough. Sorry if it was your first time. alteripse 10:47, 31 August 2005 (UTC) universe as animation frames Edit

In The Dilbert Future, author Scott Adams says, "Some physicists theorize that reality is like frames of an animated movie, with infinite universes existing at once." Any idea which physicists they are or what their theory is called. Multiverse article didn't help me. Jay 18:42, 30 August 2005 (UTC)

Probably not the physicist he was refering to, but if you're looking for that type of metaphore, I suggest you try Michio Kaku, he's pretty good for a pop culture physicist--152.163.101.12 21:28, 30 August 2005 (UTC) Ring around the Earth Edit

One thing which I've always wondered. If you were to build a steel ring with a diameter of, say, 100 metres or 1 kilometre greater than that of the Earth, and somehow place it around the entire Earth, would it more or less hover in mid-air with no support of any kind? — JIP | Talk 19:50, 30 August 2005 (UTC)

Someone's been reading a bit too much Arthur C Clarke, personally i thought 3001 was a bit of a let down, but I guess you liked it well enough--152.163.101.12 21:25, 30 August 2005 (UTC) Not really. The Earth is pretty rough at that scale - you have mountains, seas, that sort of thing - which are very inhomogenous; it'd be like dealing with a lot of weird mascons. (Consider the gravitational effects of "ring next to a mountain" vs. "ring over central Pacific). It'd probably "crash to the ground" somewhere along the length - or, indeed, buckle under its own weight and do so all over the place. Shimgray 19:55, 30 August 2005 (UTC) It wouldn't be very stable unless you spun it, but you could make it a geosynchronous orbit. You might want to look at Ringworld and Dyson sphere. Bovlb 19:59:37, 2005-08-30 (UTC) At geosynch you might manage stable, but not at a kilometer high - local mass differences are just too significant, then. Shimgray 20:33, 30 August 2005 (UTC) You might also find an interest in the space elevator concept. How is that supposed to stay where it is supposed to be? AlMac|(talk) 06:08, 31 August 2005 (UTC) The plan for that is to be above geosynchronous orbit (much much more than 1 km above the surface). Technically it would work at geosynchronous orbit, but the designers want the elevator structure to be under some tension. The ring structure would "hover" at geosynchronous orbit, except that our geoid isn't perfect and we've dumped lots of satellites up there, so I don't think it'd be particularly stable, but — as mentioned above — spinning it up would help. — Laura Scudder | Talk 15:49, 31 August 2005 (UTC) I believe the general concept is to counterweight an elevator so that the center of gravity is at the geosynch point. As Laurascudder noted, though, we'd also have to do some major orbital cleanup. IIRC, Kim Stanley Robinson discusses a lot of this in his book Red Mars — Lomn | Talk / RfC 15:54:06, 2005-08-31 (UTC) "Luminous toxin" Edit

Just curious, but is "luminous toxin" (the poison that slowly killed the character in the 1950 movie D.O.A.) a real poison? According to the movie, the poison is real (from the movie: "``The medical facts in this motion picture are authentic. Luminous toxin is a descriptive term for an actual poison. — Technical Adviser, Edward F. Dunne, M.D.") However, after I have searched the web and Wikipedia for "luminous toxin" and only found references to it in articles related to the film (no scientific/medical articles), I am starting to wonder whether "luminous toxin" is real or not. Thanks. 61.94.149.174 06:33, 31 August 2005 (UTC)

A "descriptive term for an actual poison" means thay are describing the toxin rather than giving you its name. Medical articles will refer to it by its actual name. I could describe milk as "a white liquid complete baby food" but everyone else will still call it milk. Theresa Knott (a tenth stroke) 19:30, 31 August 2005 (UTC) Expect a 40's film to be very loose with scientific merit. Very roughly, you could describe phosphosous as a luminous toxin. White phosphorous, specifically, glows and is a systemic poison, although mostly known for bone damage. This was probably close enough for the purposes of a sensational movie script. However, having several days of relative health before dying (despite the best medical intervention) is possibly more charactistic of overdose from paracetamol or deadly Amanita mushrooms, both of which cause liver failure that results in death after an initial recovery 131.172.99.15 (talk) 12:44, 23 December 2008 (UTC)snaxalotl Without a reliable citation, phosphorous or one of its compounds is just a guess. I have marked the phosphorous passage 'citation needed'. I doubt that either of the authors of D.O.A. knew of a specific chemical with the properties described in the film. It's probably literary invention. Without inside information, we cannot even be sure there really was a medical consultant on the film. David Spector (talk) 16:50, 27 January 2011 (UTC) The "luminous toxin" was iridium. Watch DOA! - that's not correct. You misunderstood the plot line.

Probability: The “Oldest Son or Daughter Phenomenon” Edit

Explain why in a group of 35 children at a school, a randomly selected individual has a high probability of being the oldest son or daughter in his or her family.

--anonym

Maybe because there are a lot of one / two child famillies nowadays? --βjweþþ (talk) 07:54, 31 August 2005 (UTC) Presumably this is a math assignment. If the average number of children per family are less than 2 (as is true in most of the western world), any randomly chosen child is more likely to be an eldest than a non-eldest.

Average children per family (CperF) can be expressed as N1+N2+N3+N4+N5+N6...+N20 divided by the NF, where N1 is total number of eldest children, N2 is total number of second children, N3 is total number of third children, etc, and NF is the total number of families. There is a value of CperF below which more than half of any group of children are eldest (N1). Is this what you are supposed to compute? It would be somewhat above 2 because preschoolers (age under 5 years) would be selectively excluded from your classroom sample.

Unless King Herod or the Tenth Plague of Egypt has visited the community lately, there will always be more eldest children than second, more second children than third, etc (i.e., N1>N2>N3...N20), so that even in a society in which there are on average many children per family (high CperF), any randomly chosen child is more likely to be an eldest than a second, third, fourth, etc. alteripse 11:16, 31 August 2005 (UTC)

Also if the scholl is a kindergarten, the youngest son/daughter might not have been born yet. --R.Koot 12:22, 31 August 2005 (UTC) Actually, this is not relevant. Whether a younger sibling was already born or not, it won't change the fact that the elder is the elder (even if s/he is the only child). Ornil 16:53, 31 August 2005 (UTC) I think it is relevant, since having a child in kindergarten correlates with having young children in general (i.e. having a young family) and therefore correlates with not having had all of your children yet; i.e. the distribution will be skewed toward having one (or few) child MORE THAN the average distribution131.172.99.15 (talk) 12:51, 23 December 2008 (UTC)snaxalotl The Expected Value of a Sweepstakes Contest Edit

A magazine clearinghouse holds a sweepstakes contest to sell subscriptions. If you return the winning number, you win $1,000,000 (USD). You have a 1-in-20-million chance of winning, but your only cost to enter the contest is a first-class stamp to mail the entry. Use the current price of a first-class stamp to calculate the expected net winnings if you enter this contest. Is it worth entering the sweepstakes?

--anonym

Yes. No. Maybe. Depends whose stamp it is. I think it's probably better to be doing your homework than entering the sweepstakes. Notinasnaid 07:58, 31 August 2005 (UTC)


If the price of a first-class stamp is $1.29, then you can expect to lose ≈ − $ 1.24 {\displaystyle \approx -\$1.24}. Power and torque output (of WRC cars) Edit

I want to ask a question about power and torque, more specifically about rally car engines. Reportedly, Peugeot 206 WRC's 2-liter turbocharged engine generates 635 Nm of torque at 4000 rpm. if i'm not mistaken, this makes 667 hp at the same rpm. but the car's maximum power output is reported to be 300 HP at 5250 rpm (and 300 HP is the maximum legal power output in the championship). How can the engine develop such a high amount of torque considering that the power output is not that high? Isn't there an inconsistency between the figures? Am I just miscalculating or how is this possible? Thanks, --Ozkaplan

According to the article on torque, if P P is power in kilowatts, T T is torque in Newton metres, and R = rotational speed in revolutions per minute, if you manipulate the constants: P = T × R × 2 π 60000 {\displaystyle P={\frac {T\times R\times 2\pi }{60000}}} If you plug in T = 635 and R = 4000 into that I get approximately 266 kilowatts, which is about 357 horsepower. Considerably over the limit, still. As to how it's done, let's just say that creative interpretations of physics, mathematics, and indeed logic pertaining to rulebooks have been a stock in trade of automobile racing for a very long time. In the 1980's, the maximum level of turbo boost in Formula One was supposed to be restricted by the use of a standard popoff valve, which opens up to bleed off pressurised air if the pressure gets too high. Teams apparently got around this by fitting huge turbos to their qualifying motors which pumped so much air the popoff valve couldn't get rid of it all...--Robert Merkel 12:18, 31 August 2005 (UTC) computers Edit

what's the speed of processor for a computer system?

Speed is measured in Hertz, or cycles per second. Modern systems run in the Gigahertz range, though one cycle does not necessarily equal one instruction. See Central processing unit and clock rate for more info. — Lomn | Talk / RfC 15:36:50, 2005-08-31 (UTC)

Analysing the performance of a computer system is a very, very complex topic, if you want to tackle it in all its glory. However, these days, for most purposes, for most people, the speed of the processor in your PC doesn't really matter - you will barely be able to tell the difference in CPU speed between the slowest and fastest x86-compatible PC Intel and AMD make. The thing that causes the most perceived delay in my use of a computer is almost always my Internet connection. --Robert Merkel 22:45, 31 August 2005 (UTC) Review of Manufacturing topic / categories Edit

I checked a minor manufacturing topic; it got me looking into Manufacturing as a whole. It seems to me that it is a very big topic that is fragmented and difficult to navigate. I am not sure how to start reviewing it without being a vandal! The categories lists also seem in need of review. I might start by creating a "Manufacturing overview" page similar to this temporary page. Joe1011010 19:41, 31 August 2005 (UTC)

I think you should go ahead with a rewrite if you think you can improve the article(s). BUT, in a high level topic normally we would not just say "The beginings of manufacturing is covered in the Industrial Revolution." Rather, it is better to have a condensed overview of the industrial revolution in the manufacuring article, along with an italisized statement indicating that more detail can be found in the article on the main article on the industrial revolution. Happy editing ike9898 17:22, September 6, 2005 (UTC) biosphere Edit

Ñ How does energy and nutrients move through the biosphere? How are these two different?

I suggest you start by reading the Biosphere article and then do you're homework yourself - believe it or not, this will help you in the long run (and, no I don't think I would have believed it when I was in school!). Thryduulf 20:50, 31 August 2005 (UTC) Truth Drug Edit

how do u test or what kind of test can you take to verify that you have been given a truth drug with out consent?

Well you would notice a sedative effect. Like being drunk. See Truth drug for more info Theresa Knott (a tenth stroke) 20:46, 31 August 2005 (UTC)

Also, a blood test for drugs (the kind pathologists give accident victims, for instance) taken afterward would also reveal their presence. I don't know how long the presence of the different types of potential truth drugs is detectable after their admission, and specifically the "window" between when they wear off sufficiently to give you enough self-control to voluntarily get a blood sample taken, and when they are no longer detectable. I suspect it's a matter of hours rather than days, but I'm certainly no expert in the area. --Robert Merkel 22:59, 31 August 2005 (UTC) S-box S5 on Substition box page Edit

First, thank you for your service.

On http://en.wikipedia.org/wiki/S-box, I believe the binary value 1010 is missing as a column heading and as an output.

I think it should be the column heading for column 11, which is currently labeled 1100.

According to http://www.tropsoft.com/strongenc/des.htm, 1010 should be in column 6 (0101), row 1 (00); column 12 (1011), row 2 (01); column 5 (0100), row 3 (10); and column 13 (1100), row 4 (11). At Wikipedia, anybody can edit, with or without an account. If you spot a logical error, go to the page in question, hit the edit this page link at the top, and go to it! For added niceness, you can then proceed to the discussion link and add a link to the source so that we've got it available for review. — Lomn | Talk / RfC 20:55:30, 2005-08-31 (UTC)

biosphere[edit]

Ñ How does energy and nutrients move through the biosphere? How are these two different?

I suggest you start by reading the Biosphere article and then do you're homework yourself - believe it or not, this will help you in the long run (and, no I don't think I would have believed it when I was in school!). Thryduulf 20:50, 31 August 2005 (UTC)[reply]

Truth Drug[edit]

how do u test or what kind of test can you take to verify that you have been given a truth drug with out consent?

Well you would notice a sedative effect. Like being drunk. See Truth drug for more info Theresa Knott (a tenth stroke) 20:46, 31 August 2005 (UTC)[reply]

Also, a blood test for drugs (the kind pathologists give accident victims, for instance) taken afterward would also reveal their presence. I don't know how long the presence of the different types of potential truth drugs is detectable after their admission, and specifically the "window" between when they wear off sufficiently to give you enough self-control to voluntarily get a blood sample taken, and when they are no longer detectable. I suspect it's a matter of hours rather than days, but I'm certainly no expert in the area. --Robert Merkel 22:59, 31 August 2005 (UTC)[reply]

S-box S5 on Substition box page[edit]

First, thank you for your service.

On http://en.wikipedia.org/wiki/S-box, I believe the binary value 1010 is missing as a column heading and as an output.

I think it should be the column heading for column 11, which is currently labeled 1100.

According to http://www.tropsoft.com/strongenc/des.htm, 1010 should be in column 6 (0101), row 1 (00); column 12 (1011), row 2 (01); column 5 (0100), row 3 (10); and column 13 (1100), row 4 (11). At Wikipedia, anybody can edit, with or without an account. If you spot a logical error, go to the page in question, hit the edit this page link at the top, and go to it! For added niceness, you can then proceed to the discussion link and add a link to the source so that we've got it available for review. — Lomn | Talk / RfC 20:55:30, 2005-08-31 (UTC)


Any way to learn statastics without senseless jargon or bad english?[edit]

Most statistics books seem to be written by people who lack basic descriptive skills or have yet to have agood grasp of English writing. Here we see a statistic textbook explaining cummulative frequency, a complex name for an extremely brain-dead operation, and explains it with great complexity:

"A cummulative frequency distribution can be created from a frequency distribution by adding an additional column called "Cummulative Frequency." For each score value the cummulative frequency for that score value is the frequency up to and including the frequency for that value."

Is there any books, or other sources anyone can recommend to me that will not try to complicate this very simple subject?

Let's paraphrase:
  1. Take a graph. A special graph called a "frequency distribution".
  2. Let's take, say, the graph of all the edits done on the reference desk...(see below)
Total number of edits made to the Reference Desk before the page split, grouped by month.
  1. Each bar represents the frequency of edits done per month.
  2. Now, draw another graph, with the same horizontal axis. (I just replaced the month names with numbers here instead)
  3. This time, each month's height corresponds to the total number of edits done up to that month. When you do this, you get something that looks something like this:
Total number of edits made to the Reference Desk before the page split, grouped by month.
This second graph is the "cumulative frequency distribution". Each bar represents the "total up to that date." Sometimes, it's called the "cumulative distribution function" if it's normalized. Hope this helps! --HappyCamper 03:09, 1 September 2005 (UTC)[reply]

Orthopedic Surgery[edit]

Is it a good idea to leave orthopedic hardware (9 screws and a metal plate) in the proximal humerus after the bone healed and maximum cure attained?--anon Alexander

That'd be a question for your surgeon.
I can tell you that I know someone who had screws left in his bones for more than 30 years, and the only problem was that they had to be removed when the area needed another operation. After that long they were a bit hard to get out, and the extra hoops (waiting for it to heal up a bit before the next, more involved operation) dragged the whole process out.
Another friend has told me that all the hardware in his legs has made running painful as the metal does not absorb impact as well as bone. I'm sure there are other potential problems that depend on exactly what is left in where.
Your doctor will know your situation much better than us here, so ask him to go over the pros and cons with you. — Laura Scudder | Talk 04:26, 1 September 2005 (UTC)[reply]
From our personal experience, the hardware of which you speak (titanium) and in the same location of your concern has had no adverse effects after five plus years. In fact, removal would present an unnecessary risk. I'm speaking to adult and not pediatric bone growth concerns. The only side effect for Heidi seems to be at airport security. ;-) hydnjo talk 21:05, 2 September 2005 (UTC)[reply]

Slug fest![edit]

I live in Bend OR. I recently witnessed 2 slugs doing their duty, ( I think!) But all I could see coming out of the aftermath were more slugs.I saw protruding tentacles, slime, ect., but no egg sack. It looked like live birth. They wrapped around each other, explored, ( as good sex should be), but then there was another slug...and another. It was beautiful, but disturbing. Did we witness hatching, or what????Do they come back to the scene of the crime? It was an intimate moment, and it looked tender ( who thought slugs could be tender), But what the >!@?":.....

The slug article does have a section on their reproduction (Slug#Reproduction_and_life_cycle), and it does say that they wrap around each other like you describe. However they apparently lay eggs in the ground, and don't have live births. Thryduulf 09:00, 1 September 2005 (UTC)[reply]

A question of Gravity[edit]

Hi there-- recently I have been finding out about gravity and a question keeps itching me. If you were to drain half of the magma from the earths core would the remaining liquid rearrange itself into a perfect sphere centred around the point at which gravity for this planet is generated?

In case that was slightly confusing what i mean to ask is that gravity is a single point in the exact centre of our planet so if there was just an open void around that point and something was placed on that point would it just float exactly still as forces are acting equally uopn it in every directon?

The gravity of our planet is "generated" in each and every part of it. When you add up all these minute gravitation pulls you get what we experience as gravity.
Assuming no other objects exist apart from the earth, and the earth was made hollow (and it didn't crumble), and the earth was a perfect sphere, inside the earth there would be no gravitational field or pull. As you get closer to one side of the sphere you get closer to the particles and therefore their pull on you will be stronger, but there'll be less particles in that direction of you. This evens eachother out. As far as I know, the first person to come up with this was Isaac Newton in his Principia.
So, to answer your question, the magma would do whatever it damn well pleased, i.e. there would be no gravitational force working on it, though there could be other forces. --fvw* 10:51, September 1, 2005 (UTC)
See also Hollow Earth. Thryduulf 11:02, 1 September 2005 (UTC)[reply]


Thanks for the answer but it leads me to another question. If every part of the earth produces a gravitational pull and these minute pulls combine to form what appears to be one force known as gravity why is it that we are not pulled slightly towards huge clusters of particles like say a mountian and also why is it that all of these forces seem to pull towards one point in the centre of the earth?

We are pulled slightly towards mountains, but AIUI the force is so weak that it is only measurable by scientific instruments. It takes the mass of something on the scale of a planetary body to produce a force strong enough for us to feel, and this is why we appear to be pulled towards one central point. Thryduulf 15:26, 1 September 2005 (UTC)[reply]
(edit conflict with Thryduulf) In fact, they don't pull you exactly towards the center of the Earth. Measuring local deviations in gravity is a huge part of geophysics and is a way to get information about the density of material under the surface. See gravity anomaly and geoid. — Laura Scudder | Talk 15:33, 1 September 2005 (UTC)[reply]

Oh and your comment about getting closer to one side of the sphere and the force growing stronger but the lack of particles meaning they even each other out. What is that supposed to mean? These particles that are fewer as you move closer to one side, are they supposed to be air particles because thats the only thing i can imagine moving through as easily as you say. And even each other out!!?? If the forces evened each other out wouldn't you be stuck in your position unable to move unless you can produce a force strong enough to move in a direction?

Basically, if you are to be pulled downwards by gravity, then the particles exerting the gravity need to be below you. When you are in the centre of the hollow sphere there is more of the sphere below you than there are when you are closer to the edge (as there is more of the sphere beside and above you). If all the forces evened each other out you would be stuck as you describe, however the force required to overcome them wouldn't be difficult - for example, consider how easy it is to move under the force of gravity from a body the size of planet earth. Thryduulf 15:26, 1 September 2005 (UTC)[reply]
What he was saying is that you're closer to a small amount of the Earth's rock, but farther away from a large portion of the Earth's rock. So less rock is pulling on you in one direction (but there's a stronger pull per bit of rock because you're closer), while more rock is pulling on you in the other (but there's a weaker pull per bit of rock because you're farther away) and they end up cancelling eachother out. This accounts for why you don't feel any net gravitational force inside a hollow sphere. A mathematical way of proving this uses Gauss's law. — Laura Scudder | Talk 15:33, 1 September 2005 (UTC)[reply]

One more thing, if 'particles' of whatever it is are responsible for gravity then is a black hole just an unspeakable number of them clustered together?

See teh Gravity and Black hole articles. A better answer is beyond my understanding! Thryduulf 15:26, 1 September 2005 (UTC)[reply]
Yes. A black hole is what you get when so much matter is in one place that it all collapses under its own weight, into a singularity - a point so small it cannot be measured. The gravitational attraction of this is, as you'd expect, pretty big, since it requires a lot of matter - the absolute minimum mass to form a black hole is given by the Chandrasekhar limit, which is about one and a half times the mass of the sun. Shimgray 16:06, 1 September 2005 (UTC)[reply]

Thanks guys

Where can I find information on James De Politerno who uses massage therapy?[edit]

i dont now

Deep yellow sky[edit]

The view in question.

Hello, everyone. I am writing to ask about something that I saw last night. After a rather grand storm, in which all light faded, suddenly not only the sky above, but all the space outside, became a disorientating, deep chemical yellow (see the picture to the right). Walking outside was like walking into yellowness. After about twenty minutes, during which time the yellow became darker, the light shut out, and we were in total darkness. I have never before witnessed such a thing. Do you know what this occurance is called, and what causes it? I would appreciate any help. IINAG 13:08, 1 September 2005 (UTC)[reply]

Well, in Texas we just say it means there's going to be a tornado with the storm, but I have of course no scientific backing for that and it doesn't help with the why. — Laura Scudder | Talk 15:38, 1 September 2005 (UTC)[reply]
Hello, Laura; thank you for your response. I had never knew before about there being a yellow sky with a tornado. The only thing is that I live right in the middle of England, in an area not noted for tornados (however, there was a small tornado in a nearby city, Birmingham, although this was a freak incident.) That has me wondering whether the same reason(s) that make(s) the sky yellow before a tornado acted when the air yellowed yesterday. IINAG 16:56, 1 September 2005 (UTC)[reply]
I'm trying to think of a plausible explaination for the yellow. It may be due to the storm kicking up an unusual amount of dust, and it being illuminated indirectly, by light that had already been depleted of blue by Rayleigh scattering. -- DrBob 17:37, 1 September 2005 (UTC)[reply]
It must be associated with some change in weather. I lived in Kansas for thirteen years and once in a long while something like you described would happen. The sky would rapidly adopt a very strange yellowish color. Often it was also strangely warm, and as Laura mentioned a decent indicator of an approaching tornado.
Just to clarify: yellow sky -> tornado, but tornado !-> yellow sky. In other words, as the former Kansas resident says, this is rarer than tornados so it must be related to a rather particular set of storm conditions. I found this article, which tries to explain it. — Laura Scudder | Talk 14:57, 2 September 2005 (UTC)[reply]
Thank you all for all of your help; that article is quite useful. The thing that confused me was that, ostensibly, there was no change between the weather before the yellowing and after it. Before it, it was dark, and it was raining heavily; after it, it was dark, and was raining heavily. IINAG 00:56, 3 September 2005 (UTC)[reply]
I'd like to point out that I live in Michigan, and saw this once. It wasn't quite as yellow as the picture you showed though! It was a bad storm, but I have seen worse. We did not have a tornado, but I have heard of that before. --Phroziac (talk) 17:12, September 3, 2005 (UTC)

Using an Ext2 NBD image under Windows[edit]

I occasionally play a little with Linux on my GameCube. For that, I have a 1GB harddrive image for use with a Network Block Device server.

However, I occasionally wish to manipulate this image without having to boot the Cube into Linux - injecting files, etc. Does anyone know if there are any tools that will let me do this? It doesn't matter if the image is mountable as a drive, or if I need to use a seperate program that mounts the image internally. --Pidgeot (t) (c) (e) 13:35, 1 September 2005 (UTC)[reply]

Search for "ext2 windows" on google. The second links looks good, but I'm sure there are several other programs if you don;t like that one. --R.Koot 15:31, 1 September 2005 (UTC)[reply]
Actually, I already have the program at the second link installed, but it only works with physical harddisks - however, Explore2Fs, the first link, did the trick - for some reason, I never thought of the image as a regular ext2 disk. >_<
So thanks for the wake-up call, I needed it ^_^;;; --Pidgeot (t) (c) (e) 15:44, 1 September 2005 (UTC)[reply]

Word to PDF[edit]

Is there any simple (free) way of transforming a word document to a pdf? I tried converting to OpenOffice, but their pdf converter makes the document ugly. Why doesn't Word have its own converter?

Thanks! --Alice

Adobe sells a PDF plugin for Word - it's not bad. The problem is that it's expensive - PDF is a proprietary file format, unlike (say) HTML, so Adobe take what the market will bear. There are a variety of free converters out there, though - do a search on doc to pdf or word to pdf - but they'll take some fiddling to work.
Also, it's quite possible the flaw lies with Word and not OpenOffice; Word uses a lot of odd proprietary formatting tricks, and often doesn't convert very well to other formats. So you may be unable to transfer some of the formatting regardless. It may be simpler to save to an RTF file, then open that through OpenOffice. Shimgray 16:02, 1 September 2005 (UTC)[reply]
It wouldn't surprise me if it *is* OpenOffice that's to blame - in my experience, it's very bad at PDFs, even with documents never opened in Word.
I suggest downloading pdf995 ([1]), a printer driver. It lets you export any printable document to PDF by printing it through a special printer. You'll have to live with it popping up a "registration reminder" everytime you make a PDF with the free version, but it's really no big issue, and you can just keep on using the free version as long as you want, there are no limitations. --Pidgeot (t) (c) (e) 16:11, 1 September 2005 (UTC)[reply]
PDF actually is an open format. Have you tried OOo 1.1? If found that its PDF converted does a much better job than OOo 1.9/2.0. --R.Koot 16:25, 1 September 2005 (UTC)[reply]
  • There's also a program called CutePDF, but the people who told me about that weren't all that happy about it. Still worth a try maybe. - Mgm|(talk) 17:39, September 1, 2005 (UTC)

Hi all, thanks for your replies.

I tried Pidgeot's suggestion and it seemed to work well. However, it changes the formatting o my document. I've got a 40+ page paper, with chapter headings at the tops of pages and References at the end. There appear to be a different number of lines per page in the pdf, which makes my headings appear in varying places down the page, and the references after 1 1/2 blank pages. How can I get it so that the formatting is the same as in my word document (and yes, I'll learn latex for the next paper!).

Thanks, --Alice

Alice, one free (but somewhat inconveient) way around this is to install a postscript printer driver (you can download one for free from Adobe, IIRC), print your document to a postscript file, and use Ghostscript to convert the postscript to PDF - it comes with a utility called ps2pdf for just this purpose. Your mileage may vary...--Robert Merkel 23:26, 1 September 2005 (UTC)[reply]
If pdf995 changed the formatting of your document, it may those changes would have been there anyway. If you were printing from OOo, that could be why. Use print preview (in Word!) to make sure it isn't a formatting error somewhere in your document.
You can also try the print->PostScript->PDF solution mentioned above, but since that's basically what PDF995 already does, it might not help. --Pidgeot (t) (c) (e) 04:48, 2 September 2005 (UTC)[reply]

Speaking of pdf, I have scanned some pages of a document and saved it as a png file (resolution 800px). I want to stitch all these pages as a single pdf file. How do I do this? =Nichalp «Talk»= 06:34, September 2, 2005 (UTC)

There are a few solutions to this, none of them necessarily easy. If you have a full copy of Adobe Acrobat available, it woudl be pretty easy, but if you are asking on here you probably don't have one. If you are using an Mac running OSX, you could "print" them as PDFs (see below) individually and then use Automator, I think, to add them into one big file. The other option I know of involves using the PDF drivers which come with PHP to stitch them together, but this takes a fair amount of coding knowledge (I've done it with TIFs before). --Fastfission 19:28, 2 September 2005 (UTC)[reply]
No, I don't have access to a Mac :(. I use Win and have a limited knowledge in Linux. What if I embed the images in each page and use OO to 'print' the pdf's? I won't lose the formatting as it is a png afterall. =Nichalp «Talk»= 05:51, September 3, 2005 (UTC)

Alice: If you are working on, or have access to, an Apple Macintosh running Mac OS X, and which also has a copy of Word (perhaps your local library or uny computer lab?), it's easy: Open the file up with Word on the Mac, select "Print" from the "File" menu, then click the "Save as PDF" button. The PDF will look more or less exactly like how the document would look if you printed it from Word. Viola! Yet another reason to love the Macintosh. Garrett Albright 06:53, 2 September 2005 (UTC)[reply]

My favorite utility so far for PDF creation is PrimoPDF. It's a wrapper around GhostScript to make it act as a printer driver, and it's all open source with no registration reminders (unlike CutePDF). --David Wahler (talk) 17:52, 7 September 2005 (UTC)[reply]

Help! Homework with physical science![edit]

What are some accurate methods you could use to determine the identity of a sample of a white solid? --anon.

your title is missleading, this isn't physical chemistry, this sounds like a general chem lab, at best--64.12.117.12 02:10, 4 September 2005 (UTC) You didn't read the title carefully, 64. Physical science is not the same as p chem. In the US, physical science is often an early secondary school subject that is a less rigorous combination of chemistry and physics for those who will not be taking them individually. Our questioner will likely never know what p chem is. alteripse 04:22, 4 September 2005 (UTC)[reply]
  • Solubility in water and other solvents
  • pH in solution and reactions to acid or alkali
  • Microscopy to look at crystal shapes
  • The sequence of procedures of qualitative analysis (organic or inorganic)
  • X-ray diffraction analysis
  • Mass spectrometry
  • Reaction of animal trained to react to drugs or explosives
  • Feed it to a guinea pig or other, even better, a disposable, non-cute and non-furry, animal
  • Taste it, like Alice did
  • Circumstances of how it came into your possession (location, container, amount, accompanying objects, labels, etc)
  • Ask the owner or provider of the sample

alteripse 03:44, 2 September 2005 (UTC)[reply]

I'd go for taste if it wasn't for the potentially deadly results. If you disolve it in water and it the water can now transfer electricity it's a salt. If you smell cloride it's a cloride salt. You can then add other water solutable salts to test whcih to ions the salt was composed of (look in the chapter Salts in your chemistry book for the exact procedure). --R.Koot 03:55, 2 September 2005 (UTC)[reply]

I was once told that Humphry Davy tested for taste of every new substance he discovered. I was surprised he survived, given that he isolated Sodium and Potassium among others. DJ Clayworth 19:41, 8 September 2005 (UTC)[reply]

Also flame test and borax bead test. These will help identify the substance, if only to show what it isn't, since some substances do not give helpful results. Melting point is another useful tool, as well as density.

Speed of Sound[edit]

Hi, I know this might not be technical enough to warrant your attention but do you happen to know the exact speed of sound in MPH, KPH and Knots? I know its around 750MPH but i need to be sure in order for my calculations to be correct. Ryan

The speed of sound is not a constant. It highly depends on the material that carries it and the temperature of that substance. --R.Koot 10:06, 2 September 2005 (UTC)[reply]
See speed of sound for some more information. --R.Koot 10:07, 2 September 2005 (UTC)[reply]

In that case can you tell me which sequence of variables they use to determine the top speed of planes ie. If a company states it's plane can go Mach 2 then which speed are they referring to as it could be anywhere between 1200mph and 1400mph?

I would bet that the Mach number is a more accurate measure of the limit as a plane's top speed will also change with conditions. — Laura Scudder | Talk 15:41, 2 September 2005 (UTC)[reply]
As stated in the Mach number article "Mach number (Ma) is defined as a ratio of speed to the speed of sound in the medium in case." It would seem that the reference speed of sound is that in which the plane is flying. hydnjo talk 19:37, 2 September 2005 (UTC)[reply]
There are a few directions you can go with this (since the speed of sound, even if you fix the medium as "air", will vary):
  • speed of sound at a standardized altitude (say, sea level)
  • speed of sound at a nominal cruising altitude, not necessarily standard across aircraft types
  • speed of sound at the best possible, albeit extremely unlikely, conditions
From a marketing standpoint, guess which is most likely to be the case :) — Lomn | Talk / RfC 21:14:14, 2005-09-02 (UTC)

Just to complicate things, note that a plane (a jet, as prop plane are always subsonic, I think) will go faster in rarefied air, because of reduced friction, but sound will tavel slower in that medium. There is a law of diminshing returns here, as rarefied air will provide less oxygen to the jet engine and it will produce less power.

Equaling out speeds[edit]

Hi,

I recently read an article on a similar site that stated; if you were on a train travelling at 1000mph (and the ride was so smooth you couldn't notice) and you fired a bullet off the front of the train (in the exact direction it was travelling) then the bullet would be travelling at 2000mph relative to a stationary point on the ground and at 1000mph realtive to your position (moving at 1000mph). This I can believe, but it also states that if you fired a bullet under the same circumstances off the back of the train (this is all theory, I'm not taking into account aerodynamics or anthing) that the bullet would be travelling at 2000mph relative to your position on the moving train but it would be travelling at 0mph in relation to a stationary point on the ground. This suggests that the bullet would float in that air on then just fall to the ground. Is that correct because it seems very unlikely?

Yes it is true, however... The bullet would not float in the air, but immediatly start to fall to the ground. If the bullet and the train are travelling at the same speeds, but in opposite directions, it will fall on the ground, exactly were you shot it. You will be quite a distance away however, because you are traveling at such a great speed.
You can test this yourself with a heavy object. Hold object, start running, then rlease it (don't throw it) it should travel along you for a brief moment, and will not fall on the floor were you released it. Do it agian, but trhrow the object in the opposite direction of which you are running, with a bit of tuning you can make the object land at the spot were you threw it. --R.Koot 11:33, 2 September 2005 (UTC)[reply]
I completely agree with you. This can be explained on the basis of the fact that motion itself is a relatie term. It depends on the frame of reference when you percieve an object to be in motion. By frame of reference i mean your position with respect to the object in motion (like the train and the stationary point). Most of the time the apparent speed of the object can be calculated using simple addition of vectors. --StratOnLSD 11:58, September 2, 2005 (UTC)
I think the air behind this 1000 mph train would be somewhat disturbed by the passage of the train, and more than able to whip a bullet around all over the place. So the actual result wouldn't be as simple as "floating in the air". Notinasnaid 12:03, 2 September 2005 (UTC)[reply]
yes, it would just drop to the ground (apart from being sucked along a bit in the train's wake, but that would not be very much anyway), but it would not travel "2000mph relative to your position on the moving train"; only 1000 mph. It is a 1000mph bullet.

Thank you to R.Koot and StratOnLSD (interesting name) for those answers, and to Notinasnaid; I believe if you read my question you would understand I was disregarding any other forces, variables and factors not covered in my question and if technicality is your specialty (which it appears to be) then you shouldn't have answered a question about a train moving at 1000mph.

Also, if you do three things simultaneously: fire the bullet forwards, fire the bullet backwards (both exactly horizontally), and drop the bullet from your hand, all three bullets will hit the ground at exactly the same time.
Another interesting result (again assuming no air resistance or other complicating factors), if you shoot the bullet directly upwards from the moving train, it will come back down at precisely the same point on the train. -- DrBob 16:38, 2 September 2005 (UTC)[reply]
Which has got to make one wonder why shooting bullets up into the air is such a common form of celebration in some cultures. Has no one ever been indirectly shot by a falling bullet? hydnjo talk 19:02, 2 September 2005 (UTC)[reply]
I'd think it's happened at some point. Similar injuries occured during WW2 as a result of shrapnel from anti-aircraft barrages eventually having to fall to the ground -- since those were usually over large cities, the chance of injury was a good bit higher. — Lomn | Talk / RfC 21:10:39, 2005-09-02 (UTC)
I don't have the details to hand, but IIRC there is at least one recorded case of an aircraft being hit by its own bullets - a modern jet, on a gunnery range, fired some ammunition and then went into a shallow dive, picking up speed. By a combination of a miscalculation and some very bad luck, it accelerated underneath its own falling shells... Shimgray 03:20, 3 September 2005 (UTC)[reply]
Yes, this definitely happens. I read an article about someone dying from gunshots into the air in a wedding somewhere in the mideast. Luckily most of the cultures that do that are in more sparsely populated areas. One specific example, my Aunt's store had a bullet come down at a very steep angle through the window in the front of the store. It shattered the window and a number of glass items in displays. I can't remember how far away they estimated the bullet was from, but likely within a mile or so. The other addition to DrBob's point, is that not only does the bullet come down in the same place if fired straigh up with no wind, but even with air resistence, it comes down at essentially the same speed it left the barrel with. fast enough to kill. - Taxman Talk 18:22, September 3, 2005 (UTC)
I very much doubt this. The Mythbusters proved you can't be killed by a penny dropped from a building, so I doubt you can be killed by a bullet falling from the air. It will be travveling at a much slower speed than when it was shot at you, due to air friction limiting the bullet to a certain speed. --R.Koot 19:52, 3 September 2005 (UTC)[reply]
They explained it pretty well below. The deaths part, and the exact speed coming down I did not look up, but the part about my aunt's window was unequivocably confirmed by finding the bullet that did the damage, and being able to discern the path it took. - Taxman Talk 16:10, September 6, 2005 (UTC)
It's also, however, more aerodynamic and heavier. IIRC, the Mythbusters conclusion was that pennies lacked sufficient mass more than they lacked velocity. — Lomn | Talk / RfC 01:46:37, 2005-09-04 (UTC)
Point 1: bullets are relatively aerodynamic, but air resistance still slows them down hugely and quickly. Take a look at the data table in external ballistics - even though this bullet has a high ballistic coefficient, it sill looses one third of its initial velocity in about two thirds of a second. This is an average decelleration of about 400 m/s^2 i.e. about 40g. Point 2: the terminal velocity of a falling bullet is between 60 and 100 m/s (depends which source you look at - probably depends greatly upon type of bullet, whether it tumbles etc.). Fast, but nowhere close to its muzzle velocity. Point 3: even so, are people killed by falling bullets ? Yes, for example it happens regularly in Los Angeles, according to the sources quoted in this article. Gandalf61 11:29, September 4, 2005 (UTC)

Need power formulas associated with electrical generators[edit]

Hello!

I'm in need of power formulas associated with both delta and wye connected AC generators.

Can anyone help or provide links?

Thanks, Rick Gilbert

Presuming a balanced three-phase resistive load:

DELTA

Vdelta = Vline

Iline = √3 Idelta

Ptotal = 3 Vdelta Idelta = √3 Vline Iline


WYE

                                                                     Vwye = √3 Vline                              

Iwye = Iline

Ptotal = 3 Vwye Iwye = √3 Vline Iline


Yoke

How do they make elbow macaroni curve?[edit]

I got stumped working on the Elbow macaroni article. They make macaroni by extruding dough through a circular die with a pin blocking the center of the circle. I see a number of references that say that a "groove" in the pin causes the pasta to curve as it is extruded. How? What does this groove look like? Why doesn't it form a ridge or valley on the finished product? Bunchofgrapes 17:11, 2 September 2005 (UTC)[reply]

It has to do with relative rates of extrusion. Pasta machines work by forcing pasta dough through a die, which simplistically speaking is just a negative silhouette of the desired shape. But if more dough is forced through one part of the die than another part, the extrusion there will happen faster, and the extruded pasta will curl.

A picture will make this much clearer. Here is the front view of an elbow macaroni die:

This is about 30cm in diameter, and as you can see it extrudes 5 tubes of pasta at once. But now let's look at the back:

The ridges you see divide the total output of the pasta machine (that is, the stream of pasta dough which this die is the target of) into ten "capture regions". (There's a rather nice analogy here to watershed.) Two regions flow into each extruded macaroni tube, but significantly, one of them (the outer one) is considerably larger than the other. Therefore, the outer edge of each tube will extrude faster than the inner, and each one will therefore curl inwards.

I haven't seen one, but I presume that the die for curly-edge lasagne consists of the long straight slit you would expect, but with some extra "capture region" at each end of the slit (corresponding to each edge of the lasagne ribbon). In this case, the extra dough volume is too localized to cause a curl, but it has to go somewhere. The edges end up being "too long" for the main body of the ribbon, so the edges end up rippling to consume the extra material.

Steve Summit (talk) 04:36, 6 September 2005 (UTC)[reply]

New Orleans City Construction History[edit]

How were portions of New Orleans built below sea level? Did these areas sink? Did they drain and/or excavate after building levees? Are below-sea-level-portions of the city built on drained parts of The Mississippi River or Lake Pontchartrain?

Any information would be much appreciated.

Thank you,

Neil Higgins

The answer is complex. Start with the Wikipedia article New Orleans, Louisiana and then take a look at Hurricane Katrina, Effect of Hurricane Katrina on New Orleans and Damage to infrastructure by Hurricane Katrina. I've found a lot of historical reference within these articles that address your questions. hydnjo talk 19:47, 2 September 2005 (UTC)[reply]

A Slot Machine[edit]

A slot machine has three wheels: Each wheel has 11 positions – a bar and the digits 0, 1, 2,…, 9. When the handle is pulled, the three wheels spin independently before coming to rest. Find the probability that the wheels stop on at least one bar.

I have two answers. I think the first one is right, but I do not understand why the second one might be wrong.

P(E1) = , where I subtract the probability of getting no bar three consecutive times (10/11)3 from 1.

P(E2)= , where I add the probability of getting one bar, two bars, and three bars.

Can you explain this discrepancy? Why do these two answers not conform to each other?

--anonym

The second method you used, while the idea of adding up the probablity of getting one, two, and three bars will give you the correct answer, you have not done so correctly, and it is much simpler to take the probablity of getting no bars and subtract it from 1. Your first method is correct. Y0u (Y0ur talk page) (Y0ur contributions) 21:36, September 2, 2005 (UTC)


To expand on Y0u's answer. The probability of getting one bar is

i.e. the probabliity of a bar on the 1st multiplied by a not bar on the other two multiplied by three (because the bar could be on any of the three not just the first one).

Likewise the probability of two bars is

Theresa Knott (a tenth stroke) 22:21, 2 September 2005 (UTC)[reply]

Old[edit]

I recently saw a commercial for Old Navy where folks were picking Old Navy products from a field of some kind of gourd-like plants, handsome and erect with big leaves. Does anyone know what kind of plants those are? Attractor 00:08, 3 September 2005 (UTC)[reply]

$12 Jean plants!!!! Ilγαηερ (Tαlκ) 01:12, 4 September 2005 (UTC)[reply]

Film synopses for the visually impaired[edit]

Does anyone publish detailed scene-by-scene film synopses that fill in plot gaps that you can't get by listening alone? Attractor 00:08, 3 September 2005 (UTC)[reply]

There's also The Movie Spoiler, which might be useful. --Sum0 18:32, 15 September 2005 (UTC)[reply]

Expectation to Open a Safe[edit]

A safe containing $1,000,000 is locked with a combination lock. You pay $1 for one guess at the six-digit combination. If you open the lock, you get to keep the million dollars. What is your expectation?

Payoff Probability
$106
(-$1) (719/720)

Thus, . Am I wrong?

--anon


.

--anonym

If there are 1000000 possible combinations (assuming 6 decimal digits), then $1000000/1000000 - $1, and if you get to try again, $1000000/999999 - $1 and then $1000000/999998 - $1 the next two times. Κσυπ Cyp   06:09, 3 September 2005 (UTC)[reply]

Modeling on a Graphing Calculator[edit]

In the World Series, the top teams in the National League and the American League play a best-of-seven series; that is, they play until one team has won four games. (No tie is allowed, so this results in a maximum of seven games.) Suppose the teams are evenly matched, so that the probability that either team win a given game is.

I would like to model the World Series with a program on my graphing calculator (TI-83), where “heads” represents a win by Team A and “tails” a win by Team B. Then I want to use the program to estimate the probability that an evenly matched series will end in four, five, six, and seven games. Even though I did a little Visual Basic programming about five years ago, I am clueless.

--anonym

the TI-83 interface is probably too primitive for you to do that, but if you really want to try, you might try looking up the rnd(0,1) command, random number generator, if I remember right.. nothing like Vb really, more like QB--64.12.117.12 02:08, 4 September 2005 (UTC)[reply]


  • I know that rand generates . All the programming commands are in the catalog menu or the PRGM menu. The same instructions should be executable on all Texas Instruments calculators, id est, they are the same. --anonym
  • Not being a TI user myself, I can't really help - but it may be helpful to know the way of calculating a random number when rand generates a number between 0 and 1 is to multiply the number by the upper end of your range (so if you wanted numbers from 1 to 10, RAND*10) and then round to 0 decimal points. Additionally, if you want to increase the lower end, try eg. RAND*5 + 2 for random numbers between 2 and 7.
  • If you want integer values though, the TI-83(+)[possibly the regular version too] can do that directly. Use randInt(near rand). Syntax is randInt(1,10) to generate an integer from 1 through 5(inclusive).
  • If you just want to know the probability of the series ending in 4,5,6, or 7 games, it's easier to do it mathematically.
    • 4:(1/2)^4*2=1/8
    • 5:(1/2)^5*4*2=1/4
    • 6:(1/2)^6*10*2=5/16
    • 7:(1/2)^7*20*2=5/16
      • if I'm not mistaken. Tell me if you want a more in-depth explanation. The probability of any win-loss combo is (1/2)^(num. games). Then you multiply by the number of possible combos for each situation. You multiply by two at the end because the quantity before that is the probability of one of the teams winning in that number of games but you want to find the probability of either of them doing it. For example, there are only two ways a series can end in four games. Team A wins four and a row or Team B wins four in a row. The probability of either is 1/16. Hence, the probability of a four game series is 1/8, the sum. However, this is all probably a huge waste of time because your teacher is requiring you to do the modelling. If that's the case, I'll provide a few hints. First: To decide who wins a game, generate a random number with rand. If it's less than .5, give it to the National league. If it's greater than or equal to .5, give it to the American leaguers. I.E.

If(rand<=.5) Then Disp "National win!" Else Disp "American win!" End

If, Then, Else, and End are in the CTL(control) menu while programming. <= is in the Test menu. Press 2nd, Math, then select it. If you need more help, tell me and I'll see what I can do. Superm401 | Talk 23:03, 13 September 2005 (UTC)[reply]

  • I was going to give up on this problem and wait until I learn about programming and then do it if I ever get a chance in the coming years. I thoroughly understand the mathematics behind it and I spent over an hour constructing the table below it. Here is my nonfunctional guess.
BASEBALL
0-->A:0-->B
For(N,1,4)
Rand--->X
Int(2X)--->Y
A+(1-y)-->B [(1-y) is either 0 or 1.]
End

Again, I am clueless about programming and this is giving me hints is not likely to be fruitful. Yet, I am meek and am not demanding an answer. If you can, then please write the program. Also, I neither attend any school nor is this a homework. I am an old adult who is doing this for satisfaction. This programming problem is from an elementary mathematics textbook. --anonym

World Series Probability[edit]

In the World Series, the top teams in the National League and the American League play a best-of-seven series; that is, they play until one team has won four games. (No tie is allowed, so this results in a maximum of seven games.) Suppose the teams are evenly matched, so that the probability that either team win a given game is .

What is the probability that the series will end in four games? Five games? Six games? Seven games?

--anonym

And how far with this homework have you got all by yourself? Theresa Knott (a tenth stroke) 16:32, 3 September 2005 (UTC)[reply]

  • I do not know how to start. --anonym.

Does P(4 games) =

--anonym

Nope. For P(4 games) you need to have one of the two teams win all four games in a row. (There is no other way it can happen). Call the teams A and B. So I reckon (and I'd like people to check my reasoning please) that

Where means the probability that side A wins game 1 and so on.

Now all these probablilities = 1/2 so that gives P(4 games) = ?

It's harder as you go to 5 games because you need A to win 4 and lose one and there are 5 different ways this can happen (do you see why?) Theresa Knott (a tenth stroke) 22:34, 3 September 2005 (UTC)[reply]

Actually, there are only 4 ways for this to happen... You don't play game 5 if you win the first 4. Similar conditions must be imposed on the 6- and 7-game scenarios. — Lomn | Talk / RfC 22:36:46, 2005-09-03 (UTC)
Oh yes you are right! Theresa Knott (a tenth stroke) 22:39, 3 September 2005 (UTC)[reply]
Technically, only the 5- and 6-game scenarios need such conditions, as P(7)=1-P(6)-P(5)-P(4). --Pidgeot (t) (c) (e) 22:42, 3 September 2005 (UTC)[reply]
True. The conditions do exist, but there's no need to run the calculations involving those conditions. — Lomn | Talk / RfC 23:04:23, 2005-09-03 (UTC)
To the anon - can you see why that's true? I have to say your teacher set a pretty difficult HW but if you study it enough you'll learn a lot. Theresa Knott (a tenth stroke) 22:45, 3 September 2005 (UTC)[reply]


  • We shall consider that team A wins. We can double the probabilities for either team winning. Moreover, we will denote the wins of A win W and the losses of A with L. We also should remember that the last game must be a win to finalize the World Series.
# of Games # of Sequences Probability for A Total Probability
4 WWW,W=
5 LWWW,W=
6 LLWWW,W=
7 LLLWWW,W=


  • (The total probability for each game)=2×(probability for A). Note that the sum of all the total probabilities is one. My table on the Edit page does not show, no matter what I do.

The last game MUST be a win


Number of games    No. of Sequences         Probability
---------------     ---------------        -------------
     4              WWW|W  = 1           (1/2)^4  = 1/16
     5              LWWW|W = 4        4 x (1/2)^5 = 1/8
     6             LLWWW|W = 10      10 x (1/2)^6 = 5/32
     7            LLLWWW|W = 20      20 x (1/2)^7 = 5/52

The total probabilities will be double the probabilities shown, So

Series ends after 4 games   =  1/8
   "    "         5   "     =  1/4
   "    "         6   "     =  5/16
   "    "         7   "     =  5/16
                         -------------
                      Total =  1.00   (so this checks)


--anonym

HTML equivalent of {nowiki} {/nowiki}[edit]

What is the HTML equivalent of {nowiki} {/nowiki}? --Commander Keane 10:44, September 3, 2005 (UTC)

One way to do it is using <pre>, but that gives you monospaced text (unless overridden in your CSS). I know I've seen another tag that works *exactly* like nowiki, but I'm afraid I can't find it anywhere right now - it *may* be an unofficial extension of the HTML spec, I'm not sure. --Pidgeot (t) (c) (e) 10:54, 3 September 2005 (UTC)[reply]
Unfortunately, I didn't remember correctly. The other tag is <xmp>, but that's an obsolete element that works just like <pre>. --Pidgeot (t) (c) (e) 11:24, 3 September 2005 (UTC)[reply]
Note that PRE is intended to treat whitespace literally. It does not suppress the interpretation of HTML tags and entities. There is a small set of inline elements that are permitted within PRE, although browsers seem to tolerate almost anything. See this website for more details. Bovlb 14:44:37, 2005-09-03 (UTC)

You could use &lt ; and &gt ; tags like this <html tag > Theresa Knott (a tenth stroke) 16:29, 3 September 2005 (UTC)[reply]

Keane, could you perhaps give us a little bit more explicit idea of what you're trying to do? Garrett Albright 17:20, 3 September 2005 (UTC)[reply]
I was trying to ask about the use of "&nbsp;" here, which I acheived by swapping the usualy "&" with "&amp". I've done it now, and I'll try to be more specific next time I ask a question. Thank you. --Commander Keane 17:36, September 3, 2005 (UTC)
You have to use the "&amp" trick even with nowiki, it's a real pain until you discover it... Physchim62 20:38, 4 September 2005 (UTC)[reply]

It's <![CDATA[...]]> (yes, it's very obscure, unless you are a XHTML user, where it's used all the time with <script> and <style>). Much like <nowiki>, it can be ended only by the exact sequence "]]>". --cesarb 17:56, 3 September 2005 (UTC)[reply]

And, as expected for Wikipedia, there's already an article about CDATA, with the wrong case in the title, being a stub, and with incomplete information. I'll see if I can make it better. --cesarb 18:01, 3 September 2005 (UTC)[reply]
And it's also an obvious copyvio. Brilliant. --cesarb 18:05, 3 September 2005 (UTC)[reply]
Found a better article at CDATA section and redirected there. --cesarb 18:19, 3 September 2005 (UTC)[reply]
FYI, I've used CDATA before, but never had any luck getting it to validate under XHTML 1.1 Strict, not that that matters ;) splintax (talk) 14:20, 12 September 2005 (UTC)[reply]

Solar System[edit]

     To whom this may concern, 

I am typing this question to you hopping that i will be takin seriously. I have taken a course in collage on chemistry, and have started to take an other course in solar systems. When i sat down i took a look at a cell make up and looked at the whole solar system. if you look at them they do have a similar look to them. Now the only difference that i can make out is that the levels of electrons on the solar system does not consist of the same law.... or does it. when we look at a cell through a microscope we shed light to the whole area of the cell. Now if we where to take that light and project it on an angle or completely to it's side would the electrons cast a shadow to the other electrons and unable us to see the other electrons needed make the cell function to what our law says. Now if we think of the Solar System the sun sheds light on an angle causing a cone shaped shadow hiding the "other planets/electrons". Also as we all know all means of life needs wheather it be light, water, oxygen, other living things... Thinking of electrons what if they needed light to be at its full ability to function so the the cell was able to split? if so what if it needed that light for a certain amount of time to make it visble to the eye weather it beeing seen through a microscope looking at a cell or lookin thruogh a telescope looking out at space. With all this my question is are us humans on a cell called the solar system?

P.S. If you could kindly help me clear my mind it would be greatly appreatiated.

Thank you

sincerly yours,

David Brideau

Quick answer: no.
Longer answer: Resemblances between cells and solar systems are superficial parallels at best. Suppose we say that the sun is like a nucleus. However, a nucleus has no appreciable gravity effect on the rest of the cell. No cell components rotate regularly about the nucleus. The nucleus is not (IIRC) the energy source of the cell (mitochondria?). Conversely, the sun does not possess any sort of genetic code equivalent needed to make another solar system. As for the electron thing... it sounds like you're rambling in hopes that obscurity will be mistaken for originality. — Lomn | Talk / RfC 01:44:41, 2005-09-04 (UTC)
Longer answer:
Hi there! You have asked a very interesting question. At best I will only be able to address partially the ideas you have presented here. Thanks for sharing them!
Remember that in science, (and for that matter in many many other things) many concepts are often presented in terms of particular models. These models are at best, an approximation to the "real world". They are designed to highlight specific properties or aspects of something, while hiding others. This can be a very good thing; should all the detail of something be presented, it might be too overwhelming for one to learn it and make sense of it all at once.
Having said this, the similarities that are observed between the cell, the solar system, and the atom highly depend on the model that is used to represent these structures. One need not represent or perceive a cell as a collection of organelles for example. You could, say, treat the cell as just a blob which exchanges energy with its environment in a particular fashion. In this particular model of the cell, its structural constituents are completely absent!
Admittingly, sometimes I do think of a cell and an atom as a "mini universe" with all its complex interactions and all, but it's also necessary and important to recognize that there are limitations to how far one can generalize these similarities. Yes, it is good to recognize the interdisciplinarity and crossover of some concepts, but sometimes they should not be interpreted too literally.
What I would highly encourage you to do, is to use college as an opportunity to secure yourself with the fundamentals. Learn the concepts thoroughly. Understand them well. Acquire a solid, reliable methodology to reason with new concepts so that you can learn and grow intellectually as you pursue your education and schooling. Learn to question effectively. Learn to think abstractly. Learn to problem solve.
Of course, take all this with a big gigantic caveat in mind: these are of course, my opinions :-)
On more technical matters:
There are a number of concepts here that are being intermingled together, some are philosophical and some scientific. What exactly is an "observation"? What about transfer of energy? Where does energy for cell division come from? Try to separate the concepts into more distinct portions, so you can answer them individually. Once you have this going, I'm sure your thoughts will become clearer. It might also help you find what you would be most intersted in as you progress further along your studies. All the best! --HappyCamper 02:22, 4 September 2005 (UTC)[reply]
David is not the first person to notice a parallel. I remember a Tharg's Future Shocks story in 2000 AD (comic), which were like Roald Dahl's Tales of the Unexpected in that they always ended on a twist. One story involved a manned space mission to explore the outer edges of the universe and detailed their journey. The final frames of the story showed the craft reaching the edge of a tiny blob (our universe) which was under the scrutiny of an unimaginably big alien who had been viewing their activities through a microscope. --bodnotbod 04:28, 15 September 2005 (UTC)[reply]
I liked the Future Shock where this elite military force were defending a sacred egg against hordes of snake-like aliens. As they fought and died the snakes overwhelmed them and got to the sacred egg. The scene then switched to a couple lying in bed, post-act, and then the empty packet for "Nosprog," a form of nanotech contraception. Although... that's kinda off topic. --Sum0 18:41, 15 September 2005 (UTC)[reply]

Furry coating on peaches[edit]

Does the furry coating on peaches serve any purpose for the fruit? --HappyCamper 01:14, 4 September 2005 (UTC)[reply]

My father always used to say that "It's only the hairs on a goosegog that stop it from being a grape." I don't suppose that helps. Bovlb 06:03:18, 2005-09-04 (UTC)
Likewise it's only the hairs on a peach that stops it being a nectarine. Theresa Knott (a tenth stroke) 17:37, 4 September 2005 (UTC)[reply]
  • I guess those hairs help keeping out unwanted insects and diseases like the skin does in humans. - Mgm|(talk) 08:38, September 4, 2005 (UTC)
  • The peach is actually a mammal. --81.154.236.221 16:59, 4 September 2005 (UTC)[reply]

.

I don't know at all, but I like the insect idea. The hairs make it more difficult for an insect to land on the fruit (into which it might lay its eggs, for example). Physchim62 03:04, 5 September 2005 (UTC)[reply]
I always thought that a peach was just a male nectarine that hadn't shaved. Okay, I'm kind of lying. (Though peaches and nectarines are actually the same species: a recessive gene mutation in nectarines causes the loss of the fuzz.) One of the big benefits of the peach fuzz is disease resistance—particularly in humid climates where mould can be very aggressive, the fuzz keeps the mould spores from settling on the fruit's skin. TenOfAllTrades(talk) 03:29, 5 September 2005 (UTC)[reply]
Really? - I had always thought that peaches and nectarines were different species of fruit - they've always tasted significantly differently to me. Score +1 for the reference desk! :-) Okay, so it seems that the fuzz is there to keep off bugs and to keep of spores, basically, sort of like a protection mechanism for the fruit. Score +2 for the reference desk! --HappyCamper 03:52, 5 September 2005 (UTC)[reply]
Being a bit of a philosopher I can't help but notice that you don't ask what the purpose is but if there is one. Well, evolutionarily speaking there would have to be one if the feature has been around for a long time, because else it would have disappeared over time (but then there is that irritating thing called an appendix...). But a peach is cultivated, so it might be that it was some aberration that didn't turn out to serve any 'natural' purpose, but was perpetuated because humans liked the touch (or whatever). Then that would be the purpose :) . DirkvdM 07:53, 14 September 2005 (UTC)[reply]
Sorry for the late edit, but that's not quite true. The natural process of evolution will only eliminate a feature if that feature has a reproductive disadvantage. If the fuzz was neither beneficial nor detrimental to the peach, it could easily stick around forever, since fuzzy peaches wouldn't be any more likely to fail in reproducing than unfuzzy ones. The appendix is a good example. Although it does kill people by getting infected, it doesn't kill everyone who has one. And it seems a random 'No appendix' mutation has yet to appear in the population. ...and if I go on any more I'll be solidly off-topic.:) HopperUK 13:31, 30 September 2005 (UTC)[reply]

Dipoles you put in your microwave[edit]

Water is a dipole molecule you will often put in your microwave, and, through dielectric heating, cooks your food. What are some other common dipoles you put in your microwave (and get significantly hotter)? --81.154.236.221 17:03, 4 September 2005 (UTC)[reply]

I'm not really sure if there are other common things that you would put in a microwave...I guess the next thing would be household cleaner which would have ammonia in it - but it sounds sort of absurd! The only time I've ever heated a solution of ammonia was when I was working with making some really nice transition metal compounds. However, that's a different story, and it was done over a hotplate in a fume hood. --HappyCamper 22:42, 4 September 2005 (UTC)[reply]

None, although it depends what you mean by "significantly". Physchim62 22:56, 4 September 2005 (UTC)[reply]

See the excellent microwave oven article, which says:
Water, fat, and sugar molecules in the food absorb energy from the microwave beam in a process called dielectric heating. ... Microwave heating is most efficient on liquid water, and much less so on fats, sugars, and frozen water. Microwave heating is sometimes incorrectly explained as resonance of water molecules, but this occurs only at much higher frequencies, in the tens of gigahertz.
It also contains the wonderful but useless fact that the microwave oven was invented by radar enegineer Percy Spencer who noticed that a chocolate bar in his pocket melted when he stood near an operating magnetron ! Gandalf61 09:20, September 5, 2005 (UTC)

Two Different Networks[edit]

Hey, everyone,

So, once again, abusing this source of Internet knowledge in lieu of a technical support service - I have a desktop and a notebook that I'd like to network together via a crossover cable (modified Ethernet cable - everyone know what I'm talking about?). The trick is, my desktop has an Ethernet cable plugged into it most times, which connects it to the Internet via my college network. In order to connect to the college network, there are all sorts of configurations I have to place on the connection. So many, in fact, that if I switch the Ethernet cable for the croosover cable to my laptop, I have to reconfigure the connection back to its original settings, which takes forever.

Here's my question: is it possible for me to set up a network between my laptop and my desktop, which ONLY comes into play when I connect them via the crossover cable? And is there any easy way to go back to the settings the desktop needs to connect to the college network? Both computers, by the way, use Windows XP.

Well, I hope at least some of the above is understandable. If any clarification is required, all you have to do is ask.

Thanks. --Brasswatchman 00:55, September 5, 2005 (UTC)

The best way IMHO would be to add another NIC to your desktop. This way not only you do not have to change the settings, but you would be able to have both connected at the same time. Another way would be to use Windows' support for multiple hardware profiles (IIRC — it's been a while since I last administrated a Windows box). --cesarb 01:04, 5 September 2005 (UTC)[reply]
If an additional NIC is not an option (lack of physical space in the desktop), you might be able to use a router to create an internal network between your desktop and laptop, whilst giving both of them full access to the outside network. If you put your desktop as a DMZ host (effectively what it is now), you could keep the laptop totally hidden from everyone else (they wouldn't be able to connect to it), and should pretty much be able to keep your current network settings on the desktop.
However, do note that this would likely require you to use a different network class than what the rest of the college network uses. For instance, if they're using a Class A (10.x.y.z), you'd have to reconfigure your machines to use a different class (such as Class C, 192.168.x.y).
If you're not concerned about other people potentially gaining access to your laptop, you could just get a hub and use that - it should more or less be plug-and-play if you go by that option. --Pidgeot (t) (c) (e) 01:18, 5 September 2005 (UTC)[reply]
I'll try figuring out the hardware profile thing first, since that doesn't seem to require money. Thanks, everyone. --Brasswatchman 21:06, September 5, 2005 (UTC)
I'm not sure exactly how to use hardware profiles, but I do know where to find the settings - Right click My Computer, Properties, Hardware, and then Hardware Profiles :-) splintax (talk) 14:30, 12 September 2005 (UTC)[reply]

Distinguishing colors[edit]

Consider the following table: ᓛᖁ♀ 01:27, 5 September 2005 (UTC)[reply]

  1. Why do the colors yellow, cyan, and magenta appear very distinct from the immediately adjacent colors?
  2. Why do red, green, and blue, in comparison, appear to cover three times as much area?
  3. Why do the boundaries between these colors appear diagonal (/ \/ \/ \) rather than vertical (| || || |)?
  4. For people with color blindness, where do the boundaries between regions of similar color lie, and what are their shapes?
As to #1 and #2, I would think it has something to do with the fact that red, blue and green are used at full strength for making yellow, cyan and magenta.
The point at which these secondary colors are created are where two colors are at equal strength - however, as one of those two colors slowly fades away, the other one becomes more dominant. --Pidgeot (t) (c) (e) 01:40, 5 September 2005 (UTC)[reply]
Our perception of color is incredibly dependent on neighboring colors. This could account for some of 1-3. — Laura Scudder | Talk 02:10, 5 September 2005 (UTC)[reply]
As for #3, I would say that it is due to the brain using the information from rod cells instead of cone cells when distinguishing grayscale. Personally, only the yellow seems to stand out from the adjacent colours, although this is certainly subjective. There are many diffent forms of colour blindness, not that I am aware of having any of them... Physchim62 02:20, 5 September 2005 (UTC)[reply]
Interesting. Oughtn't the rod cells respond more or less the same to colors of the same intensity, producing a uniform gradient from top to bottom, though? ᓛᖁ♀ 02:31, 5 September 2005 (UTC)[reply]
Oh, just in case there are creeping qualia involved, here's roughly what I see, in greyscale. ᓛᖁ♀ 02:27, 5 September 2005 (UTC)[reply]
The answer to that is a big NO,! see Image:Cone-response.png. On the other hand, each of our brains "corrects" for the response of our retinal cells, and there is no way of being sure of how another person "sees" a colour except by social norms. This can be a big problem in translation between languages, for example. My hypothesis, and I admit that it is only a hypothesis, is that the visual cortex downgrades the information received by the cone cells when there is little distinction between colours, simply because that is what it is most used to doing. In effect, humans are not evolved (or intelligently designed) to look at computer monitors! Physchim62 02:57, 5 September 2005 (UTC)[reply]
On a personal level, once again, the joins between colours do not seem diagonal to me. --bodnotbod 04:36, 15 September 2005 (UTC)[reply]

Article on collimators used in X-ray[edit]

Hi,

I just wanted to point out that (as far as I know) the information given about collimators in X-ray is wrong. The article and the picture actually describe an X-ray grid. Best regards,

Pirko

Thank you for your suggestion. When you feel an article needs improvement, please feel free to make those changes. Wikipedia is a wiki, so anyone can edit almost any article by simply following the Edit this page link at the top. The Wikipedia community encourages you to be bold in updating pages. Don't worry too much about making honest mistakes — they're likely to be found and corrected quickly. If you're not sure how editing works, check out how to edit a page, or use the sandbox to try out your editing skills. New contributors are always welcome. You don't even need to log in (although there are many reasons why you might want to). — Laura Scudder | Talk 07:32, 5 September 2005 (UTC)[reply]

Corn/Maize in China[edit]

Does anyone know when corn became widespread in China as (human) food, or as a crop? A 2005 Hong Kong film (Seven Swords by Tsui Hark), apparently set in early-mid 17th century (the early Qing Dynasty), showed people eating corn/maize ears and I was wondering if this was historically accurate. Thanks. --Dpr 06:48, 5 September 2005 (UTC)[reply]

Maize seems to have arrived in China during the mid to late Ming Dynasty, with a definite and detailed record by 1560. [2]Pekinensis 15:33, 5 September 2005 (UTC)[reply]
Thanks, Pekinensis, I'll look at the link shortly--I'm still curious as to when it became widespread as a food, and whether it would likely have been eaten in that manner (boiled/roasted on the whole cob)--though surely that manner would be very simple/easy for people in mobile or camp environments (as in the film). --Dpr 05:37, 7 September 2005 (UTC)[reply]

Plasma Balls[edit]

Does anyone know how a plasma ball works? They are glass balls with a thin stand and a sphere of crystals inside and when you turn them on purple bolts of what looks like electricity flow from the crystals to the inside of the glassball. If you place your hands on the outside of the glass ball the purple bolts meet with your fingertips and follow them around the glass until you take your hand off. Can anyone explain?

You might want to start by reading our article on them. --Pidgeot (t) (c) (e) 11:17, 5 September 2005 (UTC)[reply]

Oh now i feel kinda silly. To be honest i don't use the encyclopedia at all, I just enjoy listening to other peoples ideas about the questions I ask and sometimes it provides some entertainment in the form of arguements between intellectuals. Thanks though i think i might actually use the link provided this time

Hey, thanks for the question and especially the link! This is something I've always been interested in. :D

Technology Engine[edit]

What is a Technology Engine?

I can't find a relevant definition of the term. Could you perhaps fill us in as to what context this term is being used in? Garrett Albright 14:33, 5 September 2005 (UTC)[reply]
Does this describe some sort of "futurist" invention? (E.g. on the order of Buckminster Fuller--not to disparage him) --Dpr 05:40, 7 September 2005 (UTC)[reply]
There is a Technology Engine web site. The site says "This web site is under construction... with the 1and1 tools it may take years to get it right."

Fossils[edit]

if you were taking a group of children to a museum to look at fossils, why would you see more marine type fossils rather then terrestrial?

Probably because critters have been living on the ocean for a lot longer than they've been on land, and there's been more, and a greater variety, of ocean-dwellers throughout history too. See Timeline of evolution. Garrett Albright 19:51, 5 September 2005 (UTC)[reply]
Also marine animals are far more likely to get made into fossils, what with all the layers of silt etc that's constantly being liad down and turning into sedimentary rocks. Theresa Knott (a tenth stroke) 22:06, 5 September 2005 (UTC)[reply]
the second answer is almost entirely the correct reason. basically, terrestrial fossils are incredibly rare. when an animal drops dead in the forest, how long does the body lie there? in a marine environment, you have two factors going for preservation: first, the body tends to drop into a different ecosystem, possibly an anoxic region where nothing lives to disturb it. Second, it is much more likely to drop into a region that is slowly depositing material. On land, this is the region fossils are formed (with exceptions, like bodies getting washed into caves). Typically, this is a valley being slowly clogged with water borne debris, and this only happens in the really long term when plate tectonics are causing a consistent subsidence that can be filled with flood debris

Cinnamon[edit]

Does cinnamon bark contain vitamin C? If it does, would any remain after harvesting and drying? ᓛᖁ♀ 19:56, 5 September 2005 (UTC)[reply]

  • Cinnamon does contain some vitamin C; not a lot. 2 teaspoons of ground cinnamon contains 1.28mg of vitamin C. Couldn't find details on how much is the bark before harvesting and drying. --jpgordon∇∆∇∆ 21:43, 5 September 2005 (UTC)[reply]
Okay, thanks! ᓛᖁ♀ 21:58, 5 September 2005 (UTC)[reply]
Sounds like you are referring to the erroneous Culpeper scurvy reference that was deleted, restored and deleted again from Cinnamon. The reference was actually to Drimys winteri or "Captain Winter's Cinnamon" which is unrelated to true cinnamon. -- WormRunner | Talk 01:07, 6 September 2005 (UTC)[reply]

DivX and XviD[edit]

Do the DivX and XviD codecs take advantage of my computer's graphics card for encoding and decoding, or is it all done by the CPU? the wub "?/!" 20:59, 5 September 2005 (UTC)[reply]

AFAIK, the encoding/decoding is all done by the CPU. However, the final colorspace conversion when decoding can be done by the graphics card. --cesarb 21:25, 5 September 2005 (UTC)[reply]

Subatomic Radiation Shield?[edit]

Hey, everyone,

I'm a science fiction writer trying to cobble together a concept. Any scientists out there, would you tell me if there is anything extraordinarily stupid about what I'm about to suggest? - Radiation is, when you get right down to it, just a group of rogue particles created or emitted by a substance undergoing decay - either accelerated, in the case of fission, or slow, as in standard radioactive decay. So since radiation is a particle, it should be technically possible to filter out radiation on an atomic level through nanotechnology. I'm thinking of what basically looks like the quantum equivalent of the Strategic Defense Initiative - something that spits out a particle to deflect or destroy the radioactive particle. Or maybe a better idea would be a quantum-sized "filter" that lets in some particles but not others; basically acting as a net that lets larger particles through but grabs the radioactive particles. --Brasswatchman 23:15, September 6, 2005 (UTC) So, where am I wrong? Thanks. --Brasswatchman 21:14, September 5, 2005 (UTC)

How are you going to destroy the particle? You could annihilate it with it's antiparticle but the energy would still be there in the form of gamma rays. You can stop gamma rays with matter. A bloody great lump of lead should do the trick - bit that's probably too crude for you.

Some sort of net? The thing about radiation is that it is ionising Maybe your net could be transparent when neutral but turns opaque when it's ionised. Theresa Knott (a tenth stroke) 22:04, 5 September 2005 (UTC)[reply]

Well: (1) not all radiation is particulate (ie, massive particles); (2) even the "biggest" (most massive) particle radiation—the heavy cosmic rays—are only atomic nuclei, whereas nanotechnology is at the scale of cells or big molecules; (3) SDI presupposed a method to detect and track ICBMs from launch, but you have nothing analogous to that at the nuclear level; (4) even if, somehow, you managed to annihilate a cosmic ray, you now have gamma radiation carrying the combined energy of the original particle and your magic bullet; (5) it is misguided to say that particle radiation is composed of "radioactive" particles, since most of it is perfectly stable atomic nuclei or free electrons; (6) "quantum-sized" is not a definable term. I hope this helps. Sharkford 22:06, 5 September 2005 (UTC)[reply]

Just remember that sometimes sci-fi is more enjoyable (even for scientists) when it leaves how things work to the imagination rather than trying to cobble together an explanation from jargon. Can you tell me how every bit of a computer works? If not, can you tell me the end result of how a computer functions? Remember that most of your characters will have the same answer to those questions about their technology as you do about ours. — Laura Scudder | Talk 22:21, 5 September 2005 (UTC)[reply]

Responce to Theresa: right. So, as Sharkford mentioned, 'not all radiation is particulate.' So that irons out the 'magic bullet' idea, more or less. Hmm. Just glancing at the articles you linked to, trying to recall what little I can recall from AP Physics - what about the photoelectric effect? Is there any way that could be tapped to reduce gamma rays, in such a way as to reduce the overall radiation? As for the response to the 'netting' idea - so ionization would be the 'switch' that tells the netting to let particles in or out? That's a great idea. Mind if I steal that? Response to Sharkford: 1) Granted. Thank you. 2) When I said "nanotechnology," I was using it as a generic term to refer to any type of organized technology that operates on a smaller scale impercievable to humans. I apologize for my inaccuracy. 3) It's science fiction. I don't necessarily have to. 4) As Theresa said. Understood. In addition to my answer to Theresa - does the gamma radiation have any particular direction or vector? 5) All right. Once again, I apologize for my inaccuracy. I was just trying to describe my point. 6) Once again, I apologize for not getting the proper term. I'll have to be more careful in the future. Response to Laura: Yes, and an excellent point. Lord only knows that people have been accepting things like "deflector shields" for years now without batting an eye. All the same, I'm curious as to what scientific theory might say on the subject, especially as it pertains to how these kind of defenses might operate or behave. But thanks in any case. :) Thanks, everyone. Appreciate it. --Brasswatchman 01:15, September 6, 2005 (UTC)
When you're dealing with x- and gamma ray photons, there are typically three processes at work to absorb them when they enter a medium. Usually one of these processes will dominate depending on the energy of the photon in question.
  1. Photoelectric effect dominates at low energies: up to around 500 keV. The photon is absorbed completely by an inner shell electron of an atom. The absorbed energy kicks the electron free, and it carries any extra energy with it as kinetic energy. The efficiency of photoelectric absorption falls off as photon energy increases.
  2. Compton scattering is important at moderate energies: from 500 keV up to ~3.5 MeV. A gamma ray photon passes near an atom and transfers some energy to an outer shell electron. This ionizes the atom (again, the electron is kicked loose) and lowers the energy of the scattered photon.
  3. Pair production takes place at very high energies, and dominates at photon energies above about 3.5 MeV. A photon with at least 1.02 MeV (the probability of pair production increases with energy) passes near an atomic nucleus and spontaneously converts into a particle-antiparticle pair (usually an electron and a positron).
If you throw energetic gamma rays at a lump of matter, you'll get all three processes happening—pair production and Compton scattering convert high energy photons to lower energy ones, and the photoelectric effect finishes off whatever is left. The efficiency of the photoelectric effect is highly dependent on the atomic number (Z) of the absorbing atom; the efficiency goes up as something like Z to the fourth power. Compton scattering goes as about Z.
For particulate radiations, you have negatively charged beta particles (high energy electrons) positively charged alpha particles and heavier atomic nuclei, and uncharged neutrons. There are various scattering and absorption products that take care of these guys. Note that the rapid deceleration of a charged particle will generate secondary gamma or x-radiation called bremsstrahlung. TenOfAllTrades(talk) 03:37, 6 September 2005 (UTC)[reply]
I must ask: is anyone aware of research into encasing radioactive atoms in buckyballs? This might be of interest as well...--HappyCamper 03:52, 6 September 2005 (UTC)[reply]
I remember some discussion of that possibility shortly after buckyballs were discovered; I'm not sure if anything has come of it. There the idea was to trap these nasty atoms inside a relatively chemically-inert cage to make it more difficult for them to escape from physical containment. A buckyball shell would have only a negligible effect on radiation emitted from inside, however. (It's sort of like putting a firefly in a glass jar; it makes the firefly easier to handle, but the jar doesn't keep any of the emitted radiation in.) TenOfAllTrades(talk) 04:15, 6 September 2005 (UTC)[reply]
So any method that someone uses to screen out gamma radiation that involves the three methods you mentioned is also going to produce this bremsstrahlung secondary radiation? Very interesting... I notice that the article mentions that the bremsstrahlung resulting from blocking beta particles may, paradoxically, be stronger in magnitude that the original radiation. What about for gamma radiation? What's the magnitude of the secondary radiation compared to the original radiation when dealing with that? Anyone know? As always, thank you very much for your time. --Brasswatchman 23:15, September 6, 2005 (UTC)
To be clear, you get secondary radiation from rapid deceleration of charged particles: alpha particles, beta particles, and heavy nuclei. Medical x-ray machines work by aiming an electron beam (beta particles are fast electrons, remember) at a metallic target; as the electrons are stopped by the target, secondary x-rays are generated. The intensity of secondary radiation depends heavily on the atomic number(s) of the nuclei in the target material—tungsten is typically used because it has a high atomic number (74) and because it tolerates high temperatures.
So the type of shielding used depends on the type of radiation you anticipate. Shielding against x-/gamma rays is accomplished using dense, high atomic number materials like lead. Shielding against alpha and beta particles is best done using low atomic number materials—those have poor penetrating power on their own, so you want to reduce the generation of secondary radiation as much as possible. (Phosphorus-32–a beta emitter–is regularly used in biological laboratories. Typically sheets of plexiglass are used for shielding; these contain carbon, oxygen, and hydrogen. A sample of P-32 wrapped in lead foil is actually more harmful to humans than the same sample sitting exposed on the benchtop.)
If you're expecting a mix of different types of radiation, then you have to employ a defense-in-depth strategy of layered materials—or just use a thick layer of lead or concrete. Or, if you're Larry Niven, just use a layer of Ringworld floor material. TenOfAllTrades(talk) 14:41, 7 September 2005 (UTC)[reply]
It's predicted that Bose-Einstein condensates can be contained inside carbon nanotubes. Would this be able to stop all types of radiation, if the condensate could be kept cold enough? [3] ᓛᖁ♀ 15:09, 7 September 2005 (UTC)[reply]
That's an interesting way of going about it. I think. Sort of like shielding yourself with a black hole, without all the messy gravity. --Brasswatchman 19:57, September 7, 2005 (UTC)
In such an arrangement, the nanotubes ought to be kept positively charged, in order to absorb beta particles and prevent alpha particles from sticking, no? Perhaps some sort of nanotube heat pump could be worked in to allow condensate to boil away energy in a controlled manner. ᓛᖁ♀ 20:30, 7 September 2005 (UTC)[reply]
That's a neat idea; I was thinking that something really dense like neutronium would do it, too—but it's a bit difficult to carry around. :D TenOfAllTrades(talk) 20:52, 7 September 2005 (UTC)[reply]
Since it looks like this thread has just about bottomed out, just wanted to thank everyone for your help. It's been a fascinating discussion. Thanks again. --Brasswatchman 18:49, September 8, 2005 (UTC)
Let me add a few ideas before it's over: Some form of programmable matter might possibly be "tuned" to absorb certain radiation (as long as you don't mind stepping on Wil McCarthy's toes); There should be a whole periodic table of elements with nuclei that contain different quarks (than the usual up and down quarks in normal atoms), which could have unknown radiation absorption characteristics; and there may be all sorts of exotic matter with unknown properties. Lots of room around the edges for SF, still. -- DrBob 19:01, 8 September 2005 (UTC)[reply]
There's a neat idea. Thanks. --Brasswatchman 17:39, September 12, 2005 (UTC)

In Vivo 47th chromosome trials/studies worldwide[edit]

Please direct me to the persons/companies currently involved with this apparently promising gene vector. Viral vectors are less desirable, but am still interested, esp. for ADL/AMN.

Are you referring to this? This article describes development of an "artificial chromosome" as a vector for insertion of new genes. It names a couple of people and 2 companies involved in the research. alteripse 02:49, 6 September 2005 (UTC)[reply]

rounding to one decimal place[edit]

whats the one decimal place of 11.35? and what the one decimal place of 11.45? 218.111.213.149 03:33, 6 September 2005 (UTC)[reply]

  • Uh....11.4 and 11.5 respectively, I suppose. Is this a trick question? (I guess the answer would depend on which rounding method you use)--inks 03:39, 6 September 2005 (UTC)[reply]
  • Some rounding schemes insist that you do things differently if the digit next to the 5 is even or odd...as inks said. --HappyCamper 03:50, 6 September 2005 (UTC)[reply]
  • Yep, depends on the method. A (the most?) common method is round to even used to eliminate bias from always rounding 5's up. Using round to even the answers would both be 11.4. - Taxman Talk 15:51, September 6, 2005 (UTC)
  • Related question: 11.33444444445. Rounding to 2 d.p.; would it be 11.33 or 11.34? =Nichalp «Talk»= 07:54, September 6, 2005 (UTC)
    • 11.33. Everything after the next digit is of no importance in the rounding schemes I've encountered up to now. If you want it rounded to x decimal places, you look at the (x+1)th and then round up or down. ???????? ?–? 08:54, 6 September 2005 (UTC)[reply]

Chemical equation for combustion of coal[edit]

Hi, Can someone please tell me the full chemical reaction for the burning of coal? And petrol if possible? Thanks! --Fir0002 09:46, September 6, 2005 (UTC)

Coal is mostly carbon, so the basic reaction is C + O2 –> CO2. But incomplete combustion, and impurities in the coal (tar, sulfur, etc, etc), lead to the occurrence of thousands of other reactions, which is where all the smoke and stink come from. Petrol is mostly octane (C8H18), so the basic reaction is 2 C8H18 + 25 O2 –> 16 CO2 + 18 H2O (I think that adds up!). Again, there's various minor components which produce the smells. Also the odd nitrogen molecule in the air gets caught up in the heat of the moment and burnt to nitrogen dioxide (more smells and pollution) - MPF 11:46, 6 September 2005 (UTC)[reply]
I think it is incorrect to say petrol (gasoline) is mostly octane. From that article it states that "The bulk of a typical gasoline consists of hydrocarbons with between 5 and 12 carbon atoms per molecule." and that octane rating is the term for the combustion properties of gasoline, but it doesn't necessarily mean there is any octane at all in the fuel, because other compounds can have a higher octane rating than [iso]octane. I don't know what the most common distribution of compounds in gasoline is though, and I also failed to find a more detailed distribution for diesel than is in the article. - Taxman Talk 18:38, September 6, 2005 (UTC)
Some of the stink of burning coal (and diesel fuel, too) comes from the combustion of sulfur compounds to produce various sulfur oxides (mostly sulfur dioxide: SO2). Sulfur dioxide, in addition to its unpleasant odour, is also a key contributor to acid rain—sulfur dioxide combined with atmospheric water vapour (H2O) forms sulfurous acid: H2SO3. Modern coal plants are equipped with scrubbers which absorb SO2 by reaction with limestone (calcium carbonate: CaCO3). Many jurisdictions limit the amount of sulfur permitted in diesel fuel (in most of the United States and Canada, I think the current limit is 500 ppm, and there is a push to reduce that to 50 ppm in some areas.)
Actually SO2 reacts with water to form sulphuric acid, H2SO4 which fails as acid rain. Rmhermen 15:26, September 8, 2005 (UTC)
Thanks everyone for your help! --Fir0002 12:15, September 7, 2005 (UTC)

Actually, I think SO2 forms sulfurous acid H2SO3 which is acid rain, but not as stable as H2SO4

Ozone depletion because of cooling of stratosphere[edit]

Why would ozone levels be depleted if the stratosphere is cooled? --Fir0002 11:04, September 6, 2005 (UTC)

The two are not related - ozone depletion is due to catalytic decomposition of the ozone by various human-released pollutants such as CFCs. - MPF 11:53, 6 September 2005 (UTC)[reply]

Because when the stratosphere cools, clouds of ice crystals can form. The surface of these crystal acts as a catalyst for some of the reactions which lead to destruction of stratospheric ozone. This is why the depletion in the ozone layer is greatest at the poles and during winter. Physchim62 21:42, 6 September 2005 (UTC)[reply]

THanks for your answers. I asked because it said here that it enhanced the ozone depletion but not why. --Fir0002 11:59, September 7, 2005 (UTC)

The night sky, as seen from Mars[edit]

Hi, I'm a long-time answerer, first-time question-asker. Poking around on the JPL Nasa site led me to wonder why I'd never seen any pictures taken of Mars' night sky as seen from the ground. What with the Viking landers/MERs and various other landers, you'd think someone would have taken a photo at night. Do these pictures exist, or is there some technical reason why it wouldn't work?

Or is it all just a fake on a soundstage? :P --Sum0 11:51, 6 September 2005 (UTC)[reply]

Judging by the Mars sunset pic Image:MarsSunset.jpg being rather under-exposed, I'd say the camera on board is not sensitive enough to get a decent night photo. Or, perhaps more to the point, they've better things to do with the limited power resources available - a night photo might not teach us a lot. - MPF 11:59, 6 September 2005 (UTC)[reply]
There are night photographs, but they're not much use; you can't see much, really. The two rovers currently landed have produced a few, out of interest as much as anything else. this page has a selection from one night session, including a starscape in Orion, a very faint image of Earth, and - of all things! - a streak which is probably the Viking 2 orbiter.
More recent analysis suggested it was a meteor, part of a Martian meteor shower. They didn't know for sure because it was a long exposure, so they didn't know if was a slow-moving orbiting satellite like Viking 2 or a fast-moving meteor. -- Curps 22:25, 14 September 2005 (UTC)[reply]
In addition, Mars landers are solar-powered, severely limiting the power budget for doing things at night, and they like to keep the instruments from doing anything much during the cold, cold nights. So that features in. Shimgray 13:13, 6 September 2005 (UTC)[reply]
Thanks for the replies, I thought it might be something like that. Incidentally, that picture of Earth is rather awe-inspiring when you consider it. --Sum0 11:24, 7 September 2005 (UTC)[reply]
Mars Global Surveyor has also imaged Earth, though this time from Mars orbit - it's even possible to discern continents! [4] Shimgray 11:40, 7 September 2005 (UTC)[reply]
See Astronomy on Mars. There's a photo of a meteor linked from there, and pictures of the Earth and Moon from Mars. Also Transit of Phobos from Mars and Transit of Deimos from Mars. The Spirit and Opportunity rover cameras aren't really designed for taking pictures of the night sky, too low-res. The Earth and Moon picture was taken by Mars Global Surveyor. Also this photo of Phobos and Deimos zipping through the Martian night sky: [5]. The latter photo is part of a gallery of other photos, see [6]. -- Curps 22:15, 14 September 2005 (UTC)[reply]

Does anyone know what plant this is?[edit]

File:UnknownPlantCG1.jpg
An unidentified plant. Maybe milkweed.
File:UnknownPlantCG2.jpg
An unidentified plant. Maybe milkweed.

I was at my local zoo with my cousin and her daughters, when we came across this plant. She suspected it might be milkweed, and her comment left me very curious. I snapped this picture for reference, then cropped it a bit. Can anyone either confirm or give a different id on this plant? CanadaGirl 13:58, 6 September 2005 (UTC)[reply]

Not knowing what local is for you, and not being a botanist, I can't be sure, but I'm pretty confident you are right. The pod in the right side of the middle of the photo seems convincing to me. A shot more of the top of the plant would be easier to tell for me. Were there Monarchs nearby? - Taxman Talk 15:42, September 6, 2005 (UTC)
Local for me is southern Manitoba, Canada. I have added a wider shot of the same plant. I didn't notice if there were Monarchs. Thanks for your help. CanadaGirl 15:52, 6 September 2005 (UTC)[reply]
Looks like Buttonbush Cephalanthus occidentalis (Rubiaceae) to me. We don't have a page on that yet (not even on the genus Cephalanthus) - I'll write it if I may use the pic - MPF 18:15, 6 September 2005 (UTC) - Second thoughts - not convinced on Buttonbush any more - it doesn't quite match - MPF 18:30, 6 September 2005 (UTC)[reply]
Well that image doesn't seem to help any more, but milkweed still seems very plausible to me. Compare the pictures of the pods on the image in our article to those on yours. Based on those, I'd have a hard time believing it is anything different, but like I said, I'm no botanist. But if it is important, you could always take it to your local university botany/biology department and ask them. The people near that zoo exhibit should know too. I was thinking if it happened to be a monarch exhibit, milkweed would be a shoe in. Google searching for milkweed also showed that there is a lot more variation among the milkweed species than I was aware of. - Taxman Talk 18:24, September 6, 2005 (UTC)

Crab[edit]

Mystery crab

Moved from Wikipedia talk:WikiProject Tree of Life

It's a long shot but can anyone identify this crab, spotted on the south coast of New South Wales, Australia?--nixie 11:42, 2 September 2005 (UTC)[reply]

<insert your own joke about crabs down under, I'm tired> - Taxman Talk 03:42, September 7, 2005 (UTC)

Heat damage to bottled water[edit]

If I carry a bottle of water, that is still sealed, from the store, never been opened, on my bike, in Arizona, where summer temperatures can stay above 100 degrees for days, how long is that bottle of water still consumable?

Jrb

At above 100°C, the water will boil, so the bottle may explode when sufficient pressure has built up. It will also be lethal for the cyclist. - MPF 18:37, 6 September 2005 (UTC)[reply]
Arizona doesn't reach 100°C. Jrb is talking °F Theresa Knott (a tenth stroke) 20:19, 6 September 2005 (UTC)[reply]
So what does that archaic F cr@p translate to in real money, then?? MPF
The Fahrenheit article gives the formula, but it's about 38°C - Taxman Talk 22:36, September 6, 2005 (UTC)
I would think an unopened bottle of water from a store would remain potable pretty much indefinitely. Tap water stored in a water bottle, however, would remain potable for much less time, unless treated in some way. ᓛᖁ♀ 15:35, 6 September 2005 (UTC)[reply]
Depends on your definition of consumable. At 100 degrees, the water certainly wouldn't be very refreshing. Depending on how you store the water bottle, solar heating could take it well above that. I'm assuming you are more worried about chemicals leaching from the plastic? I think all plastics used in water bottles are relatively benign, but if you look on the bottle for the specific type, and look for the heat decomposition of that type, you may find more useful information. Was there something other than that you were worried about? You may also want to look into something akin to a desert water bag [7] that uses very effective evaporative cooling. - Taxman Talk 15:36, September 6, 2005 (UTC)
Also effective may be freezing the water before you need it, depending on how long you'll be outside. The water will melt in the heat but remain cool while there is ice left. Note that the bottles must be emptied by at least a few inches before freezing, to prevent them from bursting. ᓛᖁ♀ 17:14, 6 September 2005 (UTC)[reply]
In terms of degredation of the plastic, the two water bottles on my desk are both made of Polyethylene terephthalate (PET / PETE). The wikipedia article on PETE does mention that the plastic degrades with high temperatures, leading to the production of acetaldehyde - which is naturally occuring and, according to our article, harmless other than the possibility of hangover-like symptoms. Thryduulf 15:51, 6 September 2005 (UTC)[reply]
Additionally, "high temperatures" in this case are well over 100°F; PET decomposes around 300°C (572°F). Its melting point is somewhat lower (255°C, 491°F) — and of course the bottle would explode well before reaching this point — so at least temperature should not be a problem. [8] ᓛᖁ♀ 16:16, 6 September 2005 (UTC)[reply]
If it's not decomposing, what is the cause of the plastic/chemical taste from waterbottles that have been left in the sun? - Taxman Talk 18:27, September 6, 2005 (UTC)
That's probably due to the quality of the manufacturing process. Some chemical residue may end up in the plastic and leach out over time; PET is fairly permeable (enough so that various pollutants could get into the water as well, if the bottle is not stored in a clean place). Some companies make water bottles designed to prevent this sort of thing, such as the Platypus brand (lined with polyethylene). [9] ᓛᖁ♀ 18:40, 6 September 2005 (UTC)[reply]

inertial guidance systems[edit]

I would like to know something about the usage of inertial guidance system.Can this system be used in a car for determining its position and speed?And what will be cost of using such an instrument in a car? And if it is not feasible for economical or technical reasons,is it possible to measure the position and direction of a car over a distance of 15-30kms fairly accurately so that it can be used to know the location of a car at any moment of time in a journey.Can we use speedometer or distance meter in a car to know the distance moved by the car and can we use steering wheel movements to determine the direction of car for this purpose or some kind of a sensor which can do this thing economically? --203.197.74.148 15:34, 6 September 2005 (UTC) Sumit Malhotra s_malhotra@iitb.ac.in[reply]

Inertial systems can do the job (they'd be more accurate than the speedo/steering method); however, GPS / DGPS is a better solution, in my opinion. — Lomn | Talk / RfC 22:09, 6 September 2005 (UTC)[reply]
Yeah, unless there was some compelling reason *not* to use satellite navigation it makes little sense. A high-quality electronic compass sensor would probably cost more than a GPS, given that you can get a GPS unit with a USB interface for well under 100 USD. --Robert Merkel 23:22, 6 September 2005 (UTC)[reply]
I believe before GPS's Selective Availability was turned off (which made street-precise GPS available to the public), there were a few hybrid GPS/inertial nav. systems around. I remember hearing of a trial of one in Japan in the late 80s/early 90s. -- DrBob 23:27, 6 September 2005 (UTC)[reply]
Yeah, I remember them (and the joys of the US government playing silly buggers with GPS) too. If you're still interested in a hybrid system, marine autopilots are fitted with fluxgate compasses, and have been since before GPS existed. Some even have gyrocompasses. Marine fluxgates are not cheap (hundreds of USD); gyrocompasses are even more expensive. --Robert Merkel 08:23, 7 September 2005 (UTC)[reply]


medical doctor[edit]

Hi! Can you help me find contact information for this very famous fertility doctor: Dr. Severino Antinori, in Rome, Italy? He appears in several of your articles, but there is no contact information for him--e.g., address, email, phone. Thanks.

S. Ashe sashe@cerritos.edu 562.621-1874

  • I'm sorry. Wikipedia doesn't include contact details for people. But if you know where he works, I would imagine his hospital/clinic can help you contact him. Such organizations usually have websites you can visit. - Mgm|(talk) 10:41, September 7, 2005 (UTC)

A google search with Severino Antinori fertility yields dozens of stories about him. He has a fertility clinic (International Associated Research Centre for Human Reproduction, Italy) in Rome described as being "500 yards from the Vatican" (about 0.5 km). I found 2 links to his clinic's web page but neither seem to be currently valid. [10] and [www.raprui.it/home_i.html]. A report of his work is PMID 7782415. alteripse 10:55, 7 September 2005 (UTC)[reply]

The effect of human Bioenergy on the propagation of elastic waves[edit]

Performing research on the human body and its radiation of energy fields. Frequently called the human Aura that is linked with the Chakra System. Elastic waves define a spectrum between ultraviolet and light.

Any addition information in your encyclopedia on this subject?

Thank you,

Prof. J

You can try aura (paranormal) - and note the references at the bottom of the article; these may be useful. You might also like to try our articles on Chakra, Kirlian photography. All these articles have many useful links to other associated topics. Proto t c 10:10, 7 September 2005 (UTC)[reply]

And don't forget to submit your positive results for the James Randi million dollar prize! alteripse 11:02, 7 September 2005 (UTC)[reply]

Or grow up and visit planet earth sometime, but until then do as alteripse says

Banana Peel?[edit]

Is Banana peel actually especially slippery?


Yes. As the outside of a banana peel (the more yellow side) is actually quite sticky (try rubbing your hand across it - there's a lot of friction there), and the whiter inside has (when still moist) a very low coefficient of friction, when you put your foot on the peel, the friction of the yellow, outer side ensures your foot will stay on the peel, while the slippery inside will ensure you slide across the floor, and usually fall over.

However, for this to happen, three things need to occur: 1) The peel must have the slippery (inside) side on the floor, and the yellow outer peel facing upwards. 2) The floor the peel is on must be hard and relatively smooth (ie, a polished or tiled floor). 3) The peel must still be moist (ie, not been there for a while.

So yes. A banana peel is especially slippery. Proto t c 13:08, 7 September 2005 (UTC)[reply]

Best Reference Desk question and answer ever. Garrett Albright 14:32, 7 September 2005 (UTC)[reply]

Katrina Effect[edit]

What is the Katrina Effect? The name applied to instances where political, military, and other officials respond very poorly and very late to a major disaster.

Did you answer your own question? Anyway, that's a neologism if I've ever heard one.
Try asking this person for more details. MPF

CHEMISTRY[edit]

how heat can turns solid matter into liquid matter in molecules reaction?

HOMEWORK QUESTION. Heat is energy. Molecules in a liquid are moving around more (have more energy) than molecules in a solid. Start with that. Garrett Albright 14:32, 7 September 2005 (UTC)[reply]

Earth air travel[edit]

Suppose two points on earth equator surface which is 180° separated. And two planes is going at the same time from A to B with same speed, one move west and one move east. Are both planes going to be arrived at the same time or not? roscoe_x 16:19, 7 September 2005 (UTC)[reply]

In theory, yes, the same time. In practice, the plane which goes over the western hemisphere may have to climb a little higher to avoid Volcán Cayambe (4600m altitude on the equator), so might arrive slightly later. A more significant practical problem might be the weather, which could make it very difficult for the two planes to maintain identical ground speeds. - MPF 17:01, 7 September 2005 (UTC)[reply]
It depends on your assumptions. If the planes are travelling at the same speed relative to the ground, they will arrive at the same time. If they're moving at the same speed relative to the air, then it depends on trade winds and so on. If they're moving at the same speed relative to a co-moving Earth-centered inertial frame, then the westward moving plane will get their first, since the Earth's rotation is moving point B towards the westward plane and away from the eastward one. -- DrBob 17:26, 7 September 2005 (UTC)[reply]
To extend, in the third case, the eastward plane will "arrive" by way of the Earth catching up to the plane, assuming the plane is your standard sub-mach airliner or the like. — Lomn | Talk / RfC 20:52, 7 September 2005 (UTC)[reply]
In the third case, could I suppose, if the plane has a speed of 800 km/h relative to co-moving earth-centered inertial frame. w/o taking consideration of the wind blow and earth surface height, Could you count the time of arrival between these two airplanes?
or
In real flight, do we really have the third case? roscoe_x 16:53, 8 September 2005 (UTC)[reply]
Lorm makes a point I hadn't realized properly until I calculated it: In an ECI frame, any point on the Earth's equator is moving eastward at about 1674 km/h. So an eastward plane going 800 km/h (relative to an ECI frame) is actually moving backwards (relative to the Earth) at 874 km/h. So point B catches up to the plane from behind.
The westward plane is going 1674 + 800 = 2474 km/h relative to the Earth. So to make the 20037 km trip (half the Earth's equatorial circumference), it takes about 8.1 hours.
The "eastward" plane is actually going westwards at 874 km/h, so travels 20037 km in about 22.9 hours, for a difference of 14.8 hours.
No, this case is nothing like a real flight. A real plane's speed is relative to the air, which is already spinning at (roughly) the same speed as the Earth. So wind is the important factor. I believe the trade winds blow from the east (east-to-west), so the eastward plane will generally be slower than the westward one. -- DrBob 17:50, 8 September 2005 (UTC)[reply]

Radioactive decay in molecules[edit]

When an atom in a molecule undergoes radioactive decay, what happens to the molecule? ᓛᖁ♀ 16:30, 7 September 2005 (UTC)[reply]

Well I presume the other atoms in the molecule just float off and probably bond with other atoms/molecules is they are not stable. --ßjweþþ (talk) 17:51, 7 September 2005 (UTC)[reply]
The charge on the atom would almost instantly change, but the exact effects would depend on the situation. In a metal, this would probably be no big deal as the extra electrons/holes would just flow freely around. In a crystal, you'd probably get something like this: Na22+Cl- (sodium chloride) beta decays to Ne22Cl-; the neon, being neutral, would float off freely, leaving a free chloride ion. If the original material was in solution, this would probably alter the pH. (Note: please feel free to fix my formatting, as I don't yet know how to use the <math> tags. The numbers in parentheses are isotopes, and the +/- show the charge). --David Wahler (talk) 20:12, 7 September 2005 (UTC)[reply]
The atom in question will usually escape from whatever molecule it was attached to. When the radioactive decay occurs momentum is conserved—the atom gets an equal push in the opposite direction when radiation is emitted. The amount of kinetic energy it gets is usually quite large compared to the binding energy that held it in the molecule. What gets left behind is usually going to be an ion or a free radical of some sort.
In nonconducting solid materials, this can result in charge separations: small regions of positive and negative charge that have no easy way of getting back together again. (These little charged regions can behave very differently from their surroundings. Sodium chloride turns pale orange when exposed to ionizing radiation, and fades to gray as the positive and negative charges gradually reunite. Different types of glass can turn brown, green, or violet. Potassium bromide turns a lovely blue colour.)
As noted above, electrons in metals slosh around fairly freely, so these charge separations disappear rapidly. TenOfAllTrades(talk) 12:33, 8 September 2005 (UTC)[reply]

differences and similarities[edit]

hello!

I am a Year 9 student at Tonbridge grammar school for girls and i am doing a project and i am in need of your help!

i am trying to find out between the differences and similarities between the following:

neon and argon neon and francium

i would be grateful if you could give me a list of the similarities or point me in the right direction of finding the answers.

thank-you charlotte

p.s i do not know where to contact you so sorry if this is the wrong email place.

Hi! If you're not sure where to look, you can start with the articles on each of those substances: Neon, Argon and Francium. You can also get to any of these articles by typing the name into the "search" box near the top-left hand corner of any Wikipedia page, and then clicking Go. --David Wahler (talk) 19:54, 7 September 2005 (UTC)[reply]
It should be noted that similarities tend to run vertically on the periodic table. Our article includes an excellent summary of element families that may point you in the right direction. — Lomn | Talk / RfC 21:00, 7 September 2005 (UTC)[reply]
Make sure to pay attention to what column the three elements are in. These columns have special names (noble gases and alkali metals) because they share a number of properties with other elements in the same column. This is historically how the organization of the periodic table was developed: by observing similarities between elements and grouping them accordingly. — Laura Scudder | Talk 21:06, 7 September 2005 (UTC)[reply]


motion sickness in games[edit]

i was playing gta sa and was attacked by motion sickness. is this normal? --Phil 1970 20:45, 7 September 2005 (UTC)[reply]

Before I continue, I'd like to point you to Wikipedia:Medical disclaimer - just in case.
I haven't played that particular game myself, but no, I don't think it is. It might just have been a freak occurence brought on by a number of outside factors, but if you're concerned or it happens again, I suggest you go see a doctor. --Pidgeot (t) (c) (e) 20:56, 7 September 2005 (UTC)[reply]


It's not too unusual. Motion sickness generally occurs because of a mismatch between what your eyes are seeing and what movements are detected by your inner ear (you can get the same effect in reverse by, say, reading a book in a moving car). First person shooters are a common culprit (for instance, bits of Half-Life 2 had to be redesigned because they were too nausea-inducing), though I could see GTA:SA giving the same effect. General advice is to play further back from the screen or monitor, so that you can see more of the surroundings (also, don't play in a dark room where all you can see is the screen). And take regular breaks, get up and walk around, etc., and don't play when tired. -- DrBob 21:03, 7 September 2005 (UTC)[reply]
I once suffered from this while playing an old 3D game (I think it was Heretic or Hexen). I think what caused this was that the 3D perspectgive was wrong. --R.Koot 00:29, 8 September 2005 (UTC)[reply]
  • A lot of games can trigger migraines for me -- and my symptoms include motion sickness-like symptoms (vertigo, wooziness, slight nausea). The first one I noticed it on was Doom; lots of games that use similar motion can do it. But some games do it with no obvious similarity. Only takes about 5 minutes of play on the worst ones, and I feel like crap for a day. --jpgordon∇∆∇∆ 00:54, 8 September 2005 (UTC)[reply]

Technetium and Promethium[edit]

Note that both of these elements have no stable isotopes even though the ones next to them do.

Also note that if the Rare Earth (Lanthanides) are extended in place of the third period of the transition elements, Promethium appears right under Technetium. Also note that both atomic numbers are prime numbers whereas Manganese is not. Some discussion of if this is just coincidence or if there is a story to tell there would be interesting to me.

I would expect it's mostly coincidence. A proper extension of the rare earth elements pushes everything further right so that Technetium and Promethium do not align vertically, if I recall correctly.
That said, it's been a long time since high school chemistry. — Lomn | Talk / RfC 21:14, 7 September 2005 (UTC)[reply]

I dont' know what you mean by 'proper' A 'proper' arrangement of the elements is just as it is now, with the Rare Earths in their own group. I am suggesting an 'improper' arrangement, but one which is not random check this out.

http://en.wikipedia.org/wiki/Periodic_table

If you put Lanthanum under Yttrium, Cerium under Zirconium, et al, Promethium is under Technetium. Coincidence? Maybe. Awfully odd that the two light (relatively) unstable elements line up like that.

LCD & Plasma[edit]

I want to know? LCD monitors are dangerous to the eyes? Robert Ackerman

They are less damaging than CRTs. But what exactly do you mean by "dangerous"? Is myopia a danger, or are you more concerned about, say, eyeball cancer?--inks 23:36, 7 September 2005 (UTC)[reply]
The following caution about liquid crystal displays came with my digi camera - if the LCD cover is broken and the liquid leaks out, and gets into your eyes, then "take the urgent action indicated": "flush the affected eye with clean water for 15 minutes, and seek medical assistance". Also if swallowed, "drink large amounts of water and induce vomiting, then seek medical assistance". Seems a rather unlikely scenario, but clearly the liquid in an LCD is toxic enough to warrant the caution and advice for medical attention - MPF 00:29, 8 September 2005 (UTC)[reply]

My first impression is that if there is even a remote chance of a problem, a company will put warnings up to mitigate any possible lawsuit.

ice cubes[edit]

If I put two clouds of steam together, I end up with one cloud of steam. If I put two cups of water together, I end up with one cup of water. If I put two ice cubes together, I still have two ice cubes. What chemical/physical proces prevents solids from 'melting' together? --R.Koot 00:48, 8 September 2005 (UTC)[reply]

Steam and liquid water are the same thing. So the question simplifies to:
  • What's the difference between the liquid and solid phases of water?
The answer is more mathematical than chemical/physical. In the (3D) interior of a ice cube the interactions between the molecules are weaker than in the (2D) surface (proof left up to the reader :-). Oh yes, Pauli speaked of the "surface hell".). So, the same forces/interactions that keep the cube solid/coese, keep one cube apart from the other.
Err, sorry by my bad English, but hope this help. User:Mdob | Talk 01:15, 8 September 2005 (UTC)[reply]
This is basically the central problem of the study of adhesives -- namely, what causes molecules to stick together strongly within objects, but not between objects at surfaces, and how can this behavior be changes? --David Wahler (talk) 01:26, 8 September 2005 (UTC)[reply]
Ice lumps can fuse together easily under pressure - see Glacier#Glacial motion for details - MPF 01:30, 8 September 2005 (UTC)[reply]
Actually, IIRC, the Pauli line was: "God abominates the surfaces" User:Mdob | Talk 23:54, 8 September 2005 (UTC)[reply]
In liquids and gases, the particles have enough freedom of movement to allow other particles to squeeze between them. In solids, the particles are nearly fixed in place and cannot move aside to admit more material. Thus, when two ice cubes are pushed together, the water molecules would have to pass through each other for the ice cubes to move any farther.
However, a water molecule is surrounded by a cloud of negatively charged electrons. While the molecules are overall neutrally charged, the electron clouds will encounter each other first, and will repel each other according to the very powerful electromagnetic force. Only an extreme amount of pressure — or the energy involved in a chemical reaction — could overcome this repulsive force.
This doesn't prevent the ice cubes from freezing together (hydrogen bonding) or from sticking together by vacuum cementation (Van der Waals force; both of these effects are also controlled by electromagnetism), but the water molecules cannot physically move through each other. ᓛᖁ♀ 01:43, 8 September 2005 (UTC)[reply]

Just to pick a nit, steam and liquid water are not the same. Steam is a gas. http://en.wikipedia.org/wiki/Steam You are probably thinking of mist, which is commonly called steam, but is not scientifically accurate.

Yeah, but I was thinking about water microdroplets (in Portuguese: (vapor/vapor d'água) wich I mistranslated to steam). User:Mdob | Talk 23:54, 8 September 2005 (UTC)[reply]
"Just to pick a nit, steam and liquid water are not the same" - actually, wrong. Steam originally (earliest references circa 1000 AD) meant clouds of condensed water droplets, i.e., what was visible above a cooking pot. It is only in relatively recent times (since the industrial revolution) that it has secondarily also come to mean gaseous water above 100° - MPF 00:10, 10 September 2005 (UTC)[reply]

Question about scripting in Qbasic?[edit]

If you had to write a command to import numerical data from a .txt document, or something else like it, to be used in a qbasic program, how would you do it?? any help would be appreciated, thank you in advance--64.12.117.12 02:10, 8 September 2005 (UTC)[reply]

You mean besides telling you to use a real language that isn't dead? :) I've long since forgotten any quickbasic I learned, but in a quick google search I found Manipulating External Files on a qbasic tutorials site. - Taxman Talk 17:20, September 8, 2005 (UTC)
You'll probably want something like this:
OPEN "file.txt" FOR INPUT AS #1
DO WHILE NOT EOF(1)
  LINE INPUT #1, s$
  i=VAL(s$)
  REM Process your entry here...
LOOP
CLOSE #1
This assumes you're going to process each number as you read it. If you want to read in everything at once, you'd have to use an array. Whether you'll need a dynamic or a static array depends on the format on the textfile.
You might be able to use INPUT #1, i instead of LINE INPUT ... VAL(s$), but it's been quite a while since I used QBasic, so that approach might only be applicable to binary numbers. --Pidgeot (t) (c) (e) 17:50, 8 September 2005 (UTC)[reply]
INPUT will input numbers or text but it uses commas a delimiters, if I recall. It's generally the best way to deal with numerical info, though, because it saves you the step of converting the text into a number (and if it fails, it will fail spectacularly, rather than failing in a subtle way). --Fastfission 21:07, 11 September 2005 (UTC)[reply]

Pauli exclusion principle[edit]

Why are helium-4 atoms bosons? ᓛᖁ♀ 02:38, 8 September 2005 (UTC)[reply]

A Helium-4 nucleus contains 2 protons and 2 neutrons, each of which is a fermion with spin of ±½. As a whole, the nucleus will have a total spin of +2, +1, 0, -1 or -2 - in each case an integer, so the nucleus is a boson. Compare a Helium-3 nucleus, which contains 3 nucleons, so the nucleus as a whole will have a total spin of 3/2, ½, -½ or -3/2 - in each case an half-integer, so the nucleus is a fermion. HTH. Now ask why the electrons don't matter :) -- ALoan (Talk) 11:37, 8 September 2005 (UTC)[reply]
Ah, okay. The electrons don't matter because they have a total spin of 0, correct? Why are the individual protons, neutrons, and electrons able to occupy the same locations in a Bose-Einstein condensate? ᓛᖁ♀ 14:12, 8 September 2005 (UTC)[reply]
Well, actually, the electrons do matter. My answer above discussed the nucleus on its own, not the atom as a whole :) The electrons are also fermions, but there are two of them in a neutral Helium atom: together, the electrons will have a total spin of +1, 0, or -1, leaving He-4 a boson, and Helium a fermion (thinking further, given that the number of electrons equals the number of protons in any neutral atom, the number of neutrons is the only important factor: even → boson; odd → fermion).
AFAIK, the atoms in a BEC don't end up at the same physical location (if nothing else, electrostatic forces ought to keep them apart): they are simply in the same quantum state.
Now we need a proper particle physicist to let us know that I have not just made all this up. I hope I haven't mixed together my quantum numbers... -- ALoan (Talk) 14:20, 9 September 2005 (UTC)[reply]
But in helium, the electrons share the 1s orbital, so they must have opposite spins. For their total spin to be -1 or +1, one would have to be excited to a higher energy level. ᓛᖁ♀ 16:38, 9 September 2005 (UTC)[reply]
Hence my comment about quantum numbers. Yes, I think you are right (I was confusing myself thinking about hydrogen, where the nucleus and the electron can have the same spin - hence the 21 cm spin-flip emission line). But the answer remains the same, doesn't it? The electrons can make a difference, but don't in this case. -- ALoan (Talk) 17:42, 9 September 2005 (UTC)[reply]

mechanism for peptide bond forming reaction?[edit]

does anyone have the exact mechanism for this reaction, other than just, the n-terminal amino takes the place of the carboxylic OH, because everyone knows you make a CO2- into a good leaving group without a LOT of enzyme assistnce, thanks in advance--64.12.116.132 02:02, 8 September 2005 (UTC)

  • It appears you have some knowledge of biochemistry, so I would recommend checking a copy of "Stryer's Biochemistry" at your nearest academic library. - Mgm|(talk) 08:30, September 8, 2005 (UTC)

Logarithm common[edit]

The value of log2 is 0.3010, that of log20 is 1.3010 and that of log0.2 should be -1.3010

Is the above statement correct? When I checked up the value of log of fractional nos. (> 0 and < 1 I get wrong answers in Scientific calculators, MS Excel formula etc.

Whys is this so?

can I get a reply to my email ID vaidyaguru@yahoo.com or vaidyaguru@gmail.com

Okay I got the answer. The value is calculated as -1 + 0.3010 = -0.6990

Thanks Wikipedia anyway.

Clearly these are logarithms in base 10. Remember that log102 = 0.30103 because 2 = 100.30103, and multiplying arguments is the same as adding the logs. Thus, moving the decimal point left or right in the argument is equivalent to adding or subtracting one from the log. As you will have found out, -1.3010 = log10 0.05. HTH. -- ALoan (Talk) 11:37, 8 September 2005 (UTC)[reply]

A Sun Line[edit]

Hello,

My name is Adam Rogers and I live in Conway, AR. On a roadtrip yesterday I noticed a horizontal black line which spanned the distance of the sun and no other parts of the sky. There were no clouds and no jet trails, even had there been they would not have blocked the sunlight in such a fashion. The occurence I observed lastted about 15 minutes and the black line followed the size of the sun from top to bottom as the line rose upwards across the sun. If anyone has seen a black horizontal line on the sun before or has any explanation as to why my eyes would have perceived such a thing I would appreciate the information. Again the line began as a small black dot on the bottom of the sun and began rising upwards across the sun growwing in length to match the exact circumference of the sun as I viewed it. The line continued to grow until it reached the 1/2 way point of the sun and then began to shrink as it began its ascent to the top of the sun.

24.144.63.26 05:01, 8 September 2005 (UTC)[reply]

  • I don't know the anser to your question but having a look at out Sun article: "Caution: Looking directly at the Sun can damage the retina and one's eyesight". Please be careful. --Commander Keane 08:49, September 8, 2005 (UTC)

Make money on rising oil prices?[edit]

If I think that oil prices will continue rising, how do I cash in on that? Would it make a difference if I have $1000 or $1000000?

Well, you could buy and hold oil directly - but that will involve delivery and storage costs which could wipe out any theoretical profit. Or you could invest in oil futures or buy a call option on oil futures, which would give you economic exposure to the oil market without delivery or storage costs. Or you could invest in companies which could benefit from rising oil prices - maybe nuclear power companies or manufacturers of solar panels, wind turbines or bicycles ?? But you might want to re-examine your assumption that oil prices will continue to rise - there is an alternative view in this article, for example. Gandalf61 11:12, September 8, 2005 (UTC)
An obvious one you've missed is buying shares in oil companies, but you're a bit late into the game on that one. Another is investing in companies that have technologies to manufacture fuel from sources other than crude. The best strategy depends on over what timescale you're predicting that oil will rise, and to what magnitude. If you were convinced the oil price was going to be $100 in 3 months, options might be your best bet (and that's what it is, essentially - a bet). If you were convinced the long term trend over the next 5 or 10 years is significantly up, I'd be looking at companies, particularly oil alternative providers. Personally, I'm skeptical that prices are going to stay this high; for instance, at the moment you can make diesel from coal (of which there is no shortage) profitably.
A caveat, though. Kevin Drum's blog on the Washington Monthly site has linked to a number of stories on Peak oil, and similar questions to yours have come up. Basically, the view of most of the posters there is that the oil futures/options game is not one that amateurs want to play. --Robert Merkel 03:10, 9 September 2005 (UTC)[reply]

Is heated olive oil bad for you?[edit]

when i was single, my parents used to tell me that heated olive oil is as bad as saturated fat. I'm not very good at finding specific information online. As a result, now i know lots about olive oil, but nothing regards what i want to know. :) If anyone can help e i'll be very thankfull. Benedeta 08:44, 8 September 2005 (UTC)Benedeta[reply]

I think you mean "can heated olive oil be bad for you", to which the answer is certainly yes. ISTR that repeated heating of the unsaturated fats in any cooking oil can cause hydrogenation which can turn them into saturated fats (or even trans fats). Now you have to ask whether saturated fats are bad for you... -- ALoan (Talk) 15:40, 8 September 2005 (UTC)[reply]

Logs from first principles[edit]

How can you calculate logarithms from first principles - for example that log 2 is 0.3010 ?

If x = log10 2 then 10x − 2 = 0 and we can solve the latter using a root-finding algorithm such as the bisection method or Newton's method. Gdr 14:33:28, 2005-09-08 (UTC)
There's a handy description here. By making sensible use of of series expansions, you can generate logs. A useful Taylor series expansion is
Hope that helps. TenOfAllTrades(talk) 14:44, 8 September 2005 (UTC)[reply]
Note that ln is the natural log, to base e (see e (mathematical constant)): ln(2) is 0.69315 not 0.30103, but changing bases is relatively easy (multiply the log in the first base by the log (in the first base) of the base you want, so ln 2 = lg 2 × ln 10). In the days before electronic calculators, human calculators spent a lot of time working out log tables, trig tables, etc for the rest of us to use.
Oh happy day - there is a Unicode symbol - ? -- ALoan (Talk) 15:29, 8 September 2005 (UTC)[reply]

And let's not forget

x

?(1/x)dx = ln(x)

1



Entropy?[edit]

Where does heat go when it loses energy to entropy? I was once told that it is converted into other forms of energy that do not interact with the universe (as far as we know). Still, where does all the energy go, since it cannot be created or destroyed? --Brasswatchman 20:15, September 8, 2005 (UTC)

Usually heat is the end result of entropy. Ordered energy like electricity might pass through a wire in an electric motor to produce mechanical energy, but some of that energy is lost to heat by interaction with the wire. A nuclear isotope ejects a subatomic particle and the particle bounces against air molecules and causes them to move more vigorously, that is heat. In the case of a hot object in a cool environment, the hot object tends to radiate heat away faster than the environment it is in. In that case the answer to your question is the heat goes to warm up the cool environment, so the heat is not lost, just spread out more evenly. So there is now a cooler object and a warmer environment, this is considered to be a less ordered situation, less order means greater entropy.

So the amount of energy in the universe remains constant - it just becomes spread out over a greater volume? --Brasswatchman 22:00, September 8, 2005 (UTC)

basically, yes. See Heat death of the universe. Thryduulf 23:42, 8 September 2005 (UTC)[reply]
I understand. Thank you. --Brasswatchman 00:39, September 9, 2005 (UTC)
  • I read The Whole Shebang recently, in which Timothy Ferris explains how all the energy in the universe might actually equal zero. For instance, if you take gravity, which is an attractive force, and place it on the negative side of the scale, then sum it up with all the remaining matter/energy in the universe, which is in contrast, repulsive... some scientists believe it may just equal zero. They suggest that the big bang basically disemboweled the energy of the universe, in which it previously existed as a unified force, and ever since, what we call entropy, is the universe trying to re-establish itself into this one singular, unified energy... whatever that may imply. - Cobra Ky (talk, contribs) 09:03, September 10, 2005 (UTC)
So gravity, in this conception, would be a kind of anti-energy? Or maybe, more accurately, contra-energy? Thanks for the thought. --Brasswatchman 17:25, September 12, 2005 (UTC)

Solving 2y = 18, divide both sides by 2 or just move the 2?[edit]

Example:

2y = 18
2y/2 = 18/2
y = 9

or

2y = 18
y = 18/2
y = 9

In all my math classes they say to divide both sides by 2 and cancel the left one out, but I always just moved the 2 to the other side and don't have to cancel anything out. Is there something different between the two ways? --pile0nadestalk | contribs 20:38, 8 September 2005 (UTC)[reply]

  • In this particular question it doesn't matter because by moving to the right you are dividing the 18 leaving 1y on the left removing the 2 gives y, 2y/2 is also y. They always say divide both sides to avoid problems when things aren't as simple as that (when fractions get involved and when dividing one time isn't enough). - Mgm|(talk) 20:56, September 8, 2005 (UTC)
  • To add to Mgm's point, it's important to remember that what you are really doing is "dividing both sides by (something)" when the (something) may possibly be zero (if it's a variable, say). Your solution then won't be valid when (something)=0, because that's an invalid operation. -- DrBob 21:16, 8 September 2005 (UTC)[reply]

Short cuts like this can cause problems when the math gets complex. Take this - I am not sophisticated enough to do exponents any other way so a^2 is the same as 'a' squared.

note this step

If the step noted is skipped it is harder to see that you are dividing by zero.

formatted by Thryduulf 15:59, 9 September 2005 (UTC)[reply]
Thryduulf - Keep in mind that he's talking about his math classes, not about invalid proofs. pile0nades, you are correct, both sides are the same, and you use the same 'method' as me, but you should always check with your teacher first to see if they accept that sort of working on tests. splintax (talk) 16:39, 14 September 2005 (UTC)[reply]

I'd like to make a navigation box to clear up an immense list of specializations in the forensic science article, and I want a fitting image as in the navigation on common law. Has anyone got a good idea on what image to use? - Mgm|(talk) 20:47, September 8, 2005 (UTC)

How about a microscope? Or even a magnifying glass? Although that tends to get used a lot on the Internet for "search"ing. Zoe 21:41, September 8, 2005 (UTC)

A fingerprint, surely. Gdr 23:05:56, 2005-09-08 (UTC)

A couple of blood spatters? alteripse 01:07, 9 September 2005 (UTC)[reply]

  • I like your ideas, and thought of some of them myself, but the problem is licensing info. Only the fingerprint appears to be a free image. - 131.211.210.12 07:10, 9 September 2005 (UTC)[reply]
  • I've decided to go with a skull so it contrasts against the table background. Please visit forensics and let me know what you think. - Mgm|(talk) 10:01, September 9, 2005 (UTC)
    • It looks a little grim. I'd be happy to make you a little fingerprint icon (if you use it, great, if not, no hard feelings) tomorrow. I like making little icons. How about a little fingerprint with a DNA strand in the background? --Fastfission 05:33, 11 September 2005 (UTC)[reply]

Number with most divisors in set[edit]

I've been asked how to find out what number in a set of 1 to 1000 has the most divisors. Is there any formula that can figure this out easily? Thanks --Colonel Cow 22:09, 8 September 2005 (UTC)[reply]

I assume the divisors must be integers greater than 1 prime numbers. You would like the divisors to be as small as possible. If they are allowed to be equal to each other, then you should let them all be 2. If they must be distinct, then let the first be two, the next three, then four five, and so on. — Pekinensis 22:34, 8 September 2005 (UTC)[reply]
Sorry, I misinterpreted your question and was sloppy in my response. In the future I will only answer math questions when I'm sober. — Pekinensis 13:35, 9 September 2005 (UTC)[reply]
See Highly composite number. You may also be interested in Abundant number. I don't think there's a precise formula for this problem. ᓛᖁ♀ 22:37, 8 September 2005 (UTC)[reply]
Looks like Highly composite number explains it to me. Thanks for your help guys --Colonel Cow 22:59, 8 September 2005 (UTC)[reply]

Divisor function[edit]

In Divisor function#Definition, how is calculated? Specifically, what are the variables the article refers to? ᓛᖁ♀ 23:09, 8 September 2005 (UTC)[reply]

Just a few paragraphs above? Under "The consequence of that is, if we write [...]"? Try that. ;) ???????? ?–? 23:17, 8 September 2005 (UTC)[reply]
r is the number of distinct prime factors of n, and ai is the power to which the ith prime factor is raised. For example, if n is 24, then:
so:
Gandalf61 10:48, September 9, 2005 (UTC)

Will there be a HD compitable CRT monitor/TV?[edit]

Why all the HDTV has to be Plasma/LCD display? Actually can a HDTV be made using the CRT technology?

Yes, you can make a CRT HDTV. For instance, CRT computer monitors can easily cope with the resolution, colour, and refresh rate requirements to display HDTV; in fact, in most experts' opinions CRT displays still have better colour fidelity, brightness, and avoid the refresh rate problems that plague LCDs. The reason why plasma and LCD HDTV are more popular than CRT's is simple; HDTV is most useful when you have a really big picture size; however, CRT's have the unfortunate problem that their depth increases proportionally to their screen size. A CRT comparable in size to the typical HDTV plasmas now becoming popular would take up most of your lounge room, even in an outer suburban McMansion to which it seems most of the HDTV sets in Australia disappear off to. --Robert Merkel 13:56, 21 September 2005 (UTC)[reply]

body transference[edit]

what has to match? blood type/dna yes/no or what else or is dna/blood type a facter? is it possible to put a male brain into a female body what are the odds of the transfer being a success? what are the chances of memory loss?

I believe the article for you is whole-body transplant, which says this is impossible with current technology, but might happen someday. — Pekinensis 03:18, 11 September 2005 (UTC)[reply]
Ask Pekinesis says, currently such a thing is not possible. You might look at it as a form of organ transplant though — with a brain instead of, say, a liver, and many of the basic tissue issues are probably the same. It is likely at least an order of magnitude more difficult and more complicated, though, to do it with something like the brain. There's no data on memory loss or success rates since it has never been rigorously attempted in my knowledge, much less enough times to develop a statistical sense of things. --Fastfission 05:10, 11 September 2005 (UTC)[reply]
Given the technology to perform a brain transplant at all, putting a male brain into a female body or vice versa would be no additional complication. Transsexual people already have such a brain/body gender mismatch. 195.224.75.71 11:53, 19 September 2005 (UTC)[reply]

bananna[edit]

1 what make banana spoil

2 explain how you no what make them spoil and how long do it take for a bannana to spoil what data do you collected to see what make them spoil

Bananas spoil as cells break down, bacteria take over, and other exciting things covered in our article on decomposition. You might try taking a number of bananas and keeping them in environments with different temperatures; see how long it takes for them to spoil in a hot place versus a cold place. --Fastfission 21:10, 11 September 2005 (UTC)[reply]
You should store bananas as room temperature (never put them in the refrigirator). I think the spoiling is a reaction with oxygen. --R.Koot 21:13, 11 September 2005 (UTC)[reply]
But won't a refrigerator limit the amount of oxygen they are exposed to? That's where my nanners are sitting right now. Well, there's one less in there now after reading this, anyway. Garrett Albright 15:27, 12 September 2005 (UTC)[reply]
I know someone who puts blackened (ergo softened) bananas in the refrigerator to use in banana bread recipes later on; apparently softened bananas are easier to cook with. I suppose placing the black bananas in the fridge at that stage helps slow down the process of total decay, with your molds and bacteria eventually taking over the fruit under regular conditons. But yeah, from my experience, bananas ripen better in open air at room temperature. --Dynamite Eleven 21:52, 13 September 2005 (UTC)[reply]
True. In the (work) kitchen I use old (blackened) banana's to make Banana Bread (beacause they are softer and easier to work with). They are kept in the refrigerator, otherwise they would most definately go rotten, and their quality is not reduced in the refrigerator. The fresh banana's are also kept in a refrigerator, which does not affect them at all. --Ballchef 15:57, 17 September 2005 (UTC)[reply]

PDF to image[edit]

Does anyone know a good free and easy to use PDF to image converter? The first I tried kept asking for a dll after I downloaded it and gave it the path, and also didn't give me the chance to select the file I wanted to convert. The second worked fine, but included a watermark without telling me. Any suggestions? - Mgm|(talk) 22:25, September 11, 2005 (UTC)

ImageMagick appears to support PDF. Failing that, it also supports PostScript, so you can print the individual pages to seperate files using the print dialog, then running ImageMagick on those. --Pidgeot (t) (c) (e) 22:33, 11 September 2005 (UTC)[reply]
  • It seems to support the right files, but I'm at a loss as to how to download the thing. - Mgm|(talk) 22:47, September 11, 2005 (UTC)
Go to http://www.imagemagick.org/script/binary-releases.php and select a Windows package (it shouldn't matter which one, but if you want to be certain you're not missing anything, choose a static one). After installation, use convert from the command-line. Exactly what options you need for PDF conversion, I don't know - you'll have to read the documentation for that. --Pidgeot (t) (c) (e) 00:07, 12 September 2005 (UTC)[reply]
  • ImageMagick should run on Windows 98/ME/NT4/2000/XP. It is recommended to use an NT-based version of Windows (NT4, 2000 or XP). Starting with ImageMagick 5.5.7, older versions such as Windows 95 are not supported anymore. Where can I get an older version. I still run Windows 98 and I don't have permission nor do I want to change to a newer version of Windows. - Mgm|(talk) 07:31, September 12, 2005 (UTC)
    If you'll note it states it will run on 98, but not 95. You should try the current release first. --Pidgeot (t) (c) (e) 08:57, 12 September 2005 (UTC)[reply]
    (And honestly you should think about upgrade your operating system! 98 was very buggy and is no longer supported at all. I upgraded my 500 mHZ PIII from 98 to XP and it works a million times better AND faster than it ever did before. No joke. I've never had a problem since upgrading.) --Fastfission 12:45, 12 September 2005 (UTC)[reply]
  • Are you sure? I've got a snaily 266 MHz which isn't mine per se, so I can't change the operating system. But I'd expect the thing to fall apart if I try to install it. Besides, using Windows 98 keeps out a lot of viruses targetting new Windows systems. - 131.211.210.12 13:29, 12 September 2005 (UTC)[reply]
  • Well, that might be pushing it -- didn't realize it was that low! You know, you could get something three times that speed for next to nothing these days... --Fastfission 00:42, 13 September 2005 (UTC)[reply]
Ah, if only you had a Mac. Open with Preview (Mac OS X's built-in image viewer), select File>Save, select an image format, done. Garrett Albright 15:25, 12 September 2005 (UTC)[reply]
Ah, if only you didn't have a Mac. Don't Open with Preview (Mac OS X's built-in image viewer), select File>Save, select an image format, done. Garrett Albright 15:25, 12 September 2005 (UTC)[reply]
  • Adobe Reader (the program you use to view a PDF file) has this facility in built - there should be no need to use any additional program. Click on the little icon of a camera surrounded by a dotty line (it's called the Snapshot Tool). Drag around what you want to export as an image (the whole page if you want), then go to Edit and then select Copy. Or press CTRL + C. Then simply paste this into any image program (or MS Word). Proto t c 12:33, 20 September 2005 (UTC)[reply]

Secure Digital: MiB or MB?[edit]

For Secure Digital memory cards, are the capacities quoted in MiB (10242 B) or MB (10002 B)? The CompactFlash article claims that capacities for that type of memory card are in MB. Andrew pmk | Talk 23:09, 11 September 2005 (UTC)[reply]

Eh? 1 MB = 10242 bytes as well. I believe that MiB is just the French abbreviation for the term or something. Garrett Albright 15:22, 12 September 2005 (UTC)[reply]
No, "MB" is ambiguous. See Mebibyte. Bovlb 15:29:29, 2005-09-12 (UTC)
In French, we use "Mo" (megaoctet). Andrew pmk | Talk 20:34, 19 September 2005 (UTC)[reply]
I would guess that since it's a memory device, it would be 1MB = bytes. They could be nice and have 1MB = bytes also. Alphax τεχ 14:48, 14 September 2005 (UTC)[reply]
To clarify: they will take "kB" to mean 1024 bytes, "MB" to mean 1000 kB. If they can rip you off, they will. The only place they won't use the powers of 10 is on RAM (for historical reasons). Alphax τεχ 10:26, 15 September 2005 (UTC)[reply]
Actually, I would expect that the MiB (2^20) is the correct interpretation because flash memory is a form of RAM. Rather than "historical reasons", 2^10 is the kilo- basis for RAM because it logically conforms to the need for individual addressing lines. A 20-line device can address 2^20 distinct units (1 MiB). While a 1 MB (10^6) RAM device could be constructed, it would still require 20 address lines -- so why not fill out all the lines? Hard drives / CD-ROMs / tape drives are all non-RAM devices (in varying degrees, HDDs are not purely sequential) and so don't have the logical connection between address and size. — Lomn | Talk / RfC 16:04, 15 September 2005 (UTC)[reply]

disodium phosphate[edit]

What is disodium phosphate and how is it used by the body?

Sodium phosphate is a salt and a buffer. In the body Na and phosphate ions exist in the extracellular and intracellular fluid in significant amounts. The amount of Na and H bound to the valence-3 PO4 anion varies with pH. The principal "use" of PO4 in the cells of the body is as a detachable moiety for high enery phosphorus compounds like ATP. Another principal use is as part of bone mineral in combination with calcium. There are pharmaceutical preparations of sodium phosphate as a phosphate source for intravenous or oral replacement of phosphate depletion (e.g., for treating people with diabetic ketoacidosis or X-linked hypophosphatemic rickets ), or as a buffer for those with certain types of impaired renal acidification. alteripse 11:24, 12 September 2005 (UTC)[reply]

Calculating energy released by nuclear fission[edit]

Hi all,

I'm just writing a basic high school paper on nuclear fission, and in order to illustrate its efficiency over regular chemical reactions, I'm trying to find the amount of energy released by complete fission of 1g of uranium-236.

From Nuclear fission, a fission reaction produces about 200 MeV of energy, and according to google, this is equal to 3.20435292 × 10-11 J. So I tried the following equation, but the end result seems to be horribly wrong..

n(U-236) = m/M = 1/236 mol (Number of moles in a 1g sample of U-236) N(U-236) = n * N(A) = 1/236 * 6.02 * 1023 ≈ 2.55 * 1021 atoms (Number of atoms in that sample) E(U-236) = N(U-236) * 3.2 * 10-11 ≈ 81.6 GJ (gigajoules) (number of atoms multiplied by energy released by each fission in joules)

So am I using incorrect numbers to start with, is this procedure invalid, or does 1g of uranium really yield 81.6GJ? o_O

Thanks in advance for any help, as always.. splintax (talk) 14:26, 12 September 2005 (UTC) [reply]

Just as a note, I'm pretty sure you mean u-235, not 236. (238 is natural uranium, 235 is what is usually meant by enriched, though I believe 233 can also be as well). --Fastfission 00:13, 13 September 2005 (UTC)[reply]
No, he got it right - What happens is that U-235 absorbs a neutron to become the very unstable U-236, which then almost immediately fissions.
As to the calculations, I think it's about right - it's only about 22,000 kWh, or about 0.02 kilotons. The Little Boy article claims that of the 60 kg of HEU in that bomb, 0.7 kilograms underwent fission. 700 * 0.02 = 14 kilotons, close to the reported yield of 13 in the article.
As to why the figure seems so high, even for nuclear fuel, you have to keep in mind that typical reactor fuel is, apparently, enriched to only somewhere between 2 and 5 percent U-235, burnup in the reactor is incomplete, and not all the energy released by the nuclear reaction is converted to electricity. --Robert Merkel 13:57, 13 September 2005 (UTC)[reply]
D'oh, you're probably right about 256. I didn't even think about the fact that it was absorbing the neutron when it was undergoing fission. --Fastfission 12:57, 14 September 2005 (UTC)[reply]
Thanks everyone. Glad to see that I was right on all counts - and especially thanks to you, Robert Merkel, because you told me exactly what I needed to hear - why the amount of energy released is apparently so high (since obviously it must be less). Thanks a LOT everyone! :D splintax (talk) 15:26, 14 September 2005 (UTC)[reply]

Transistor or Transformer?[edit]

1) Does a remote control have a transformer or transistor inside it?

2) Is inside the AC adapter for a CD player a transformer?

--John
1) I suspect there are many transistors but no transformers, as remotes tend to be powered by batteries.

2) Yes --inks 23:32, 12 September 2005 (UTC)[reply]

Why Capacitor and Inductor?[edit]

Why are there capacitors and inductors since batteries already do the job of storing electric charge or voltage?

--John

John, batteries, capacitors and inductors all have different properties that are related to the way they store energy. Batteries store energy as chemical energy, whereas capacitors and inductors store it in electrical and magnetic fields, respectively. As I understand it, the latter tend to have simpler and easier to calculate properties; for example, they charge and discharge according to a simple negative exponential function with time. Batteries, on the other hand, have more complicated characteristics.
The upshot of this is that capacitors and inductors are useful in a wider range of situations. Capacitors, for instance, can be used to filter DC from a circuit while allowing AC to flow, while inductors do the opposite. --David Wahler (talk) 01:44, 13 September 2005 (UTC)[reply]
Also, they can be used to selectively filter out certain AC frequencies... the articles RC circuit and LC circuit should provide more info. Alphax τεχ 14:48, 14 September 2005 (UTC)[reply]

sending page as email in Firefox[edit]

When browsing in Mozilla`s Firefox, how can a page be sent as email, instead of the link? Thats the only option i see, except for saving it to file, and then sending. Thanks uncleed

MHTML may be what you want. This requires a Firefox extension [11]. —Masatran 10:31:06, 2005-09-12 (UTC)
I seem to remember using CTRL-A to select all and then CTRL-V to paste in the compose window inserts a perfectly formatted page into Google's Gmail. May work in other contexts. I seem to remember being told you need to use CTRL-A rather than trying to highlight everything with the mouse for it to work. --bodnotbod 02:56, 19 September 2005 (UTC)[reply]

Computing: Programming Language for a Small Project[edit]

So I've discovered that my MP3 player plays files in alphabetical order, by file name. This is a good thing, since the player has no other way of ordering song selections, except a cumbersome playlist editor that I've given up on in frustration. So now if I want to play files in order, I can just copy them onto the player and then change their file names - to a, b, c, d, what have you.

Now, let's say that I wanted to write a program to save myself the trouble of changing each and every file name. I'm imagining something that has a window that I can drag-and-drop the songs I want into it, in order - then I can click a button and it'll write the files to the player, using alphabetical names.

The problem, of course, is that I haven't done any programming since my QBASIC days.

So, what's the simplest programming language, compatible with Windows XP, that will let me write this program I have in mind? Also, what would be the cheapest? Are there any other options that I have that might be able to do the trick?

Thanks. --Brasswatchman 17:33, September 12, 2005 (UTC)

One solution: do you have Microsoft Excel? If so, it would not be hard to write a little VB Script program which would 1. import all of the filenames from a given directory into a spreadsheet (at which point you could use formulas to rename them etc.) and 2. go through and replace any given filename with another one. --Fastfission 00:03, 13 September 2005 (UTC)[reply]
There are many different options - Delphi, C/C++, C#, Python, Visual Basic, Java... pretty much any language capable of doing Win32 GUI programming can be used for this (and as a matter of fact, QBasic can as well by executing shell commands and reading file names from a text file, though this is slightly more complex than plain drag-and-drop).
Personally, I quite like Delphi, and there are plenty of tutorials, etc. available. It's not free (you can find a Personal edition in some magazines as a promotional offer, meaning you only have to pay for the magazine), but if you'd rather use a free alternative, Free Pascal should work as well.
C, C++, C#, Python and Java are all free to use (compilers are readily available) - however, C and C++ are probably not a good idea for this case, since they're quite complex. Based on what I've been told, Python's supposed to be quite good.
I'm not sure if there are any free compilers for Visual Basic, but I don't think so. --Pidgeot (t) (c) (e) 00:20, 13 September 2005 (UTC)[reply]
Great. Thanks. --Brasswatchman 22:44, 14 September 2005 (UTC)[reply]
Alternatively, use something like metaprogramming to read a list of your files, and then print out a Windows shell script, something like
move filename1.mp3 aaa_filename1.mp3
move filename2.mp3 aab_filename2.mp3
and then just execute that shell script. Dysprosia 03:56, 16 September 2005 (UTC)[reply]
I would write it in Python with a PyGTK GUI if I need one (requires intallation of GTK). I would prefer high level languages like Python, Perl, Java or C# for something like this (all of these are free except for perhaps C#). C++ and especially C lack a good standard library. Maybe I that doesn't matter much in this case but often that results in unnecessary code. I have not tried to do drag and drop using the Win 32 api but I know that it's quite simple in GTK. Jeltz talk 11:03, 18 September 2005 (UTC)[reply]
Am I the only one curious to know which mp3 player it is that plays songs alphabetically (rather than allow uploading of a playlist, presumably) and has a playlist editor so bad that someone will take up programming rather than use it? --bodnotbod 03:00, 19 September 2005 (UTC)[reply]

Grounding in Electricity[edit]

  1. Please explain to an extremely inquisitive 12-year-old child what grounding means in electricity?
  2. What is a grounded connection?
  3. Is grounding only for AC circuit? Can it be used for DC circuits, too? I have heard it used in connection with household electricity, which is AC.
  4. What is the relation of grounding to getting an electric shock or electrocution?

--Geoffrey

You might be interested in reviewing the following Wiki articles (and some of their external links):
  1. electricity;
  2. electrical conduction
  3. electric shock
  4. surge protector for a computer
AlMac|(talk) 07:56, 13 September 2005 (UTC)[reply]
# We might also be interested in a discussion of static electricity in this context ... how to behave to lower or increase the risk of getting a shock from that. AlMac|(talk) 07:49, 13 September 2005 (UTC)[reply]
  • There was a question similar to this above, and the link Why grounding? was given, it might be useful.
  • Another wikipedia link here has some infomration, although it's a little complicated.
  • I'll try to give my understanding, at a 12 years old level, forgive me if it's incorrect. Firstly, electricity travels from places with high volatge to places with low voltage and electricty takes the least path of resistance. As you might expect, the ground (or earth that we stand on etc, the floor of our homes) has a volatge of zero, so electricy always wants to travel to the ground. A grounded connection is one that is connected to the ground. This means that if you apply some current to a grounded connection, the current will flow to the ground. In some cases (eg a lightning strike, or a dodgey toaster) a person may complete the circuit from the current to the ground. This means that the electricity will flow through them to get to the ground, electrocuting them in the process. A ground wire (that you have in your home) can prevent electocution by providing a path of lesser resistance for wayward current, so its travles down the the wire to a stake into the ground, rather than through your body. I'm seem to remember using a ground when I made a DC circuit a while back, but I'm not sure if the principle is different or AC or DC.--Commander Keane 09:36, 13 September 2005 (UTC)[reply]
An alternate explanation: The Earth is really big right? So you'd expect that it has a lot of charges in it that can move around just by virtue of the fact that it's so big. Well, what happens if I put something with a net positive charge against the ground? Charge flows to try to equalize potential differences (or rather, potential differences exert a force on charges causing them to move in the direction that leads to an equalization of potentials). So the charges in the Earth and that object touching it will flow until the two are at the same potential. Because the Earth is so big, you could do this with many many positively charged objects and never be able to tell a difference in the net charge or potential of the Earth itself. You could repeat everything I just said with negatively charged objects. We tend to say that the ground is at zero potential because of the fact that it (roughly) doesn't change potential and zero is a convienant number.
You are usually electrocuted by grabbing onto something at nonzero potential while standing on the ground. So for instance, (don't try this at home) if you had a trampoline below a power line and jumped up and grabbed the bare power line, nothing bad would happen to you. But let's say that you are standing on a metal ladder without thick rubber shoe soles and grab a power line; you'll probably get electrocuted pretty bad because suddenly you're in electrical contact with both a high voltage source and the ground. — Laura Scudder | Talk 18:30, 13 September 2005 (UTC)[reply]
Do you ever see birds sitting on electric wires overhead? They do not get electricuted, do they? That's because they are ONLY touching the dangerous wire, and not some other place for the electricity to go. AlMac|(talk) 07:20, 15 September 2005 (UTC)[reply]

order of reaction between iodide ion and persulfate ion[edit]

(no question)

  • You'll have to be a bit more specific than that. Do you want to know the difference in their reactivity? Or the order by which they react with each other? The last one should be simple, they don't react with one another. - Mgm|(talk) 08:13, 13 September 2005 (UTC)[reply]
I have never hear of this before now. I would guess it is an aggrsssive oxidizing agent and mixing the two together
would liberate elemental Iodine.
  • The question seems sufficiently specific on its own to me: the person is enquiring about the Order of the reaction. And they certainly do react. The reaction as well as the rate equation (from which the order is easily found) can be found here. Chuck 21:36, 15 September 2005 (UTC)[reply]

Query about a computer security term[edit]

Could someone fill me in on what an escalated privileges attack is? It's mentioned in MDAC, but not well explained. If someone wanted to give it a shot in filling it in as a stub, maybe that would be helpful also. Anyone got any ideas? - 203.134.166.99 08:12, 13 September 2005 (UTC)[reply]

If you look at the external link referenced near the paragraph that mentions the attack, you'll find a link to go to Microsoft's Security Glossary of Terms. There, the following is mentioned about this sort of attack:
Privilege elevation
The ability of a user to gain unauthorized privileges on a machine or network. An example of privilege elevation would be an unprivileged user who could contrive a way to be added to the Administrator's group.
In other words, an escalated privileges attack is an attack that exploits a flaw in a program to gain access to things usually unavailable to the attacker. --Pidgeot (t) (c) (e) 10:39, 13 September 2005 (UTC)[reply]
What can be done with certain programs is to have them run under a certain different user, even though you still are using your account. This is done to let certain programs do things you'd need privileges for, like a program to format a disk, for example. Privilege escalation means that you deliberately "break" the program so you can get access to a shell, or to run a nasty tool, or whatever. Say you want to get a shell. Since the program is running under a privileged account (maybe even root, God forbid), then the shell that you create will be running under that privileged account too and you will have then the privileges associated with that account accessible through the shell.
Of course, you can invert this idea, and use privilege limited accounts to run certain programs that don't need these privileges, so if the program is "broken" for you to spawn a shell, you can't do anything.
This all is however a bit of an oversimplification, but I hope it gets you the right idea. Dysprosia 03:53, 16 September 2005 (UTC)[reply]

What is the chemical nature of the food reserve in the corm of snowdrop?

Pace of speciation[edit]

What is the current pace of speciation relative to different periods? On what factors does this depend? -- Sundar \talk \contribs 10:08, 13 September 2005 (UTC)[reply]


Computing[edit]

Can you list 4 items to look for when determining the reliability of a web site? What is flag searching? Thanks

Depends on what you mean by "reliability." Are you referring to the reliability of online stores to actually deliver the products you pay for? If so, you could try searching or asking around online for others' experiences with dealing with the store. You could also find out where the physical headquarters of the store is located and contact the local Better Business Bureau for the area. Garrett Albright 11:22, 14 September 2005 (UTC)[reply]

I mean the reliability of information of a web site. For example, spelling. If they misspell a lot in that web site, so it's hard for us to belive its information. Do you know 3 more items? And what is flag searching? Thanks a lot.

Software used to update web sites is often user-hostile when it comes to doing spell checking. When you do e-mail, or word processing, it comes with spell checking, and it also comes with the ability of the end user to mess it up. The vast majority of web sites are created to look good from the perspective of the web browser version being used by the person who created the web site. Most web sites are hostile to people using other types of browsers, older computers, and older humans, whose eyesight has more trouble with color contrasts. Do you know how many people have disabilities, like blind, key stroke problems ... it is a huge percentage of the population that has some kind of disability, and most web sites are hostile to them. But most people could care less. More info on this kind of topic at http://en.wikipedia.org/wiki/Wikipedia:WikiProject_Usability AlMac|(talk) 07:37, 15 September 2005 (UTC)[reply]

Why is ist important to back up files and data?[edit]

Why is it important to back up files and data? What type of Operating System would be best for a school environment?

  • Short answer to your first question: It provides you with a copy of your files in case your computer crashes, when you're hit with a virus or when something else causes you to lose data. - Mgm|(talk) 08:40, 14 September 2005 (UTC)[reply]
Take a look at the Wikipedia article Backup for some help with the first question. Your second question is difficult to answer because it lacks context. What level of school, what hardware (if any) is already available, and so on. hydnjo talk 00:09, 14 September 2005 (UTC)[reply]
Commodore DOS or ProDOS, I say... :) --Robert Merkel 00:22, 14 September 2005 (UTC)[reply]
As for the OS, assuming the choice is between Linux and msWindows I'd say Linux makes more sense for three reasons. Firstly, with Linux (or any other Unix) the pupils will get in closer contact with the computer and thus learn more. This would be a reason assuming the purpose is to let pupils learn about computers. But that can also be seen as a positive side-effect. msWindows shields the user from the hardware, which is more practical in everyday use when all goes well, but can be frustrating if things go wring or if you want to do something special. Linux is much more powerful. Put differently, if you use msWindows you learn msWindows, if you use Linux you learn the inner workings of the computer. That requires an intellectual investment, but, hey, we're talking education, right?
The second reason is cost. I'm not entirely sure about this, but I believe that for msWindows you need to pay a license per computer, whereas Linux is a one-time purchase (about 50 euro in stead of several times a few hundred euro). At least, for a distribution like Debian, Suse or Red Hat. You could even download it for free, but then you'd need an expert to install it. Additionally, with a Linux distribution you get a whole bunch of software for free and installed straight away. With msWindows you'd have to buy all that software separately (which costs money) and install it (which costs time). In case you wonder about ease of installation, the basic installation of Linux is simple. Installing drivers and extra software is often more difficult, but less often necessary than with msWindows. (And I've heard Debian is easier for this because it resolves dependencies automatically - the major pain with Linux.)
The third reason is more ideological; choosing Linux means not choosing a specific commercial OS (ie msWindows) and thus 'force' pupils into that corner of the market. On the other hand it makes sense for pupils to learn the de facto standard in personal computers, which is msWindows (this does not go for 'professional' use like servers, though, which usually use some Unix). But for exactly that reason they'll get sufficient exposure to it in daily life, so they don't need to be taught that at school.
As Hydnjo points out, the hardware is relevant. If the computers are brand new there may be a problem with the availability of drivers. But then that is easily tested at a relatively small cost. And if in the end you choose msWindows you can always install Linux as well. In this case you'd have to either use a dual boot and install msWindows first (it would overwrite a previous installation of Linux - Linux is much kinder to other OS's and doesn't do that) or install them on different hard disks and use the bios to choose the drive, and thus the OS, to start (press 'delete' (usually) during boot up).
One last irritating remark. Don't worry if you don't understand. Your pupils will :) . DirkvdM 07:26, 14 September 2005 (UTC)[reply]
The Apple Macintosh is the best computer for just about any environment, including the educational one. Many of the complexities DirkvdM mentions above will not be an issue with Macintosh computers. While Linux computers will be less expensive, they will also be more complex, and they will not have the wide range of educational software available on the Mac. Microsoft Windows-based computers are the most popular in the "real world," but they are also the worst for the educational environment (or any other environment, actually) due to their weak security and the wide range of computer viruses and spyware which will be bound to infect the system; you will be doing much more maintenance on Windows computers than Mac or Linux machines.
You're right, I forgot about Macs, which I don't really know much about. But that's a hardware choice. Which is also a coice between msWindows and MacOS, put simply. But Linux runs on both Macs and 'pc's'. Wich is another advantage of Linux. It runs on any computer (well, maybe if you find something really, really exotic...).
By the way, 'pc' may not be such a good term, but 'msWindows-based' is even worse because that refers to software and we're talking hardware here. They're sometimes called Intel-based although that term isn't really correct either since there is also AMD (and others). DirkvdM 16:16, 14 September 2005 (UTC)[reply]
Also consider the purpose of the computer(s) for the school ... there is a huge difference between
having computers to help the students learn something about computers ... in which case the anser might be exposure to a variety of OS to show that there is not one way things are done
having computers to help the students learn stuff unrelated to computers ... such as drill and practice in an environment where you not want any computer hassles ... in which case an Apple Macintosh might make the most sense
having computers to help the faculty and staff keep track of grades, attendance, lesson plans, etc. ... many schools use an AS/400 for that
AlMac|(talk) 07:26, 15 September 2005 (UTC)[reply]

As for backup ... basically if it does not matter if you lose everything that is on your computer ... e-mail, files, documents, jokes, games, software, records of what you bought and paid for so you can prove you are a licensed user of something on the computer when you go to get help putting it back on after a crash, then you do not need a backup, or a computer either. But if what is on your computer is important to you, and you want to minimize the recovery time after a crash, melt down, virus, theft, natural disaster, etc. then you need to have various kinds of protection, such as surge protector anti-virus off-site backups, etc. AlMac|;(talk) 07:31, 15 September 2005 (UTC)[reply]

Nothing to do with the question; rather, the response. I despise it when people decapitalise the MS part; you wouldn't like it if I started deliberately misspelling Lunix. Whoops. :P Rob Church Talk | Desk 23:34, 20 September 2005 (UTC)[reply]

Plant and Animal identification[edit]

I recently uploaded some photographs, among which some 20 of plants and animals that I've only partially managed to identify. Here are some of those I haven't a clue about. For more thumbs see User:DirkvdM/Photographs#Plants_and_Animals, with, for example, a poison dart frog and a hummingbird, but which ones specifically? DirkvdM 07:41, 14 September 2005 (UTC)[reply]

whatisthis_costa_rica-2.jpg
panama_what_insect_is_this.jpg
what_lizard_is_this.jpg
Glasswing_butterfly_Panama.jpg


The second insect (from Panama) looks like a leaf beetle. Gdr 20:56, 14 September 2005 (UTC)[reply]
Actually I'm fairly sure both of the first two are Heteropterans. The first one is pretty clear. You can tell by the way their wings cross over their back. That doesn't narrow it down very far though, there are thousands of species in that suborder. More Heteroptera here. You could look through the subgroups there and maybe get an idea. Or you could suffice it to say they are true bugs. - 207.74.176.195 02:28, 15 September 2005 (UTC)[reply]
Neither the wikipedia-articles on leaf beetles/chrysomelidae and heteropterans nor image-Googling those terms gives any images anything like these. I appear to have photographed some pretty rare insects if these classifications are correct :). Ironically, though, image-Googling 'insect panama' gives one image of the second insect. No indication what it is, though. DirkvdM 19:06, 17 September 2005 (UTC)[reply]
I believe that the butterfly in the picture is what is commonly refered to as a glasswing butterfly. --Aluion 08:52, 16 September 2005 (UTC)[reply]
It is (I image-Googled it). It's scientific name appears to be Godyris duillia. But when I image-Google that I get many images of a rather different looking butterfly (just one that looks a bit like mine). But I suppose 'Glasswing' is at least right. Thanx.
I'd like to point out once more that there are many more photographs that need identifying at User:DirkvdM/Photographs#Plants_and_Animals. These four here are just meant as an eyecatcher. DirkvdM 19:06, 17 September 2005 (UTC)[reply]
I'd also say that the first two are heteropterans. I'd venture to say that the first one (from Costa Rica) looks like a Reduviid (aka Assassin) bug. Tjunier 22:45, 19 September 2005 (UTC)[reply]

Wikipedia search bar in browser[edit]

Is it possible to get a dedicated search bar for wikipedia at the top of my browser. WP has rapidly become my first stop for just about anything I'm thinking about. Also, would there be a way to get it to automatically open the article in a new tab in Firefox? The Google search bar opens on the same page, which I find annoying.

Thanks! -Mary

If you download the tabbrowser preferences extension, and the Mycroft Project plugins, you can do this. Alphax τεχ 15:06, 14 September 2005 (UTC)[reply]
In Firefox I simply enter wp foo in the address bar and that takes me straight to the page. But I can't recall if that is a standard behaviour or something I taught it with a keyword. Give it a try. --bodnotbod 03:02, 15 September 2005 (UTC)[reply]
You are using the Quick search feature. I think the wikipedia one is by default. See also Wikipedia:Searching#Using_Mozilla_to_automatically_search. -Fang

Mistery picture: what insect it this?[edit]

File:Mistery insect.jpg
Mistery insect

Today I found this stange insect hanging on a leaf of my Oleander. The picture is misleading: that thing is very small, long no more than a couple of millimeters. I've never seen it before - does anybody know what it could be? Thank you. --Davide125 13:52, 14 September 2005 (UTC) [reply]

You have a green lacewing. Here are some pictures. Couldn't tell you the species - 207.74.176.195
To me, it looks more like a mayfly. Tjunier 22:53, 19 September 2005 (UTC)[reply]

Limit of the number of characters in a filename[edit]

What is the limit of the number of characters in a filename? Does this apply to all operating systems? Thanks.

Try looking at Comparison of file systems. For the most part, the limit is 255 characters, but there might be additional limits imposed by the operating system, although these are fairly rare these days. --Pidgeot (t) (c) (e) 18:23, 14 September 2005 (UTC)[reply]
In the interests of backwards and forwards compatibility, many business OS do have limitations like 10 characters for names of objects, in which the first character must be a letter A-Z, and some special characters may not be embedded within the name, like for example a blank space is not allowed. AlMac|(talk) 19:16, 15 September 2005 (UTC)[reply]
You mean we're no longer limited to 8.3? Anyways, various operating systems have various naming conventions. For instance, good old MS-DOS was limited to an 8-character name (which was not permitted to start with a number, and could not contain anything other than alphanumerics), a period (.) and a three-character extension, also alphanumeric; the origin of the expression I used above. Unix and later rip-offs had different restrictions again, and of course, Windows 95 and later incarnations had long filename support via a very messy workaround. So, the answer to your first question is, "it depends" and to your second, "no". Rob Church Talk | Desk 23:40, 20 September 2005 (UTC)[reply]
Actually as a person who wrote assembly code for MS-DOS the above is not quite correct. MS-DOS did limit characters in file names -- specifically the characters "+", "-", "&", "|", "<", ">" and " " were forbidden, because those characters had reserved meanings in parsing file names (except that spaces were allowed at the ends of names). some command line programs that were often used to enter and manipulate file names had the stricter rules you mention, but you could (and I did) create under program control files such as "21:59, 21 September 2005 (UTC)DES (talk).DES (talk)" and files that included non-printing charactrs above 127. Some comercial File-management programs such as Xtree(tm) also could create and re-name such files. Note that the filename rules for MS-DOS were inherited from CP/M. DES (talk) 21:59, 21 September 2005 (UTC)[reply]

Perhaps a more basic answer is required? Number of characters is the number presses on the keyboard to write something eg budget 2001-02.xls is 18 characters. Every operating system has a maximum allowable number of characters when naming a computer file. Today, a typical maximum is 255. --Dlatimer 10:16, 21 September 2005 (UTC)[reply]

"Number of characters is the number presses on the keyboard to write something". Not necessarily. Some character sets use "multi-byte" characters, where a single keypress (and a single character displayed on the screen) is represented internally as multiple bytes. For example say I have a file called "ファイル". This appears to be four characters long but in fact is encoded as twelve bytes. This difference may matter when using filenames which don't stick to the "standard" (USASCII) character set. Tonywalton  | Talk 21:36, 21 September 2005 (UTC)[reply]
If the file name is not on a computer, the limit is your stamina writing down an infinity of characters. Some OS and software permit files to have labels in addition to the file name, in which the label can be extremely long compared to the file name. Most computer fields are fixed format, but you can define variable format fields such that the space occuplied is what is needed, with no blanks or wasted disk. So that one object in a collection has a name that is 2 characters wide and occupies minimum bytes, while another object in the same collection has a name that is thousands of characters wide. The original question cannot be answered except on an OS by OS basis. AlMac|(talk) 21:46, 21 September 2005 (UTC)[reply]

What is the correct method for sourcing a web site?[edit]

What is the correct method for sourcing a web site (MLA - Modern Language Association)? Give an example.

Wikipedia:Citing Wikipedia has examples of how to cite this website, in various forms including MLA. Generalise to other examples. Shimgray 17:49, 14 September 2005 (UTC)[reply]

CSS font-family or FONT tag: how to avoid fallback to default font?[edit]

When a font is specified that doesn't actually exist on your computer, the browser falls back on the default font. Is there any way to disable this?

Specifically, I'd like to be able to display a chart in a Wikipedia page that says "this is what Font X looks like"... and if Font X doesn't exist, I'd like all the glyphs to be blank (or solid black, or something) rather than displaying the default font. Is there some way to do this? I already tried w3c.org, and wasn't able to formulate any useful Google search strings. -- Curps 19:18, 14 September 2005 (UTC)[reply]

I don't believe that's possible, at least not in a way that will work universally across browsers (especially text browsers, like Lynx). Consider making an image of the font instead, it would be much more user-friendly (not to mention easier to work with). --Pidgeot (t) (c) (e) 20:13, 14 September 2005 (UTC)[reply]
That's not really practical. For one thing, I don't have most of these fonts; for another, the image files would be extremely large (these are Unicode fonts). The purpose is to be able to use templates like {{Unicode chart Latin Extended-B}}, {{Unicode chart Latin Extended Additional}}, etc. under the various different fonts, in order to help refine the ordering specified in {{unicode fonts}} or {{IPA}} or {{polytonic}}. -- Curps 04:40, 15 September 2005 (UTC)[reply]
There's no way to do precisely what you're asking, but you could specify a fallback font which is very obviously wrong. An appropriate font for Mac OS might be LastResort. On Linux and Windows, dingbat fonts like Wingdings might work. —Brent Dax 18:56, 17 September 2005 (UTC)[reply]
Thanks, this idea seems to work fairly well. -- Curps 00:21, 20 September 2005 (UTC)[reply]
I have to mention my friend's creation : Fontweaver in this context. It will produce an image for each character in a font automatically. The try before you buy download might get you there, then rather than using its website integration features, which wouldn;t work on Wikipedia, you'd be able to upload the individual character images it creates. Though it would be a heavy investment of your time. --bodnotbod 03:23, 19 September 2005 (UTC)[reply]
Thanks, but that would be a lot of images and it doesn't seem practical. -- Curps 00:21, 20 September 2005 (UTC)[reply]

Increased surface area[edit]

Hello,

I am doing my biology 12 on my own and am having difficulty finding the answer to this question.

Give an example where increased surface area is required for an organism?

I'm not sure whether you mean the organism as a whole or parts of it, but you might like to think about convoluted or folded structures (hint - inside a body) that take up little volume but have a large surface area. -- ALoan (Talk) 22:04, 14 September 2005 (UTC)[reply]
A lot of animals have big ears so that they can cool off my circulating blood through the ears. The point is that the amount of heat transfer is proportional to the surface area, so the higher your surface area to volume ratio, the better your heat transfer. Same deal for lungs with oxygen transfer; brains and interconnectivity, leaves with light. Lots of stuff. -Lethe | Talk 02:32, 15 September 2005 (UTC)[reply]
Brains and interconnectivity? Are you referring to the grooves (what are they called again?) in the neocortex? But that surface doesn't connect to anything. The connections are inside the brain. Right? And now I start wondering what those 'gooves' are for (or a result of). DirkvdM 07:01, 15 September 2005 (UTC)[reply]
Sulci (singular sulcus). The "sticky-up" bits between the grooves are called gyri, and they're there because there's more brain there than you could stuff into a skull if it were smooth, basically. Tonywalton  | Talk 21:45, 21 September 2005 (UTC)[reply]
You need to be careful here - not everything has a teleological reason. Many structures are the way they are because it's easy for them to be that way. Some grooviness may just exist because it is convenient for them to form that way, and that forming that way doesn't have any particular harmful effect. Leaves are also a bad example - it's not total surface area that matters here, but surface area that is exposed to sunlight - often, more surface area is bad because of water loss problems. Most of the time, though, it's a combination of evolutionary teleology and simple practicality. For this question, they probably expect you to refer to the digestive system, or other systems where the organism is exchanging something with the outside environment. -Fang (not logged in)
Tapeworms have an increased surface area so that they can absorb digested food more readily from their host. Also many simple organisms require a large surface area for gas diffusion.--82.44.216.80 18:40, 16 September 2005 (UTC)[reply]

gray box on desktop[edit]

(copied from info-en with permission of questioner. I'll post a screenshot if neccessary. From what I've seen of the screenshot, the box has the appearance of the background of a Wikipedia page (opened grey book)) - Mgm|(talk) 21:10, 14 September 2005 (UTC)[reply]


What I did do was put a shortcut to your website on my desktop. The short cut did show up. But later at some point while I was on your site I think I may have inadventenly clicked on something. I can actual expand the square gray box and it will cover my entire desktop like it was an image I selected to be my desktop image.But I can click on it and it goes back to being a smaller image. I have gone to my control panel and looked at what is selected for my desktop image and it shows what I want to be there. It does not show the gray box as part of it. I have scrolled the different desktop images available and nothing shows up for a gray box. It actually acts like another desktop. I can put short cut icons on it!

At the very top of the gray box is a down arrow. If I "left click" the options that come up are:
Right Click
Make available offline
Customze my desktop
Split desktop with icons
cover desktop (if I click this the gray box enlarges to cover my entire desktop display)
Close

If I "right click" the options that come up are:
Arrange icons by...
Refresh
Undo rename
New
Properties (which brings up display properties options).

Nothing I do can get rid of this gray box.

I was finally able to get a copy of the desktop image. Hopefully you can open the attachment and see what I'm talking about. P.S. My husband is very knowledgable about computers and has no earthly idea what has happened. At some point in the past few days messing with this thing, something somewhere came up that did identify the image has having something to do with Wikipedia but I haven't been able to find where I saw that again.

Any suggestions?

It sounds like you've activated Active Desktop and added a webpage as an element. Clicking the Close option should remove it. --Pidgeot (t) (c) (e) 21:34, 14 September 2005 (UTC)[reply]
  • I've just received word that this issue ahs been resolved. - Mgm|(talk) 07:30, 16 September 2005 (UTC)[reply]

science[edit]

how would starch be a good source when ur dieting?

A good source of what, calories? alteripse 00:34, 15 September 2005 (UTC)[reply]

The enzyme amylase converts starches into sugars in the body. Sugars are carbohydrates, referred to, I believe, as "carbs" or "carbos" in the world of dieting. So the question could well have something to do with carbohydrates. Incidentally, carbohydrates are used by the body to provide the bulk of the energy it needs; that's what that part of your food does. Fats are also an energy source, but tend to be stored and used as needed. Hope this helps; give us something a little more specific if it doesn't. Rob Church Talk | Desk 23:45, 20 September 2005 (UTC)[reply]

Fly and coordinate system[edit]

Can anyone tell me what is the relationship between a fly and the coordinate system? It's a common example used in Algebra books. --67.115.220.195 00:28, 15 September 2005 (UTC)[reply]

  • René Descartes actually got the idea for his Cartesian coordinate system while lying down and looking up to a fly crawling across his ceiling, and realized that he could define the position of the fly by just two numbers: the distances of the fly from each of the two nearby walls.--Pharos 04:55, 15 September 2005 (UTC)[reply]
Nothing to do with the question, per se, but I've only just realised that that's where we get the expression Cartesian co-ordinate from. Rob Church Talk | Desk 23:46, 20 September 2005 (UTC)[reply]

The fifth dimension - does it exist? What is it?[edit]

If it has any meaning at all, and I have reason to believe it does, how would a mathematician attempt to describe "the fifth dimension" to a layperson?

And please note: the answer "with great difficulty" would be less than helpful ;o) --bodnotbod 00:41, 15 September 2005 (UTC)[reply]

If you are not already familiar with it, you might look for ideas in Abbott's Flatland, which does a nice job of exploring the difficulties of communicating the concept of a a higher dimension to those whose perceptions are limited to a lower number of dimensions. alteripse 01:19, 15 September 2005 (UTC)[reply]

from a mathematical perspective, it's easy to describe a fifth (or indeed many more) dimension. what's hard it imagining pictures extra dimensions in your head, not describing them mathematically. The number of dimensions a space has can most easily be explained to the layman as the number of numbers required to describe a point, the number of degrees of freedom. for example, to describe the position of a body requires 3 numbers, so space is 3 dimension. To describe the time and place of an event requires 4 numbers, so spacetime is 4 dimensional. to describe the position and velocity of an object requires 6 numbers (3 coordinates of position, 3 of velocity), so the phase space of an object is 6 dimensional. The phase space of 10 particles is 20-dimensional. To describe these mathematically requires only the imagination to write down a list of 20 numbers. the number of independent variables in models of complex systems like the Stock Market may be in the hundreds or thousands, meaning that the solutions to those equations live in spaces with thousands of dimensions. It's actually quite easy to describe. Just don't ask for a graph of a trajectory in this space. Obviously, higher dimensional spaces have just as valid a mathematical existence as low dimensional spaces.
But I suppose what you really wanted to know about was the possibility of extra spatial dimensions of our Universe, as found in Kaluza-Klein and string theory. It's the same as before. If those dimensions exist, it takes more than three numbers to specify just the position of a single particle. The reason they're not noticeable is that while the first three numbers range from subatomic scales to meters and up into the millions of lightyears, the extra dimensions only take on values on the subatomic scale, which is invisible to the naked eye. HTH -Lethe | Talk 02:26, 15 September 2005 (UTC)[reply]
OK. I looked at string theory and Kaluza-Klein. I'm part of the way there. If I am referring to a fifth spacial dimension, is it safe to say that such a dimension is - thus far - only theoretical? Or is a fifth spacial dimension accepted? I'm assuming the former. --bodnotbod 03:09, 15 September 2005 (UTC)[reply]
Kaluza-Klein isn't really a serious theory these days, but string theory is. String theory predicts not just a fifth dimension, but 10 dimensions (9 of which are spatial, one for time). So it's a serious theory. On the other hand, string theory is certainly not proven, and you can find many physicists who are sure it is wrong, so at this point, the existence of subatomic extra spatial dimensions is very speculative. -Lethe | Talk 08:58, 15 September 2005 (UTC)[reply]
I thought of string theory too, but I believe I once heard that it is inherently impossible to disprove and therefore doesn't even qualify as a proper scientific theory. DirkvdM 18:58, 15 September 2005 (UTC)[reply]
That is open to debate. -Lethe | Talk 00:45, 16 September 2005 (UTC)[reply]
As a visualisation for the fourth dimension I use a projection of a four-dimensional hypercube in the third dimension. The images in that article aren't too clear. You might try this. Or just image-Google 'hypercube', as I did. Alas, extending this to the fifth dimension is something my brain isn't up to :) . DirkvdM 07:12, 15 September 2005 (UTC)[reply]
I suggest explaining that the first three dimensions are based upon directions which are at right angles to each other.
Now explain that the 4th dimension is at right angles to these, but it is harder to visualize since it is more abstract.
Now, the 5th dimension is at right angles to the previous 4. The concept of right angles in abstract dimensions might be
hard to explain, but the idea is to show via the first three that components in one dimension cannot be used to describe
a component in the others.
Aha, I like the right angles idea. OK, now I'll lay my cards on the table. I'm writing a blog that takes a pedantic, wry look at advertising campaigns. Here in the UK there's currently a TV add for a cleaning tablet you place in your dishwasher. It claims to "clean in five dimensions". All this actually means (as far as I can gather from the advert) is that the tablet is coloured in five different ways, giving you a visual cue that it possibly has five different working ingredients that possibly take effect at five different points in the cleaning cycle.
I think I have enough now to make some kind of joke about the potential of a dishwasher tablet that really does function in the 5th dimension. Although if a mathematician wishes to lower his or herself to make one for me, I shall be listening ;o) Thanks all. --bodnotbod 21:05, 15 September 2005 (UTC)[reply]
Here's a nice little thought experiment. Think of a beachball in a black box (to remove all the other visual noise). Let's say this is three dimensional. Now, imagine the beachball bouncing up and down. We've just introduced another "dimension", time, to the beachball, so this all now is four dimensional. Now imagine two images of bouncing beachballs, one bouncing up and down, and another left to right. We've just introduced another "dimension", where something different is happening. So you can think of what you are seeing as being five-dimensional.
This thought experiment is however inaccurate in some senses, but it can help visualization. Regardless, the idea of dimension and all that jazz is closely related to concepts of vector spaces, so if you're further interested in one mathematical way of looking at it, check that out. I think this however is different from the physics/string theory way of looking at it, though. Dysprosia 03:45, 16 September 2005 (UTC)[reply]
it's not necessarily different. The dimension of a manifold is built out of the notion of dimension of a vector space, and the dimension of spacetime in string theory/Kaluza-Klein theory is just the dimension of a pseudo-Riemannian manifold. However, there is another way of looking at it. You can get rid of the anomaly by either asserting extra dimensions of spacetime, or just do 2d field theory with enough extra scalar fields. There is a conjecture with a preponderance of evidence to support it that every critical scalar field theory is isomorphic to a string theory on some Calabi-Yau. In that sense, even when the dimensions in string theory are not the same, they are the same. -Lethe | Talk 04:01, 16 September 2005 (UTC)[reply]
Well, I'm not exactly a physicist :) Dysprosia 04:32, 16 September 2005 (UTC)[reply]

Search engines[edit]

What is flag searching? Which search engine use flag searching? use boolean searching? phrase searching? stemming? truncation? proximity searhing?

What do diagnosic tools do?

Thanks very much

A search for a flag? ("What countries' flags are green, blue, and white?") Seriously, because a Google search for "flag searching" or "flag search" turns up very little in the context of search engines. Where did you hear of flag searching? What is the context of the question?
For the other parts, this page has a nice table summarizing the features of several search engines.
A diagnostic tool, in general, is something that checks to see whether something else is working correctly. Chuck 15:11, 16 September 2005 (UTC)[reply]
A flag is a property which can be applied to something, so perhaps flag searching is the searching for documents/files with a particular property. The term "diagnostic tool" is derived from the word diagnose, the definition of which is "[to] determine the root cause of a problem"; diagnostic tools are thus tools which try to pinpoint the cause of a problem. Rob Church Talk | Desk 23:51, 20 September 2005 (UTC)[reply]

What are the four main functions of a LAN?[edit]

What are the four main functions of a LAN? Describe each function

From the top of the page: Do your own homework - If you need help with a specific part or concept of your homework, feel free to ask, but please do not post entire homework questions and expect us to give you the answers.
Additionally, I should note that this question is almost certainly biased towards either your professor or your textbook. AFAIK, there is no universal standard as to the "four main functions" of a Local area network (as opposed to, say, the seven layers of the OSI network architecture). — Lomn | Talk / RfC 16:12, 15 September 2005 (UTC)[reply]

Please take Lomn's advice, above, to heart. However, I can tell you that the four main functions of a LAN are typically

  1. Quake III Arena,
  2. streaming MP3s,
  3. instant messaging, and
  4. sharing pornography.

Hope that helps. TenOfAllTrades(talk) 16:21, 15 September 2005 (UTC)[reply]

I had a similar list in mind, though I was planning on starting with StarCraft :) — Lomn | Talk / RfC 04:40, 16 September 2005 (UTC)[reply]

Climate Change[edit]

What is critical desalination point?It occures due to the melting of ice from the poles as a result of which a huge quantity of fresh water mixes with saline water.I know it upsets the ocean currents(the 'gulf stream' for example);but why does it happen?Does it cause any major disruption to global climate?

That's easy to look up (in other words, you could have done it yuorself - take the hint :) ). See gulf stream and the link near the bottom. It might trigger an ice-age, so, yeah, that would be a major change in the Earth's climate. I suppose the 'critical desalination point' is the point at which this happens (at least the term suggests that). DirkvdM 19:37, 15 September 2005 (UTC)[reply]

What is the purpose of the type of server?[edit]

What is the purpose of the type of server?

Huh? DirkvdM 20:37, 15 September 2005 (UTC)[reply]
Going out on a limb, different types of servers are optimized for various operations. A database server, for instance, prizes I/O speed and data reliability, so a high-end RAID array is a prime consideration. For a rendering server, amount and speed of RAM quickly becomes a major concern. For a purely computational server, little more than a CPU is required. On the whole, though, your question needs context if we're going to answer it properly. — Lomn | Talk / RfC 21:09, 15 September 2005 (UTC)[reply]

How do I create a Rich Text File[edit]

Open Wordpad. Select File/New, and wordpad should open up a dialog box with which you can select Rich Text. - Cobra Ky (talk, contribs) 19:03, 15 September 2005 (UTC)[reply]

I wanted to simply say 'save it as such', but you were just ahead of me (edit conflict). Since this even works with wordpad, you can imagine there will hardly be a text editor with which you can't save a text as rtf. So I wonder where the question comes from. DirkvdM 19:12, 15 September 2005 (UTC)[reply]

To be a little more accurate, all (or very close to all) word processors (programs that lets you apply formatting to some extent) can save files in RTF. Text editors, however, cannot, as they only operate on plain text (the RTF info would be lost). --Pidgeot (t) (c) (e) 00:22, 16 September 2005 (UTC)[reply]
Be aware though that on a Mac the application called "TextEdit" (which is despite the name a word processor) saves in RTF unless you very carefully tell it not to. Tonywalton  | Talk 21:52, 21 September 2005 (UTC)[reply]

Oxidation state of MnO4[edit]

I know the oxidation state/number of Permanganate is +7, but why is it +7? If Oxgygen here is -2, and Manganese range from +1 to +7, why is Permanganate's oxidation state also +7? Also, how would KMnO4 have a neutral oxidation state if K has +1? --Miborovsky 00:03, 16 September 2005 (UTC)[reply]

It sounds like some information got garbled somewhere. The permanganate ion actually has a net charge of -1; it's really MnO4-. In general, one doesn't usually refer to the oxidation state of entire ion or molecule—only to oxidation states (or formal charges) of their constituent atoms.
By convention, the formal charges of all the atoms involved must add up to the net charge of the ion or molecule in question. With very few exceptions, oxygen is almost always assigned a formal charge of -2. Since the total charge of the permanganate ion is -1, some algebra gives us the formal charge (oxidation state) of the manganese atom: +7.

Given the correct charge on the permanganate ion, the reaction

K+ + MnO4- → KMnO4

now balances nicely: a +1 charged ion and a -1 charged ion combine to form a product with no net charge. TenOfAllTrades(talk) 03:56, 16 September 2005 (UTC)[reply]

measoporous materials[edit]

Perhaps you're looking for information on mesoporous materials? Chuck 15:18, 16 September 2005 (UTC)[reply]

Atoms[edit]

How do you find the number of neutrons per atom by looking at the periodical chart?

It depends on how detailed the periodic table that you are looking at is. If it gives you an atomic mass for each element, then the number of neutrons is the difference between that weight (rounded to the nearest integer) and the atomic number of the element. For example, Oxygen has an atomic mass of just about 16, and an atomic number of 8, so it has 8 neutrons (and 8 protons).
It can get more complicated than this, because some elements have more than one isotope, which means they can have different numbers of neutrons. If you need a quick rule of thumb, then remember that H doesn't normally have any neutrons, and most of the lighter elements have the same number of neutrons as protons. The heavier elements have more neutrons than protons.-gadfium 04:00, 16 September 2005 (UTC)[reply]
Usually you can't—but you can make a guess at it. Atoms on a periodic table are arranged in order of increasing atomic number. The atomic number is equal to the number of protons in the nucleus of the atom. Hydrogen, with atomic number 1, has a single proton in its nucleus; Uranium (atomic number 92) has ninety-two.
Most periodic tables list an atomic mass for each element. Aha! One would expect that knowing the weight of a proton, and knowing the weight of a neutron (both conveniently close to one when measured in atomic mass units), a simple subtraction should give the number of neutrons, right? Oxygen has atomic mass 16.00 and atomic number 8; one expects to find eight neutrons in its nucleus to account for the balance of its mass. So this type of subtraction can give you a rough idea of the number of neutrons in some atoms.
Looking at chlorine, its atomic number is 17 and the specified atomic mass is 35.45—where did the extra half-neutron come from? Unfortunately many elements come in several variants–called isotopes–all having the same number of protons, but different numbers of neutrons. Specific isotopes are identified by mass number: a real sum of the number of protons and neutrons. When specifying a particular isotope, the mass number appears after the name when written out in full (chlorine-35) or as a leading superscript when using the chemical symbol: 35Cl.
Chlorine has two major natural isotopes: 35Cl and 37Cl. Both have 17 protons (the atomic number of a given element is always the same) while 35Cl has 18 neutrons and 37Cl has 20. In nature, 35Cl is about three times more common, so the given atomic mass of 35.45 on your periodic table represents a weighted average.
To find lists of all the isotopes for an element, consult a good physical chemistry or nuclear physics reference, or check the Wikipedia article for each element—the box on the right hand side lists the common isotopes down at the bottom. TenOfAllTrades(talk) 04:16, 16 September 2005 (UTC)[reply]
I suggest that TenOfAllTrades has the right idea - look up the Wikipedia article for the element, and check the most common isotope, which will be at the bottom of the fact table on the right (the one with the highest relative abundance %). After the mass number it will say "<element> is stable with X neutrons". X is the number you want. Example: Germanium - 35.94% - Ge is stable with 42 neutrons. splintax (talk) 03:19, 17 September 2005 (UTC)[reply]
Isn't it that one cannot state that an element has a specific number of neutrons, because it is defined by the number of protons? So there is no proper answer to the question, right? Normally the number of neutrons and electrons is equal to that, but if not, that doesn't change what atom/element it is. If the number of neutrons differs you get an isotope (different mass) and if the number of electrons differs you get an ion (different charge (and slightly different mass)). Right? My chemistry classes were a long time ago, though :) .DirkvdM 09:34, 21 September 2005 (UTC)[reply]

Phillip Chance, MD[edit]

The [First Result] from a [google search] of your title seems to have some detail of a Genetic Researcher Dr. Philip F. Chance. --Ballchef 16:13, 17 September 2005 (UTC)[reply]

pseudoscience[edit]

what is an example of pseudoscience and what phenomena did it claim to have?

Pseudoscience is an inherently controversial claim to levy, as no viewpoint advocate claims their perspective to be pseudoscientific; rather, they all deny it (and frequently accuse an opposing viewpoint as being pseudoscientific). However, we've got a great big List of alternative, speculative and disputed theories that, while not universally agreed upon, are at least frequently cited as such. — Lomn | Talk / RfC 04:35, 16 September 2005 (UTC)[reply]
See the article on pseudoscience for how it can be identified. There are certain things that are no longer really believed in by anyone and so are definitely pseudoscience - e.g. "Phrenology is seen today as a classic example of pseudoscience" would be the easiest to look at (e.g.) for a school project. But it's a bit boring. "Creation science" or "intelligent design" is much more interesting, but be warned it can get very complicated at times. See category:pseudoscience for more. Dunc| 13:07, 16 September 2005 (UTC)[reply]
You might like to take a look at Wikipedia:WikiProject Pseudoscience as well. Physchim62 19:24, 16 September 2005 (UTC)[reply]
To answer the question "what phenomena did it claim to have", this is a bit strangely worded - Pseudoscience is merely claiming to be able to determine something using an "unproved" method of science (as I understand it). To take Dunc's example - Phrenology - it claims to be able to determine aspects of the patient's personality by the shape and features of their head. splintax (talk) 03:22, 17 September 2005 (UTC)[reply]

It's also interesting to look at historical science like phrenology, and ask whether, if they were pursued honestly and with integrity at the time, but no longer meet the criteria for 'good science' today, the term 'pseudoscience' is really appropriate.

That's certainly true for theories like Phlogiston theory or Luminiferous aether; they were in their time seen as perfectly credible, "good" science (despite problems like requiring negative mass, in the case of phlogiston). It's also possible that something seen as pseudoscience today will turn out to be anything but "pdeudo" in the future – try explaining something like the Web or Wikipedia to someone from a short a time ago as, say, the 1940s. Tonywalton  | Talk 22:01, 21 September 2005 (UTC)[reply]

Architecture and Interior Design[edit]

What is T.I.? Judy

  • TI'S (Tenant Improvements) - TI'S is a term used to define the interior improvements of the project after the Building Envelope is complete. TI'S usually include finish floor coverings; ceilings; partitions; doors, frames, hardware; fire protection; HVAC consisting of branch distribution duct work, control boxes, and registers; electrical consisting of lighting, switches, power outlets, phone/data outlets, exit and energy lighting; window coverings; general conditions; and the general contractor’s fee. The cost of tenant improvements are generally born by the tenant and the costs of tenant improvements will vary with every building, and with tenant requirements. - from [12]
Cobra Ky (talk, contribs) 07:10, 16 September 2005 (UTC)[reply]

Hawking's formula[edit]

I just saw a documentary on BBC about the 'big mistake' of Hawking, the information paradox. In this a formula was presented that was said to be so simple it had to be true, namely . It was presented as a big breakthrough of the greatest physiscist of our time (so a big one I'd say), but little else was said about it, so I thought I'd look it up here, but to no avail. I've looked at thermodynamics, because that's where the S comes from, and of course at Steven Hawking. Should I look elsewhere?

the formula you quote is for the entropy of a black hole. That isn't really pure thermodynamics, so you won't find it in that article, but rather some mixture of quantum mechanics, gravity, and thermodynamics, mixed with some great intuition. Check out black hole entropy, Hawking radiation, and the holographic principle. You can find that formula in the first article (with a slightly different convention for constants). -Lethe | Talk 09:46, 16 September 2005 (UTC)[reply]

And isn't there (or else, shouldn't there be) an article with all physics formula grouped together (well, at least the most important ones)? Is there a set of formula that completely describes the universe as we know it today? The physics article didn't help in that respect. DirkvdM 08:34, 16 September 2005 (UTC)[reply]

I'm doubtful about both the possibility and the utility of such a page. Physics isn't indexed by formulas. The index of physics articles could probably be expanded and organized better though, and that would have helped you.
Regarding your second question, a set of equations that describe everything, well, at present, one might claim that the equations of motion of the Standard Model, along with the Einstein equation, describe the whole universe. But these equations are incompatible, and furthermore getting emergent properties from complex systems from these fundamental equations is not really possible, and requires additional assumptions. -Lethe | Talk 09:46, 16 September 2005 (UTC)[reply]

Quantum Computers[edit]

How does the quantum computer determine the right calculation after using Everett's many world interpretation? The quantum computer used the many world interpretation to do series of calculation in different universes, then how does it determine the right answer?

You need to make a distinction between the quantum computer that was invented in the IBM laboratory and is in actual usage, and whatever Science Fiction story you are referring to whose author probably not know that IBM already has a quantum computer. AlMac|(talk) 21:16, 17 September 2005 (UTC)[reply]

azo compund[edit]

Why Azo compunds are colored? (what is the mechanism behind their dyeing property?)

Please refer to our article, azo compound. Briefly, the azo dyes have large delocalized electron clouds due to a system of conjugated chemical bonds. These large conjugated systems tend to be very efficient chromophores: chemical centers that absorb light. TenOfAllTrades(talk) 13:50, 16 September 2005 (UTC)[reply]

Atomic bomb image[edit]

I am trying to find an image of the city on a bullseye illustration that was used in the 1950s to show radius of destruction of the atomic bomb. Thank you Mary S.

Mary, there have been hundreds of similar images drawn over the years, for instance here, and in the Congressional report linked to from it. If you are looking for one specific version it would be helpful if you could provide some more information - was it a specific city? Where precisely was it used? --Robert Merkel 23:53, 17 September 2005 (UTC)[reply]
It depends what city you mean, Mary. Such images were widely created during the Cold War as part of Civil Defense evaluations (the goal was to make it so that major industry and services would not be knocked out by an atomic bomb attack). There were also ones created of Hiroshima (such as this one on Commons). All of these were modeled after maps created of the Strategic Bombing Survey, which modelled how well the firebombing of Dresden and firebombing of Tokyo worked. There is an interesting article on these maps and their impact on urban development by Peter Galison in the book: Antoine Picon and Alessandra Ponte, ed. Architecture and the sciences (New York : Princeton Architectural Press, 2003). Hope that is helpful. --Fastfission 02:37, 22 September 2005 (UTC)[reply]

Nap Jerk / Sleep Spasms[edit]

What causes the full body spasms that occur either during a nap or right before a person falls asleep?

Here's a simple explanation: kidshealth. You could also see our article Nocturnal myoclonus for a more serious version.-gadfium 19:17, 16 September 2005 (UTC)[reply]
When you're just falling asleep–or sleeping uncomfortably–the phenomenon is a hypnic jerk. It's perfectly normal, though the explanation for it isn't entirely clear. Note that nocturnal myoclonus refers to a sleep disorder that involves involuntary muscle movements after you've fallen asleep. Both phenomena fall under the broad heading of myoclonus: involuntary muscle movements. TenOfAllTrades(talk) 19:28, 16 September 2005 (UTC)[reply]

what happened to Radio Shack[edit]

See RadioShack. I'm not in North America, but it looks like they are alive and expanding. If you are asking about the computers they used to make, see TRS-80.-gadfium 19:23, 16 September 2005 (UTC)[reply]

In North America, they've been bought out by Circuit City and are now called The Source by Circuit City, which is certainly not promising to the dozen electronics hobbyists that hadn't gone elsewhere for their parts. — mendel 20:14, 16 September 2005 (UTC)[reply]
That seems to be the case only in Canada, where Circuit City tried to run stores under the name "RadioShack", but lost a legal battle with the U.S. based corporation that operates Radio Shack. LarryMac 20:33, 16 September 2005 (UTC)[reply]
This would seem to be news to the RadioShack Corporation, which seems to think they're still in existence -- http://www.radioshackcorporation.com/about/index.html. User:Zoe|(talk) 20:37, 16 September 2005 (UTC)[reply]
According to this page, Radio Shack spun off its international operations back in 1986. A Canadian-based company called InterTAN (now a subsidiary of Circuit City; I don't know if they were then) licensed the rights to the name in Canada and operated the stores under the name Radio Shack or RadioShack until 2005. At this point the real RadioShack company won their lawsuit seeking to terminate the license, and the stores were renamed; RadioShack now plans to open its own stores in Canada in competitition with InterTAN's. --Anonymous, 07:15 UTC, September 17, 2005

RE: Wind up watches[edit]

How do wind up watches work?

Best regards, Felipe Barreda

All wind-up clocks operate by storing potential energy in a spring, using the internal mechanisms of the clock to release that energy at a known rate. You can find a good bit more about this in our article on clocks as well as many of the related articles it references. — Lomn | Talk / RfC 19:36, 16 September 2005 (UTC)[reply]

Planting in Peachtree City, GA[edit]

What is the average date of the last frost in the spring and the first frost in the winter in Peachtree City., Georgia?

Thank You, John W. Merrick DVM, drjohnwm@netzero.net

This page has that information for five large cities in GA, perhaps one of them is close enough? LarryMac 20:12, 16 September 2005 (UTC)[reply]

Can I edit an Adobe .pdf doc without spending money?[edit]

I've got OpenOffice and happily used that to save from OO's native file format to .pdf. However, I now want to edit a .pdf that I didn't create. When I try to open it in OO I get a bewildering array of "filters" to choose form, none of which seems the obvious choice. Any tips? Other siftware I can use? I appreciate, of course, that Adobe may have tried to tie up the format - but they're not always successful, are they? ;o) --bodnotbod 20:58, 16 September 2005 (UTC)[reply]

You can use ImageMagick to convert it into PostScript, which you could then attempt to edit or convert to a different format. --Pidgeot (t) (c) (e) 21:03, 16 September 2005 (UTC)[reply]
  • Thanks. And foolishly I hadn't bothered to look at our Portable Document Format before asking here. A lack of initiative I'm always berating people for when I look through the Reference Desk. I can see Adobe provides a tool for transferring .pdf to HTML and normal text too, though I'd rather try and retain all the formatting. I'll see how I get on with your suggestion. Astonishingly quick reply, thank you. --bodnotbod 21:14, 16 September 2005 (UTC)[reply]
  • I'm actually still having problems finding the part of the documentation that tells me how to convert a file. - Mgm|(talk) 23:43, 16 September 2005 (UTC)[reply]
    • In general, it should be enough to just type convert file1 file2 where file1 is the input file and file2 is the output file. Apart from that, you might want to look at [13], which has quite a few examples and such. --Pidgeot (t) (c) (e) 23:48, 16 September 2005 (UTC)[reply]
I little side-tracking. Are you sure you want to use pdf? It's specifically meant to make printouts. It's meant for paper, not for reading on a screen. So the logical procedure is to use some other format and only convert to pdf at the moment you intend to print it out (and even then I doubt the usefulness of pdf, bat that may be for lack of experience). But keep the original in a 'true digital format'. I (and many others) hate it when a text on a website is in pdf format in stead of html (for one it's terribly slow). To take this a bit further, I suggest doing all your editing in html because that is the only format that is truly universal. One problem with that, though, is that I only know of one comfortable wysiwyg-like editor, Mozilla Composer. That's free, though (one of your concerns). DirkvdM 09:40, 17 September 2005 (UTC)[reply]
Well, disgressing further (and since Bodnotbod seems on track to solving the question, it seems ok), but I disagree that pdfs are for priting only. pdfs are pretty close to the only widely-read, easily understood, and perfectly managable format out there. If I want to, say, host an on-line resume, I'd hate to do it in HTML — the formatting would be different for everybody that loked at it: I'd have much less control over the formatting, the user's browser would do it for them. A pdf, however, guarantees that the document looks the same whoever looks at it. This is the same for all sorts of things. I spend all my time reading scientific articles — I'd hate it if I read them in HTML, with diagrams and tables places any which way. If pdfs download slowly... well, broadband is awfully cheap nowadays... ;-) — Asbestos | Talk (RFC) 22:57, 17 September 2005 (UTC)[reply]
Yes, I like seeing PDFs onscreen too, though I'd not set up my entire website as one ;o) --bodnotbod 03:32, 19 September 2005 (UTC)[reply]
I've got broadband (adsl), which is why I'm used to instant results to clicking a link. So if it takes a few seconds (sometimes even tens of seconds) that is relatively irritating. I don't think this has to do with the speed of the connection but maybe with the rendering. I'm not sure about this, but if it is true more and more people will start to get irritated by pdf's as broadband gets more widely used. For this reason I don't even bother to look at pdf-hits when I've Googled something unless I'm sure that's where the answer is. And I suppose I'm not the only one. So a lot of people will miss your work for this simple reason (assuming you publish it on the net).
Another disadvantage of pdf's is that it is almost impossible to copy/paste text from them (unless you paste into another pdf, I suppose). This is an example of irritating proprietary behaviour; "if you use our product you can't use any other products anymore". This is anti-standardisation to hopefully attain a monopoly. As a 'believer' in the free market I hate monopolies. And I'm a strong supporter of standardisation.
It may be true that most people will now have pdf-rendering ability (although it is still optional). But with html that is true by definition on the Internet. If someone goes on the Internet they do that with a browser, which is all that is needed to render html.
It's true that thml isn't truly wysiwyg, nut I don't see that as a disadvantage. To the reciever it can actually be an advantage because they can get the presentation they prefer. Such as links underlined or not. Hell, you can't even make links in a pdf, can you? DirkvdM 09:57, 21 September 2005 (UTC)[reply]
You can indeed make links in a PDF, just as you can have PDF forms to fill in. Although you probably need to use Adobe's software to get some or all those features in your PDF file, I haven't tested much software in that field... --Pidgeot (t) (c) (e) 10:55, 21 September 2005 (UTC)[reply]

What is reffered to as the brains of the computer?[edit]

The CPU. --Pidgeot (t) (c) (e) 22:48, 16 September 2005 (UTC)[reply]

That's an interesting question, I would tend to semi-disagree with Pidgeot and say that it could be a number of things. Generally it would refer to what I call the "tower" in a desktop computer (although it has many names, most of them incorrect) - by this I mean the box with all the wires coming out of it and the power switch on it, inside which are all the sub-components of the computer.
As far as sub-components go, you could say that both the CPU (as per Pidgeot), the RAM, the motherboard and the hard disk drive perform functions that are similar to the brain. The CPU performs calculation tasks, and is essentially equivalent to what your brain does to analyse a situation and work out what to do. The RAM is equivalent to your short-term memory - it is impermanent, so when the computer shuts down, all information in RAM is lost. The motherboard is also similar to the brain in that it coordinates all other parts of the computer and processes input (eg from keyboard and mouse). Finally, the hard disk drive is equivalent to your memory, as it stores data "permanently" - though it is still prone to failures (amnesia) ;-) splintax (talk) 03:13, 17 September 2005 (UTC)[reply]
What defines a brain? The most likely answer is its information-processing qualities. But that is done by neurons and those extend to all parts of the body. Something similar can be found in a computer. The CPU (the name says it; Central Processing Unit) is probably the best equivalent of a brain. But in the olden days there was sometimes a mathematical co-processor and these days you have all sorts of equipment that takes on specialised aspects, most notably the graphics card. This could be likened to the neurons in the back of the eye that do so much processing that they could also be seen as part of the brain (and coincidentally (?) also has to do with processing of visual data). So I basically say the same as Splintax, just with a different wording. But I wouldn't call the 'tower' the brain. One might say the motherboard is a frame that connects the different parts of the 'computer brain'. The computer/brain analogy is popular and interresting, but (as all analogies) flawed. DirkvdM 09:27, 17 September 2005 (UTC)[reply]
It depends what you mean by "brains". If you mean the equivalent of what you get when you take the brain out of a person's head then it's the CPU, motherboard and disk drives, as mentioned by splintax. If you're thinking in terms of "braininess" (eg, "he's got no brains but she's very brainy"), then you'd have to include the software as well. Ferdinangus 00:56, 23 September 2005 (UTC)[reply]

Science- About Shadows[edit]

Hello, I was wondering if you can help me out on these questions that I am asking not the my teacher asking me. Thanks a bunch!=)

1.how is the umbra and the prnumbra related to the shadows?


2.How does a translucent shadow and transparent shadow different from an opaque shadow?


3.How soes a shadow get bigger and smaller in the relation to the angle and intensity of the light?


4.Why do liquid shadows appear ethereal?


5. Why do shadows appear colored?

You may want to start by reading Shadows. From a quick glance at the article, it appears to answer several of your questions. splintax (talk) 03:08, 17 September 2005 (UTC)[reply]
You might also consider asking your science teacher. :D TenOfAllTrades(talk) 21:34, 17 September 2005 (UTC)[reply]

Fossils[edit]

What is a fossil? What is the oldest fossil? who is a famous fossil finder? year 3 Northam Primary

Hello Northam Primary! Did you read our article on fossils, and the article linked from it on paleontology (the study of fossils)? These two articles contain good answers to all your questions. If you are having trouble understanding the articles, ask your teacher to help explain them. --Robert Merkel 05:50, 17 September 2005 (UTC)[reply]

Vicosity of petrol and desiel[edit]

what is viscosity of petrol and disel

I assume that you want viscosity in Pa s (pascal-seconds). While I couldn't find any information on the Wikipedia, a quick Google search turns up this as the second result. I don't understand the results on that page but hopefully you will.
This page should also help you convert the viscosities given in that document to the SI units (Pa s). splintax (talk) 14:16, 17 September 2005 (UTC)[reply]
Oil products are usually characterized by their kinematic viscosity measured in centipoise (cP), as this is the unit used by the ASTM standards. 1 cP = 1 mPa s. Petrol (gasoline) has a viscosity of around 0.6 cP at room temperature, diesel (kerosine) is about 2.5 cP. Both these figures vary (greatly) with temperature and (slightly) with the sample of fuel. Physchim62 18:41, 17 September 2005 (UTC)[reply]

Unscrewing old screws[edit]

I've got a couple of screws that are old and jammed really tightly in their sockets. Using a good cross-head screwdriver all I succeeded in doing was dent the heads — the metal seems really soft. I stopped before I destroyed the crosses in the heads entirely, though. What should I do to get them out. Would a standard lock lubricant work, or do I need something more powerful? — Asbestos | Talk (RFC) 10:33, 17 September 2005 (UTC)[reply]

Once the slot is damaged you will probably have to sacrifice the screws but it should be possible to get them out. Put a couple of drops of penetrating oil on and leave for an hour. Tap the top of the screwdriver with a hammer to loosedn the screw. Wipe all the oil away from the head and try again. If the head is proud of the material you might be able to grip it with pliers to turn it. Last resort is to drill out the screw. Choose a drill bit just over half the width of the screw head and drill it out, then use thin nose pliers to extract the rest of it. Shantavira 12:49, 17 September 2005 (UTC)[reply]
There is a tool called "Easy-Out" (or EZ Out) that works by drilling a hole in the center of the screw and inserting a left turning tapered screw. The Easy-Out screw will bind into the hole and then force the jammed screw to turn. Some illustrations are here. --hydnjo talk 16:33, 17 September 2005 (UTC)[reply]

Thanks all. I ended up solving it even before getting back to my question. I used a spray-on lubricant — WB-40 or whatever it's called — let it sit for a few minutes, tapped in my screwdriver using a hammer, and then unscrewed it successfully. Thanks! — Asbestos | Talk (RFC) 22:50, 17 September 2005 (UTC)[reply]

WD-40. Rob Church Talk | Desk 23:58, 20 September 2005 (UTC)[reply]
In future, you could try Loctite (iirc), or this tip I heard off Tool Time (yes, from Home Improvement) that has oddly stuck with me - take a hacksaw and cut a slot into the head and use a regular slotted driver. Dysprosia 07:29, 18 September 2005 (UTC)[reply]
That's why I prefer to use slotted screws. They're much more solid. Both the screw and the screwdriver last longer (less frequent slipping). And because of the simple function it is by definition standardised, so to say, which Phillips screws and the like are not. You need a whole bunch of screwdrivers to be prepared for all screws. With a slotted screw you don't even need a screwdriver; a knife or coin or something similar will usually do the job too. And, conversely, your trick works only with this design (which is so simple you can hardly call it a design). DirkvdM 10:10, 21 September 2005 (UTC)[reply]

Role of a clinical project leader[edit]

What is the role of a clinical project leader in phase 3 clinical trials for a new beta blocking agent?

Hmmm. That sounds like a homework question for a medical or biological science student, in which case all the information you need should be in your textbook. In general, if you do a search for "clinical project leader" and role on Google, you get a whole bunch of useful links on the topic of what a clinical project leader does, in general. Our link on clinical trial explains what the different phases of clinical trials are - phase 3 trials are the last stage of trials before a drug is approved for general use and attempt to be comprehensive analyses of efficacy and safety. --Robert Merkel 23:43, 17 September 2005 (UTC)[reply]

How to calibrate the value of an altimeter?[edit]

Hi, I am a undergraduate student and currently carried out a project related to altimeter.The purpose of this project is to reduce the error of the altitude value due to the changing of the atmosphere pressure within a day. Since we know the pressure will be varied in a day regardless to the change of the altitude.This has cause the value of altitude given by altimeter become inaccuratue since altitude is calculated by pressure. The result is that,the altimeter can not give the actual value of the altitude at any location for a long duration.I would like to know is it possible to reduce or to calibrate the value of the altitude so that it can become more accurate?

I just edited your wikicode, because you were messing up the page's layout. I didn't edit anything you said. For future references, please avoid using a space character at the start of a line on Wikipedia, as it activates "monospaced" text, which can make a page very difficult to read (as was the case here). As far as your question goes.. Sorry, can't help you.. :-( splintax (talk) 14:08, 17 September 2005 (UTC)[reply]

Sure, with a barometer. The altitude at your location does not change, just the air pressure. All you need are some simultaneous barometer and altimeter readings at various air pressures at a constant altitude. Then you can calculate the magnitude of air pressure effect and compensate when you try the altimeter and barometer together at different altitudes. alteripse 15:00, 17 September 2005 (UTC)[reply]

The Wikipedia article Altimeter describes this effect and how it is dealt with. --hydnjo talk 16:59, 17 September 2005 (UTC)[reply]

Asexual Animals[edit]

Could you please list at least 3-5 asexual animals please?

Tasha

Check out our article, Asexual reproduction. — Asbestos | Talk (RFC) 14:53, 17 September 2005 (UTC)[reply]
That article, and the links therein, should get you started on your homework. TenOfAllTrades(talk) 15:05, 17 September 2005 (UTC)[reply]

Processor management[edit]

Do Novell NetWare have a processor management? Where can i find the resource?

I am not knowledgeable on how Novell does things differently from other major Operating Systems, although I am aware that just about anything that can be done with Microsoft Windows, Apple Macintosh, IBM AS/400, UNIX, etc. can also be done with Novell ... have you checked the Novell website? AlMac|(talk) 21:22, 17 September 2005 (UTC)[reply]

domain vs kingdom[edit]

I am currently taking a Biology course & on our upcoming test we are going to have to answer "Differences between domain & kingdom." I am having a hard time with this as kingdom is a division of domain, right?

Does your course have text book, and have you opened it? AlMac|(talk) 21:23, 17 September 2005 (UTC)[reply]
Wikipedia's search function can be accessed using the box on the left side of the page. I would suggest searching for domain and going from there. TenOfAllTrades(talk) 21:29, 17 September 2005 (UTC)[reply]

Yes I have opened the book and it explains what a domain system is and what the kingdom system is but the way I am understanding it is kingdom is a division of the domain system so I guess I don't understand how to discuss the differences. Thanks for your "answer"

  • According to Kingdom (biology), in bilogy, "kingdom" refers to like all animals, or all plants, or all minerals (I am simplifying) ... go to that article for more specifics. In other words it has to do with how the different "things" in science are organized. AlMac|(talk) 01:20, 18 September 2005 (UTC)[reply]
    • Here is a "history" [14] of how Taxonomy, a methodology for organizing what we see in nature into different sub-topics, had a human evolution as science gained more understanding of the differences between "things" in nature at a microscopic and atomic level. AlMac|(talk) 01:44, 18 September 2005 (UTC)[reply]
  • According to this source [15], in biology, "domain" applies to a method of problem solving, or scenario analysis. In other words it has to do with how we figure things out in science. AlMac|(talk) 01:26, 18 September 2005 (UTC)[reply]
  • So by the above definitions, kingdom is not a part of domain, and domain is not a part of kingdom, rather domain is a way of exploring kingdom.
    • It would be like saying what is the relationship between an automobile and a highway, is an automobile part of a highway, or is a highway part of an automobile, and the answer is neither, an automobile is a way of traveling on a highway. An auto can also be used for other things, and there are other ways of traveling on a highway without using an automobile. That is an analogy to domain and kingdom, as defined at the above links. AlMac|(talk) 01:26, 18 September 2005 (UTC)[reply]
  • Other links via [16] indicate that sometimes domain is used at the microscopic or molecular level. AlMac|(talk) 01:31, 18 September 2005 (UTC)[reply]
  • Thus, if we study the biology of some animal in the "animal kingdom", we might find some tiny microscopic particles in the brain of some creature, that can be said to be in some kind of domain. AlMac|(talk) 01:31, 18 September 2005 (UTC)[reply]
    • this link [17] gives yet another perspective.
      • In the 18th century, biology was split into the animal kingdom and the plant kingdom, but as our scientific understanding grew, it became neccessary to organize that know-how into a larger number of different kingdoms. About 35 years ago, we had got to five kingdoms, but in the last few years, study at the bacterial level has led to a system of organization using 3 domains and 6 kingdoms.
        • The five kingdoms were
          • monerans,
          • protists,
          • fungi,
          • plants,
          • animals.
    • So this is yet another way of answering the question.
  • The important thing is to correlate what your teacher says and what your textbook says because to pass the class you need to be able to communicate that you learned what they are trying to teach, which could be quite different from what other reference sources are saying.
    • The publish DATE of your text book becomes critical, because it might predate the evolution of biology science. However, to pass the course, it is more important for you to learn what the teacher wants, than what is the latest evolution of science. AlMac|(talk) 01:39, 18 September 2005 (UTC)[reply]
That's a very oldfashioned stance that I disagree with very much. It's easier on both the student and the teacher (a nice simple worldview; "what this book says is the truth") but but also a tunnel-view. The truth (which should be taught at schools, right?) is never that simple. So if you give an answer that is sufficiently widely accepted then the teacher should accept it too. Even give you a higher grade because you've gone 'beyond the line of duty' for a student by not just copying form the textbook but actually trying to find an answer yourself. It may not surprise you to know that I had some quarrels with my teachers in my time :) . But, luckily, in the Netherlands that oldfashioned attitude has largely disappeared with a system that is more focused on self-study than listening to a 'talking head' (which I always hated - in one ear, out the other). DirkvdM 10:26, 21 September 2005 (UTC)[reply]

website design[edit]

And your question is...? --Pidgeot (t) (c) (e) 20:52, 17 September 2005 (UTC)[reply]

The Human Head[edit]

How large is the largest Human head or cranium ever found, and who did it belong to? -Blaze

I did some searching. Most of the results I got turned out to be jokes. So, this information is hard to find. You might try the Guiness Book of Records. — Nowhither 22:51, 20 September 2005 (UTC)[reply]

The Guiness Book of Records has no such entry for Human Heads. I've found this information very hard to find myself. I'm sure it is out there somewhere though. - Blaze

lobsters[edit]

Hello, please could you tell me why a female lobster is called a hen and a male a cock - strange question but am most curious

many thanks Rupert

Rupert: No particular reason. Someone just decided to call them that, and it caught on. The names, by the way, come from bird usage; any female bird is a "hen", and any male bird is a "cock". Similar phenomena are found with rabbits, which are named using terminology derived from that used for deer (doe/buck). Also moose, despite being deer, are named using terminology derived from that used for cattle (cow/bull). — Nowhither 00:11, 19 September 2005 (UTC)[reply]


Embedded systems[edit]

What does an embedded system reside on?

  • It resides on many places.
    • Consider a thermostat to control the temperature of a building, or to trigger whether the air conditioner is to kick on or not because the temperature has got to be too hot where the theromostat is located.
      • Suppose a thermostat is connected to a computer system, so that the computer will do something automatically, such as tracking the usage of electricity.
    • Consider a vending machine. There are electronics in there that tell if the vending machine is running low and needs to be replenished.
      • The vending machine can be connected to the Internet, and the vending machine company can connect to see which machines need more stuff put in (and the coins for the old stuff taken out). Hackers can spoof the info there, so the vending machine company not know there has been a break in.
  • Basically, an embedded system is any electronics that was designed for some simple function, like controlling a pipe line, or heating cooling a building, operating an elevator. The act of connecting that simple system to a computer system, is said to be embedding it into the network. Since these systems, before embedding, were designed pretty much as stand-a-lone, they did not come with much in the way of security thinking. But computer networks are hotbeds of malware, hackers, insider crime and so forth (see computer insecurity for more examples), so the act of embedding them, has created a computer security nightmare. AlMac|(talk) 01:54, 18 September 2005 (UTC)[reply]
    • Not true. See embedded system for the real meaning of "embedded" in this context. Gdr 18:22, 18 September 2005 (UTC)[reply]

what is a non-sustainable development[edit]

Non-sustainable development is development which consumes more resources than can be regenerated. Think of it this way; imagine there's a town next to a forest. Whenever the people in the town wants to build a new house, they go into the forest, chop down a few trees, and use that wood to build a house. However, if the citizens of the town are chopping down trees faster than the trees can grow, eventually they will chop down all the trees, and therefore not be able to build any more houses. This is non-sustainable development. If, on the other hand, the citizens limit the number of trees they cut down so that the forest can regenerate itself and/or plant new trees to help the forest grow, they will never completely run out of trees, so they are practicing sustainable development. Check out our article on sustainable development for more. Garrett Albright 03:38, 18 September 2005 (UTC)[reply]

How to make the value of sinus function become a straight line?[edit]

I recently doing an experiment at school and the result that I got was like a sinus function.But the problem is that the result should be in a constant regardless to the change of the time.Can you tell me is there any mathematical way to make or calibrate the value of sinus function to become a linear function?

Yes. y=0 sin(x) is a straight line through the origin. y=0 sin(x)+b is a straight line through b. Both of these can be considered degenerate sine functions.

I think you mean a sine function, not a sinus function. Graphed on a normal cartesian coordinate system, a sine function can never be a straight line, AFAIK. Garrett Albright 03:38, 18 September 2005 (UTC)[reply]
Don't be such an anglophile. Many languages call the sine function sinus. See, eg da:Sinus (matematik). -Lethe | Talk 09:10, 18 September 2005 (UTC)[reply]
That's well and good, but what language are we using right now again? Remind me… :P Garrett Albright 10:06, 18 September 2005 (UTC)[reply]
Well, sine does say 'Our modern word sine comes, via sinus ("bay" or "fold") in Latin'. Regardless, it's a bit hard to say anything either way because we don't know the problem. However, sin x is approximately x for small x, which may or may not be helpful. Dysprosia 07:27, 18 September 2005 (UTC)[reply]
Where x is measured in radians Theresa Knott (a tenth stroke) 07:32, 18 September 2005 (UTC)[reply]
And sinus says, "Sinus is a Latin name for the trigonometric function sine, a concept of mathematics." And, by the way, Theresa, in mathematical usage, x is generally a number, not an angle. Thus it has no units. And sin x is equal to the sine of an angle with measure x radians. — Nowhither 00:05, 19 September 2005 (UTC)[reply]

If you give details of the experiment you did we may be able to help. Also, provided you did the experiment correctly, the results you got are the "truth". Rather than trying to fudge the answer to make it a straight line you may be better off doing a little more experimenting. For example look at the frequency o thr sine wave. If it's close to 50 herts (in the UK, I don't know about other countries) then I'd suspect mains hum. Theresa Knott (a tenth stroke) 07:41, 18 September 2005 (UTC)[reply]

50 Hertz is standard accross Europe, the US uses 25 Hz, Southern Japan 60 Hz. The frequency is a constant with respect to time: maybe this is what you are looking for? Physchim62 12:06, 18 September 2005 (UTC)[reply]
The US is on 60Hz, FYI. — Lomn | Talk / RfC 16:38, 18 September 2005 (UTC)[reply]
Additionally, a sine function may render as a straight line in non-cartesian systems. Perhaps a polar plot or the frequency domain? I haven't really played with this stuff in a while, so I don't remember. — Lomn | Talk / RfC 16:38, 18 September 2005 (UTC)[reply]
In polar coordinates the sine and cosine functions graph as circles. However, their reciprocals, the cosecant and secant functions, graph as straight lines. I'm not sure what it would mean to graph the sine function "in the frequency domain". — Nowhither 17:21, 20 September 2005 (UTC)[reply]
See fourier transform. Lunkwill 09:52, 22 September 2005 (UTC)[reply]
Well ... yes and no. Yes, that is what the "frequency domain" is all about, but no, that page doesn't make sense of the phrase "graph of the sine function in the frequency domain". — Nowhither 00:56, 23 September 2005 (UTC)[reply]

wireless network[edit]

wireless router

Check out #Wireless router?? above. Garrett Albright 03:43, 18 September 2005 (UTC)[reply]

Biology[edit]

what kind of organic compound involved in fat formation contains carbonxyl groups?

Fat molecules are made from glycerol and 3 fatty acid molecules. Fatty acids are basically carboxylic acids with long carbon chains. It is the fatty acids that contain the carboxyl group.--82.44.216.80 10:24, 18 September 2005 (UTC)[reply]

Prokaryote Organism[edit]

Hello Sir, Could you please tell me how the prokaryotic organism, lets say E. Coli develop? I do not enough information on this organism developmant. Thanks.

I think when such a cell is large enough and the environment is favorable, the DNA doubles and it just divides. We don't usually use the word development in this context, but maybe I am misunderstanding your question. alteripse 12:52, 18 September 2005 (UTC)[reply]

application of computers for design.[edit]

p/s assist me find a page that dealth on the above subject extensively.its applicaion in geometri modelling, engineerig anlysis, design review and evaluation and automated drafting should be inclusive. thanks.

You might do a search for Auto CAD which is the most popular system in the USA for on-line engineering design (mainly because it is the cheapest). AlMac|(talk) 21:01, 18 September 2005 (UTC)[reply]
And you might as well start with AutoCAD, the proper name for the article. ;) --Pidgeot (t) (c) (e) 21:18, 18 September 2005 (UTC)[reply]

Radiolarians[edit]

Do radiolarians use silicic acid in constructing their exoskeletons, as do diatoms? ᓛᖁ♀ 08:48, 18 September 2005 (UTC)[reply]

food equivalency name for graham crackers in australia[edit]

HI graham crackers? Are they equivalent to our SAO or more like our GRANITA? Also what is cool whip?

I dunno about graham crackers, but Cool Whip at least is a sugary marshmallow-like spread somewhat like whipped cream. I believe it's mainly a foamed emulsion of vegetable oil and sugar water. ᓛᖁ♀ 16:13, 18 September 2005 (UTC)[reply]
Graham crackers are moderately sweet, and made from whole wheat flour ("Graham flour", from a health-food fad about 100 years ago). They are about 4mm thick, light brown in color, rectangular; they tend to be made in semi-perforated sheets, so you break off a certain amount at a time to eat it. They fall somewhere between what we in the U.S. would call a "cracker" and a "cookie" or what I believe the Brits would call a "biscuit" and a "sweet biscuit". I have no idea of the Australian terminology. Major American comfort food, especially if dissolved in milk. They are also often crumbled to make a pie crust. -- Jmabel | Talk 03:46, 19 September 2005 (UTC)[reply]
This is probably irrelevant to the original question, but according to our article, Graham flour is actually somewhat different from typical whole wheat flour. To make Graham flour, the bran, germ, and endosperm are ground separately, the endosperm ground finely and the other parts ground coarsely, and then the parts mixed back together. Also, I'm sure you didn't mean to imply that the consumption of whole grains was limited to a fad. — Pekinensis 13:31, 19 September 2005 (UTC)[reply]

thermal power plant[edit]

flow diagram of thermal power plant

Could you be looking for this? (See what links there.) You may wish to clarify your question so that we can help you better. splintax (talk) 16:17, 18 September 2005 (UTC)[reply]

Who first proposed / discovered that the sun is a star?[edit]

That's it - didn't find the answer in sun. Just curious

--joseI 16:07, 18 September 2005 (UTC)[reply]

  • It was probably more a question of the realisation that the stars are like the Sun. The view of the Sun didn't have to change. Notinasnaid 16:42, 18 September 2005 (UTC)[reply]
This idea was poorly accepted in the 16th century, when Dominican order monk Giordano Bruno put forth the claim that "Innumerable suns exist; innumerable earths revolve around these suns.... Living beings inhabit these worlds." Because of this and various other ideas, he was imprisoned, tortured, and finally burned at the stake on February 17, 1600 by the Roman Inquisition. [18] [19] ᓛᖁ♀ 17:01, 18 September 2005 (UTC)[reply]
Well, it wasn't really his cosmological speculations that got him in trouble. More likely, it was association with Bruno that got cosmological speculation such a bad reputation with the Church! Bruno was a believer in the Hermetic doctrines, and quite possibly wished to revive the Roman gods. He was definitely not a Christian believer, and while his books attempted to veil in layers of mystery and metaphor exactly what he did believe, the Inquisition was not amused. He recanted, then he unrecanted; I blieve all of this is in our article on him. Note, also, that Bruno didn't so much say that the Sun was a star as that the stars were suns. -- Jmabel | Talk 03:53, 19 September 2005 (UTC)[reply]
Which is an interesting distinction. In saying the sun is a star, you devalue it (it goes from the unique to the common), but in saying that stars are suns, you somewhat overvalue them (not all stars are actually suns, in the notion that they have planets near them, much less have the other nice properties we often associate with our nice medium sized sun). --Fastfission 01:31, 20 September 2005 (UTC)[reply]
Keep in mind, though, that by the Church's view, the fixed, unchanging stars were the nearest to the perfection of Heaven that could be seen. To this view, the statement that stars are simply ordinary matter — comparable to the matter of Earth, closest to Hellgreatly devalues the stars and may be seen to threaten the purity of Heaven. ᓛᖁ♀ 02:54, 20 September 2005 (UTC)[reply]

vapours from ice[edit]

why does vapours come out from the surface of ice?

What you see are condensations of water from the air due to the cooling effect of the ice. alteripse 16:31, 18 September 2005 (UTC)[reply]

In life science, what is a base?[edit]

See base. I think you want the ninth(?) definition (In chemistry...) (number corrected Superm401 | Talk 20:55, 23 September 2005 (UTC))TenOfAllTrades(talk) 18:39, 18 September 2005 (UTC)[reply]
In life science, which is very cosy with chemistry, you might also be interested in the tenth (number corrected Superm401 | Talk 20:55, 23 September 2005 (UTC)) definition of base (As in DNA Base Pairs).--inks 23:21, 18 September 2005 (UTC)[reply]

Ultraviolet light[edit]

Does ultrviolet light have a heat index. Could it increase the heat of a pond, stream, river, or ocean? --anon

Any energy in the electromagnetic spectrum can be converted into heat, so yes. — Lomn | Talk / RfC 02:09, 19 September 2005 (UTC)[reply]

in mechanical drawing what is the order of precedence?[edit]

More fun with PC 800 RAM![edit]

Hey, true believers - so, I bought some used RAM for an old Gateway 700S desktop I have. Figure I can get a little life out of the new system this way. Only problem is, I seem to have purchased PC 800-[b]45[/b] RAM. My computer only seems interested in taking PC 800-[b]40[/b] RDRAM. Is there any way to force my computer to accept the new RAM in a way that does not compromise its functionality? Thanks. --Brasswatchman 02:36, 19 September 2005 (UTC)[reply]


Answer: "If the RAM does not fit in the alloted slot, you will not be able to use it."

It does fit in the alloted slots. The problem is the Gateway 700s doesn't accept it. Even the technician at Gateway couldn't figure out why it wouldn't accept the 800-45. It usually takes an 800-40. I'm a different poster than the one above asking the same question, but I'm also trying to figure out what way for this to work.

Of all things, why should deletion and moving of files slow my computer?[edit]

I'm asking out of curiosity rather than dire need, since it's rarely an issue. Here's the thing:

My computer's fairly old by most people's standards, 200mb RAM, 133mhz, still running Win98. However, it serves me well, rarely giving me any problems, I can do pretty much everything I need to do with it and plenty of stuff I needn't: it gets lots of use and rarely complains.

What puzzles me is that occasionally I manually go through all the folders and files just to chuck out clutter and sometimes I've been very lazy about where I put my creations, so I do some tidying of files etc.

After a while of deleting and moving files my system slows to a crawl beyond anything that seemingly more intensive tasks (listening to audio files, streamed radio, playing a game with some graphical content, looking at flash animations etc) does.

It slows down horribly (moving one file from one folder to another can take 20 seconds, opening another folder in explore view takes 30 seconds). Why should that be? It's all solved with a reboot, as you may guess. --bodnotbod 02:43, 19 September 2005 (UTC)[reply]

How often do you compress your disk space, that is to say run the utility to clean up the spaces on disk space between chunks of data? I also have Win 98 and I use Norton Utilities Speed Disk for this purpose. It is possible that if your disk space is severely fragmented, such that the sizes of the spaces are not big enough for the files you want to move, or if it has to search through a lot of spaces to find what is needed for the move, that would explain part of the problem.
I also find it neccessary to reboot my PC when it starts to go sluggish ... in my case it goes sluggish after
  • I spend a modicum of time on the Internet, like here at Wiki;
  • Do some heavy duty keying in e-mail or other word processing;
  • Do some real work telecommuting to the office;
  • Play some old fashioned DOS games.
I suspect that some activities are not doing a good job of returning memory and other rsources when finished, meaning the resources not being used effectively on the next task that I wish to run.

AlMac|(talk) 04:38, 19 September 2005 (UTC)[reply]

  • Ah, yes, I shall definitely defrag my drive later. I tend to do that very rarely. Although, perhaps because I tend not to download very much or install new software I can leave it for quite a long time and still get "your disk is only x% (where x is a small integer) fragmented and doesn't require...". I don't tend to have trouble with the other things you mention, in particular using the net and sending emails which I spend the bulk of the time using this for. Anybody else have something to add? --bodnotbod 17:29, 19 September 2005 (UTC)[reply]
Yes, close any apps that you're not wanting to use real soon and empty your browser's cache. --hydnjo talk 19:24, 19 September 2005 (UTC)[reply]
I close the apps when I'm doing one of the cleaning sessions. Not sure I want to delete my browser cache: it comes in handy. --bodnotbod 02:12, 20 September 2005 (UTC)[reply]
When your system slows, try emptying your recycle bin. Files you delete go into the recycle bin and still occupy diskspace. Why it slows your pc is beyond me, but it might help. Z.

what is the discipline of logic[edit]

You might want to start at logic. -- Rick Block (talk) 03:06, 19 September 2005 (UTC)[reply]

According to Galen, the venous and arterial systems were separate. This was the prevalent view in medicine until:[edit]

William Harvey -- Jmabel | Talk 03:55, 19 September 2005 (UTC)[reply]

Galen also has the answer, though I suspect your textbook would as well. Garrett Albright 03:59, 19 September 2005 (UTC)[reply]

But see also Ibn Nafis, who described the circulation of the blood in 1242 (though his work was forgotten and did not influence the prevalent view). Gdr 10:35, 19 September 2005 (UTC)[reply]

refrigerator humidity drawers[edit]

My fridge has two humidity drawers; one labeled high humidity, and one labeled low humidity. I'm just learning the ways of fridges, and I have some vegetables that are starting to wilt. Can you tell me what the two drawers are for, and which one my broccoli is supposed to go in? -Lethe | Talk 05:10, 19 September 2005 (UTC)[reply]

From a layman standpoint, I'd go with things that stay moist and things that don't. Since fresh broccoli is usually under a mister at a grocery store, I'd assume it's high-humidity and put it there. As for low... apples maybe? I'm not sure. — Lomn | Talk / RfC 05:15, 19 September 2005 (UTC)[reply]
  • High humidity: leafy vegetables. Medium humidity: apples, grapes, other thin-skinned fruits. Low humidity: Citrus fruit. Keep the broccoli in an open bag (or a perforated plastic bag) in the crisper (the other name for the high humidity bin.) --jpgordon∇∆∇∆ 05:23, 19 September 2005 (UTC)[reply]
  • On a related note, am I right to assume that the drawer next to my crisper, marked for meat, is simply a low humidity drawer? — Laura Scudder | Talk 05:41, 19 September 2005 (UTC)[reply]
So the high humidiy drawer is for vegetables, and the low humidity drawer is for meat? sounds good to me. -Lethe | Talk 05:44, 19 September 2005 (UTC)[reply]
On some brands/models some cold air (from the freezer) is piped into the meat and low humidity bins. This has the additional effect of lowering the temperature by 2 or 3ºF. --hydnjo talk 19:15, 19 September 2005 (UTC)[reply]

aeroplane[edit]

what are the parts of an aeroplane and what are their funtion???

Uh...how much detail do you need? (Don't forget the wings, and does the pilot count as part of an aeroplane?)--inks 09:44, 19 September 2005 (UTC)[reply]
You probably want to check out control surfaces and Category:Aircraft controls. — Lomn | Talk / RfC 13:09, 19 September 2005 (UTC)[reply]
Also Flight controls. DJ Clayworth 14:02, 20 September 2005 (UTC)[reply]

How does asparagus grow?[edit]

Arr, sparrow-grass grows much like reg'lar grass, matey. I suggest ye be readin' our article at asparagus for more. Garrett Albright 12:13, 19 September 2005 (UTC)[reply]

What is formed from ammonia and the breakdown of simple sugars[edit]

Is this a homework question? (If so, please see the notes at the top of this page). Notinasnaid 10:42, 19 September 2005 (UTC)[reply]

urinary system[edit]

(no question)

  • What would you like you like to know about the urinary system ? Notinasnaid 11:06, 19 September 2005 (UTC)[reply]
  • You're not just taking the...sorry. DJ Clayworth 13:59, 20 September 2005 (UTC)[reply]
    • It's when you're boredly reading something and you come across these little gems of beautiful humour that you realise why you do this. Thanks. Rob Church Talk | Desk 00:05, 21 September 2005 (UTC)[reply]

Gigabyte[edit]

Hi, I was just wondering... How many characters (text-wise) could fit into a gigabyte? Cirrial 11:34, 19 September 2005 (UTC)[reply]

  • Like most questions, the answer is "it depends"...

For someone from the western world, they are used to working with characters that occupy exactly one byte. "An egg" would use six bytes. Note that spaces and new lines are all characters (a new line is often two characters). In this case, a gigabyte would hold exactly as many characters as bytes in a gigabyte. But how big is a gigabyte? Either two to the power 30 or ten to the power 9, respectively 1073741824 or 1000000000. If we say "about a thousand million" we won't be far wrong.

So, for many people in the western world, about a thousand million characters. But what about people outside the western world? In Japan, they often use a system where a character fits in two bytes. So in this system, the Japanese could get about 500 million characters.

Many people, all over the world, use a system called Unicode. There is more than one kind of Unicode. If they use a kind called UCS-2, every character is 2 bytes. That means that if you use UCS-2, you get about 500 million characters, no matter what language is used. Another kind of Unicode is called UTF-8. In this, non-accented English letters use one byte, european accents use two bytes, and Japanese uses three bytes. So it really depends now, somewhere between 300 million and 1000 million characters.

This all assumes a file contains nothing but text. In Windows terms, a TXT file. Other types of file may have overheads so there are less characters than you would get in TXT, or compression, so there are more characters. DOC files have overhead, so it is less characters (of the actual text). PDF files have overhead and compression and may well get more characters than a TXT in some cases (not others).

Clear now? Notinasnaid 11:46, 19 September 2005 (UTC)[reply]

Is light matter?[edit]

Why don't you check out light and matter. — Laura Scudder | Talk 15:52, 19 September 2005 (UTC)[reply]
No, it is not. Light is made up of photons, which are massless. Besides, photons are bosons, not fermions out of which matter is made. —AugPi 05:58, 20 September 2005 (UTC)[reply]
Minor nitpick—while the protons, neutrons, and electrons of which matter is made (mostly) are fermions when examined individually, in some composite particles they become bosons. Helium-4 atoms, for instance, are bosons. TenOfAllTrades(talk) 07:06, 20 September 2005 (UTC)[reply]
Many hands make light work. --Sum0 20:38, 21 September 2005 (UTC)[reply]

un authorized access to computers[edit]

ho ppl ive got an assignment to make for university..and i really need some help. the topic is "Discuss and describe how we can stop unauthorized access to our computers?" please send me as much research on this topic as possible.. thx.bye

Arr, who you be callin' a "ho ppl?" You be walkin' the plank in a moment, but first I be suggestin' you check out our articles on computer security and computer insecurity. Arr. Garrett Albright 16:31, 19 September 2005 (UTC)[reply]
Ahoy! In case any of ye landlubbers be wondering, today (September 19) is ITLAPD: International Talk Like a Pirate Day. Arr. TenOfAllTrades(talk) 16:56, 19 September 2005 (UTC)[reply]
Aw come on, it's like fight club. You're not supposed to tell them it's fight club! :) - Taxman Talk 18:54, 19 September 2005 (UTC)[reply]
You can start with articles
  • computer security talks about how to design computer systems so that they are secure.
  • computer insecurity talks about the reality that most people, companies, governments, being in a world in which there are cheap insecure computers, and expensive secure computers, buty the cheap ones, then spend enormous time and expense to deal with the insecurity problems.
  • I started on computer security audit and when time permits (I been extremely busy of late), clean it up some more so it conforms to Wiki style. This talks about a type of checking that can be used to dramatically reduce the cost and time and effort needed to make insecure systems more secure, and identify what areas of education its owners and managers need in the area of improving security.

AlMac|(talk) 18:02, 19 September 2005 (UTC)[reply]

DNA computing magazines[edit]

What are the most distinguished scientific magazines for DNA computing? Can you point me to any good review-articles? -EnSamulili 16:16, 19 September 2005 (UTC)[reply]

Academic magazines usually don't come along until a research area has been around awhile. Research will start in related conferences, then get its own. You might start here and then start looking up author names on scholar.google.com and following bibliographic references. Lunkwill 09:41, 22 September 2005 (UTC)[reply]

How do I get rich[edit]

Hey how do I become really rich and also maybe famous. Cheers. -JoeMoe

You could become rich by winning a lottery a time or two. Once rich, proceed to behave eccentrically and fame (or at least undue media attention, which is almost as good) will shortly follow. Alternately, rob a bank. Infamous is the same as maybe famous, right? — Lomn | Talk / RfC 18:47, 19 September 2005 (UTC)[reply]
Ayy, shiver me timbers, edit conflict. If you check the archives, matey, this question has been answered before. But basically getting rich requires either producing the value yourself or stealing it. The first way involves producing a good or service that other people are willing to pay more for than it costs you to produce. That may be making a widget or hitting a golf ball really well. Stealing could be direct, or embezzling. Ask the Tyco guys and others how that went for them. As to getting famous, simply do something or be something that people want to talk about and think about. That may be personality, acting, doing really dumb or really smart things, or being really wealthy. - Taxman Talk 18:51, 19 September 2005 (UTC)[reply]
Also, on no account consider sending money to anyone advertising who claims they will make you rich. You may, however, consider placing such an ad yourself, though this road to riches is paved with criminal convictions. But also consider "There is a tide in the affairs of men / Which, taken at the flood, leads on to fortune." - William Shakespeare. I've always been drawn to that quote, I think it sums up very well the money made by a) being at the cutting edge and b) fads. So it accounts for the fortunes of the dot-com boom and Furbies alike. --bodnotbod 02:08, 20 September 2005 (UTC)[reply]
A rhetorical answer to a rhetorical question: by some standards you already are rich. You have a computer or access to one, you have the time on your hands to ask this question, you probably have more stuff than you need, you probably don't want to earn your own stripes, and you most likely, don't want to hear any of this. Oh well and too bad, you asked. --hydnjo talk 02:24, 20 September 2005 (UTC)[reply]
Becoming famous is easy. Get a Wikipedia account. Make lots of useful edits, write some needed articles, and get them up to featured article standard. Voila, you're famous. Fortune is sure to follow.-gadfium 05:07, 20 September 2005 (UTC)[reply]
Barring theft and luck, the best way to make money is to already have it. The next necessity is brains to make it multiply. Alas I cannot provide them for you :) . As for the initial money, you might try a bank, but you won't be the first one not to succeed there. The faster you want to get rich, the more spectacular your idea will have to be and the less likely it will be you get money for it. Sorry. DirkvdM 11:40, 21 September 2005 (UTC)[reply]

auto alternators[edit]

just read your article on alternators - i have an older car with an alternator without a built in regulator - the car presently has a voltage meter which i would like to remove by adding a diode to the system - the diode would connect to the alternator and to the warning light in the car - need a diode that could be wired to the ignition such that the light would be lit when the car ignition is on but would turn off when the alternator was producing at least 13 volts - is this part available - if so could you refer me to the source - thanks in advance for the help - love your site

I don't think a simple diode is quite what you're after, though I haven't found quite what you're looking for. I know something along the right lines exists, however; most modern vehicles are fitted with a light that goes on when the alternator's not producing anything and the battery is being drained. Maybe you'll have to visit an auto electrical supply shop.
Though I'm a little confused as to why you'd like to remove the volt meter. Is it spoiling the appearance of your car? --Robert Merkel 04:40, 21 September 2005 (UTC)[reply]
There's a reason they call them idiot lights - gauges are much more useful in warning you of a problem. Anyways, if you REALLY want to do this, you probably need a zener diode. - Bloodshedder 02:54, 23 September 2005 (UTC)[reply]

global ecosystem[edit]

The global ecosystem is also known as the Earth's (this is where the answer goes)?

See Ecosystem. --hydnjo talk 01:14, 20 September 2005 (UTC)[reply]

Different link styles on one webpage[edit]

How can I put different link styles on one page? I want the links in a menu sidebar to be block and not-underlined; text links in the text to be inline and underlined; and image links in the text to be inline without a border (like the speaker we use for audio samples on Wikipedia). Bonus question: how can I get the audio samples to play without loading Quicktime in a new page? I want the reader to be able to continue reading the article while the sample downloads and plays. Thanks in advance, Mark1 02:12, 20 September 2005 (UTC)[reply]

  • To change particular links, you need to meddle with User:Markalexander100/monobook.js. I don't know which id the side menu has, but I'm pretty sure you can change those links seperately from the rest. For the other question: simply install foobar and set it to play .ogg files by default. Tada no more new Quicktime window. If you're using Mozilla Firefox, there's an extension that let's you incorporate certain media players into your browser without opening additional windows. - Mgm|(talk) 08:27, 20 September 2005 (UTC)[reply]

Sorry, I was unclear. The link formats I want are for a website which I'm producing, not for Wikipedia. And I want readers of the site to be able to play the samples without opening a new page, (ideally) whatever browser they're using. Thanks, though. ;) Mark1 08:52, 20 September 2005 (UTC)[reply]

You want to use Cascading Style Sheets, segregating your styles by general location on the page. You should define common styles (like text color) without prefix, and then prefix contextual links with their containing element (a <div> works well). Also note that the image links will have most of their styles on the image, not the link. Here's a quick sample:
a { color: #00F; }
#sidebar a { text-decoration: none; display:block; }
#maintext a { /* nothing past default needs definition */ }
a img { display:inline; border-width:0; vertical-align: middle; }
Also, W3Schools is a pretty good reference site for CSS. — Lomn | Talk / RfC 13:08, 20 September 2005 (UTC)[reply]

Thanks a lot- I'll give that a go. Mark1 01:43, 21 September 2005 (UTC)[reply]

chalk blocks[edit]

How is chalk made into blocks from powder form. nellie

  • Did you mean blackboard chalk? Well, if you did, blackboard chalk is genrally not chalk (limestone) but gypsum. Our gypsum article says that gypsum has an "unusual property: when mixed with water at normal (ambient) temperatures, it quickly reverts chemically to the preferred dihydrate form, while physically "setting" to form a rigid and relatively strong gypsum crystal lattice". I think this basically means that if you you wet powdered gypsum, mould it and let it dry, it keeps the new shape: blackboard chalk. If you are more specific we can help you further. --Commander Keane 07:09, 20 September 2005 (UTC)[reply]

Paper vs. Trees[edit]

How many pieces of 8 1/2 x 11 or A4 paper would you need to conserve to save a single tree?

And how many angels can dance on the head of a pin and how many roads must a man walk down ... --hydnjo talk 03:56, 20 September 2005 (UTC)[reply]
Industry produces paper in vast, bulk quantities, driven by market forces. You would need to interrupt supply and demand in some way. ᓛᖁ♀ 04:27, 20 September 2005 (UTC)[reply]
  • I think it's vaguely possible -- don't quote me on this, though -- that the questioner may indeed have been wondering how many sheets of paper you get from a single tree. --jpgordon∇∆∇∆ 05:19, 20 September 2005 (UTC)[reply]
You could make a rough estimate by weighing a ream of paper and dividing it by 1000 ( I thnik there are a thousand sheets ina ream). Then you could compare this mass with the mass of a typical felled tree (Which I don't know but someone else might) Theresa Knott (a tenth stroke) 05:22, 20 September 2005 (UTC)[reply]
Actually there are 500 sheets of paper in a ream. --WhiteDragon 20:55, 26 September 2005 (UTC)[reply]
I suspect a major flaw here. Paper is largely dry and wood probably contains more than 90% water, so you might have to multiply the result by ten or something (ie you would need ten times as many trees). DirkvdM 11:45, 21 September 2005 (UTC)[reply]
A minor flaw, too...a significant weight of kaolin goes into white paper. It's not all pulp.--Joel 06:19, 27 September 2005 (UTC)[reply]
There's a start here, though these seem to be paper industry stats. - Nunh-huh 06:44, 20 September 2005 (UTC)[reply]
  • There is a misconception here. Paper companies plant trees to make paper from, based on consumer demand. If people start buying less paper, they plant fewer trees. Using less paper does not "save" trees.

    On the other hand, as noted by jpgordon, we can still ask how many sheets of paper can be made from one tree. Based on the link provided by Nunh-huh, we can get 89,870 "sheets of letterhead bond paper" from a cord of wood, and 10 – 15 cords from an acre of forest. Thus: 898,700 – 1,348,050 sheets from an acre. Now all we have to do is figure out how many trees are typically in an acre. Random Google searches produce numbers like 25, 50, or 100. So let's say 25 – 100 trees per acre. Dividing, we get 8,987 – 53,922 sheets per tree. In summary, it varies, but 10,000 – 50,000 is probably a decent estimate.

    Nowhither 18:01, 20 September 2005 (UTC)[reply]

  • Without wishing to turn this into a brawl over logging practices, that's rather an oversimplification. Some paper companies clearfell old growth native forest which is then turned into paper. Sure, they replant the trees afterward, but the forest does not recover its biodiversity for many years afterwards, if ever. See Gunns Limited, for instance. --Robert Merkel
    • Yes, that's true. The replanted trees are not there to make a forest; they are there to turn into paper. Tree farms, like most farms, are monocultures. But then it gets really tricky. Why do we want to "save trees"? If global warming is your concern, well, a tree farm is probably a fine way to sequester carbon. And with someone watching carefully to prevent it from burning, I suppose it is probably better for carbon sequestering than a natural forest. In any case, higher demand for paper is definitely going to increase the total tree biomass in the world. So if you want to prevent the Antarctic ice sheets from melting, then buy lots of paper. On the other hand, if biodiversity is your goal, then buy less paper. What if you like both? That's really hard. — Nowhither 23:24, 21 September 2005 (UTC)[reply]
Don't forget production and disposal costs of the paper. All of that burns hydrocarbons. Lunkwill 09:33, 22 September 2005 (UTC)[reply]
Also, people seem to be missing the fact that not all logging companies replant at all. Some just move on. Superm401 | Talk 21:19, 23 September 2005 (UTC)[reply]

I recall seeing a poster at my school claiming that conserving 118 pounds of paper is equal to saving one tree. Sorry, but I havn't got a source or anything to back me up.

Well, I'm afraid that poster was quite wrong (not that one shouldn't conserve paper). — Nowhither 00:58, 23 September 2005 (UTC)[reply]

phosphatidylserine for brain function[edit]

  • And your question is? - Mgm|(talk) 08:29, 20 September 2005 (UTC)[reply]

How does Kate's tool work[edit]

Could you give me a general explanation on how something like Kate's tool works. I'd ask User:Kate but they are probably rather busy. The only language I know is MATLAB, so a qualitative response would be appreciated. --Commander Keane 07:35, 20 September 2005 (UTC)[reply]

I presume it queries the database (no idea if directly or indirectly via something like Special:Contributions) and parses a list of edited pages for known prefixes like Wikipedia: or User talk:. Any entry without such a prefix is in regular article space and is counted as such.
All that said, there shouldn't be any problem dropping Kate a quick note asking for a simple (and correct) answer. — Lomn | Talk / RfC 13:18, 20 September 2005 (UTC)[reply]
You're not all that wrong. It will most likely issue a couple of SQL statements to the MySQL server, using the database specified in the drop-down box. The queries are likely similar to these:
User ID: SELECT user_id FROM user WHERE user_name='(your user name)';
I'll refer to this value as UID from here...
Total edits: SELECT COUNT(cur_id) FROM cur WHERE cur_user=UID;
             SELECT COUNT(old_id) FROM old WHERE old_user=UID;
Those two are added together, obviously.
Distinct pages edited:  SELECT COUNT(cur_id) FROM cur WHERE cur_user=UID GROUP BY cur_title;
                        SELECT COUNT(old_id) FROM old WHERE old_user=UID GROUP BY old_title;
Again, added together.
Edits/page (avg) is calculated as .
First edit: SELECT cur_timestamp FROM cur ORDER BY cur_timestamp ASC LIMIT 1;
            SELECT old_timestamp FROM old ORDER BY old_timestamp ASC LIMIT 1;
from which the smallest is selected (assuming both queries return a value)...
Edits by namespace: SELECT COUNT(cur_id) FROM cur WHERE cur_user=UID GROUP BY cur_namespace;
                    SELECT COUNT(old_id) FROM old WHERE old_user=UID GROUP BY old_namespace;
For each namespace, the values are added together and mapped to the proper name.
Naturally, I can't guarantee that's exactly how it works, since I haven't seen the source code of the tool, but based on the database structure of MediaWiki, and the PHP source of SpecialContributions.php, that would seem to be the way to do it (or at least, *a* way). --Pidgeot (t) (c) (e) 14:19, 20 September 2005 (UTC)[reply]
Well, for the best answer, look at the source itself. Pidgeot's on the right lines, but there's no longer a separate "cur" and "old" to worry about (since MediaWiki v1.5) and she's used "JOIN" statements that get things from multiple tables in one go, and keywords like DISTINCT and MIN to do some further processing inside the SQL. There's not really much that can be done to "qualitatively" explain an SQL statement, since it's not a procedural language, and most of the syntax is designed to be fairly self-explanatory. - IMSoP 01:26, 29 September 2005 (UTC)[reply]

The website related to the change of atmosphere pressure[edit]

Can you give me some website that is related to what will cause the change of atmosphere pressure?

have you looked at the Atmospheric pressure article on wikipedia? Boneyard 11:34, 20 September 2005 (UTC)[reply]

ODBC - Pros & Cons[edit]

ODBC is a useful technology for allowing a developer to connect to a wide range of databases without needing to alter their applicationcode. But some people have a belief that ODBC is not as good as developing an interface direct to the database's native code. Can anyone list the pros and cons of using ODBC to connect to the database and what pitfalls or advantages their might be in using ODBC?

Welll... I guess the more layers of abstraction the more function calls you must call. Calling on an external library which then calls on a database function takes longer than just directly accessing the database. To be honest though, I think won't cause too much performance problems, and accessing a standardised interface has many more pros than accessing different databases in different ways. - 203.134.166.99 08:11, 20 September 2005 (UTC)[reply]

MDAC and ADO.NET[edit]

Question: where does ADO.NET sit in the MDAC stack? Do Microsoft consider it part of MDAC at all? The MDAC roadmap does mention it, but doesn't state whether it is part of the .NET framework or whether it is part of the MDAC framework. I know that ADO.NET must use a .NET Managed provider, but these just appear to be a simplified version of an OLE DB provider, even though they don't use COM and instead use the .NET CLR. Can anyone give me an answer to this? I'd like to update the MDAC article. - 203.134.166.99 08:08, 20 September 2005 (UTC)[reply]

Answer to my own question: the ADO.Net Managed provider is part of MDAC and part of .NET. - 203.134.166.99 06:47, 27 September 2005 (UTC)[reply]

Testing[edit]

what is the difference between client server application testing and web application testing?

Web App testing is properly a subset of client-server app testing. However, both are generic enough (and test methods and requirements are flexible enough, depending on particular application), that little more comparison can be made. — Lomn | Talk / RfC 13:13, 20 September 2005 (UTC)[reply]

human body[edit]

why do we have a running nose when we catch cold?

have you looked at the Common cold article? Boneyard 11:32, 20 September 2005 (UTC)[reply]

Gary Killdall[edit]

Does anyone know who Gary Killdall was and why he was important to the computing industry ?

Mark Davis

The correct spelling appears to be Gary Kildall (that's the one used by his employer, Digital Research, on their website). That should cover what you're looking for. — Lomn | Talk / RfC 14:38, 20 September 2005 (UTC)[reply]

Phosphate rock[edit]

By how much will the oceans rise if all the ice melts (assuming the amount of water in the atmosphere stays the same)? A.F. Stewart, Dresden.

Well, according to our article on the West Antarctic Ice Sheet, if it melted completely ocean levels would rise about 5 metres. That is rather unlikely any time soon, even if global warming is real. --Robert Merkel 02:13, 24 September 2005 (UTC)[reply]
5 meters?? That's it?? what about preasure changes due to the heat of water vapor?? If atmospheric preasure were dropped low enough, wouldn't that affect the total system volume>?--205.188.117.71 04:20, 29 September 2005 (UTC)[reply]
If ALL the ice in the world melts, sea levels will rise about 60 metres. - MPF 20:26, 29 September 2005 (UTC)[reply]

real estate[edit]

What is the name of the person who came up with real estate? —Preceding unsigned comment added by 207.62.88.10 (talkcontribs) 10:46 EDT, 20 September 2005

The concept of real estate, that is land as property, dates back to the start of farming, and there is no record of how the concept was first devised. i doubt that any one person was responsible. See Real estate for much more information. DES (talk) 15:55, 20 September 2005 (UTC)[reply]

Rocks[edit]

What is your questiion about them? DES <sup>(talk) 16:00, 20 September 2005 (UTC)[reply]

Rocks rock. — Lomn | Talk / RfC 16:10, 20 September 2005 (UTC)[reply]
How igneorant. Rob Church Talk | Desk 00:09, 21 September 2005 (UTC)[reply]
Now, now; be gneiss. TenOfAllTrades(talk)
This is clearly a hard question --jonon 12:24, 23 September 2005 (UTC)[reply]
Indeed, the questioner has put us between a rock and a hard place -- Qaz (talk) 04:23, 29 September 2005 (UTC)[reply]

Recusive function[edit]

consider a network of streets laid out in a rectangular grid, for example. In a northeast path from one point in the grid to another, one may walk only to the north(up) and to the east(right).How can I Write a C++ program that uses a recursive function to count the number of northeast paths from one point to another in a rectangular grid.


General Idea That I have is that one simply recourses north until one falls off the grid.Recursion automatically takes care of backtracking to a legal grid point & then recursing to the east.

Assuming you can always go north and right at each grid point, the number of paths is equal to , where h is number of grid points horizontally and v the number of gridpoints vertically. This can be computed using or recursivly using Pascal's triangle. --R.Koot 18:03, 20 September 2005 (UTC)[reply]
If you can't make that assumption, think about this: the total number of paths that go from your present location to the destination is the sum of a) the number of paths to the destination from the location to your east (if you can go east) and b) the number of paths to the destination from the location to your north (if you can go north). --Robert Merkel 00:40, 21 September 2005 (UTC)[reply]

Salt and vinegar[edit]

Do salt and vinegar potato chips have any notable health effects? ᓛᖁ♀ 17:46, 20 September 2005 (UTC)[reply]

Well, too much salt is bad for you, they always say. Vinegar I'm not sure about. The crisps themselves may be a problem due to acrylamide content, though I don't think that's settled yet.
Having moved from the UK to the USA, I do miss them. They're much harder to find over here. -- DrBob 18:06, 20 September 2005 (UTC)[reply]
Where in the USA? They are practically a staple in the northeast. Yum, --hydnjo talk 18:14, 20 September 2005 (UTC)[reply]
The southwest. Nor can you get cheese and onion, prawn cocktail, or smokey bacon crisps for love nor money. Probably because all the hideous chemical flavourings are banned in the US, but still. --DrBob 18:21, 20 September 2005 (UTC)[reply]
They can make you fat, of course. More generally, they are very high in carbohydrates, with all their associated troubles. And salt is problematic for some people. But vinegar is generally pretty healthy stuff, so I doubt there would be any health troubles associated with this type of potato chips that wouldn't also be associated with normal potato chips. — Nowhither 18:47, 20 September 2005 (UTC)[reply]
  • Salt and carbohydrates can both be a good thing when used with caution. Too much of either isn't healthy but too much vitamins isn't healthy either. Salt is useful to keep up the salt content of your body and carbohydrates are good for energy (of course it's no use if you don't use the consumed energy). The brain needs a particular sort of carbohydrate (I think it was sucrose, but I'm not sure), so don't even bother not eating any carbs at all. - Mgm|(talk) 19:17, 20 September 2005 (UTC)[reply]

The flavouring used is sodium acetate, which would seem to be remarkably inocous as artificial flavourings go. My answer would be "no more so than any other crisps (sorry, chips)". Physchim62 14:49, 22 September 2005 (UTC)[reply]

Megaparsecs[edit]

The furthest quasars detected have been seen to have a distance of 5500 megaparsecs. If a megaparsec approximately equals 3.26 x 10^6 and if the maximum possible distance something can be detected away from us is 13.7 billion light years away which equals around 4202 megaparsecs much less than the distance from the furthest away quasar how is it possible to detect a quasar when light hasn't had enough time since the beggining of the universe to reach us?--Mr A Pinder 21:45, 20 September 2005 (UTC)[reply]

Without extensive research on my part, most likely because 13.7 bn ly is no longer the detection limit. Astronomers observing a new oldest-thing-ever is fairly common. — Lomn | Talk / RfC 20:08, 20 September 2005 (UTC)[reply]
That doesn't seem to explain anything though. How is 13.7bn ly no longer the limit, did we increase the speed of light? Go forward in-time? The only assumption I can make is that the object in question (a quasar with 5500 megaparsecs distance) was in fact within the 13.7 bn ly distance when the light first got here using it's redshift the actual distance was calculated. That's the only explanation I can think of though. --82.7.209.62 11:39, 21 September 2005 (UTC)[reply]
The exact age of the universe isn't known. As scientists learn more (by finding new quasars for example) the numbers get revised. I think that is all that is happening here. Theresa Knott (a tenth stroke) 22:23, 21 September 2005 (UTC)[reply]
March 2005 Sciam article "Misconceptions about the Big Bang". Whole article is interesting, but in case the link doesn't work, the relevant part:
If space were not expanding, the most distant object we could see would now be about 14 billion light-years away from us, the distance light could have traveled in the 14 billion years since the big bang. But because the universe is expanding, the space traversed by a photon expands behind it during the voyage. Consequently, the current distance to the most distant object we can see is about three times farther, or 46 billion light-years. Frencheigh 22:44, 21 September 2005 (UTC)[reply]

Eyelashes[edit]

Many hairs on the human body appear to initially grow in a locally prefered direction, but curly hairs generally seem to curl in random directions. Eyelashes on the other hand are highly ordered and align in their direction of curling. Is there an interesting story as to how this happens? -- Solipsist 19:41, 20 September 2005 (UTC)[reply]

  • Maybe it's their similarity to whiskers (or vibrissae)?

Wisteria trees[edit]

How can you tell a male from a female wisteria tree?

Trees, being a form of plant, have both genders; their flowers have both stamens and stigmae. Rob Church Talk | Desk 00:11, 21 September 2005 (UTC)[reply]
The situation described in plant sexuality is rather more complex and certainly allows for the possibility of separate male and female plants, at least for some species. As for wisterias, I would have no idea, but to wait for beans. — Pekinensis 23:19, 21 September 2005 (UTC)[reply]
I just checked with an expert -- my local Master Gardner helpline. I was told that Wisterias have "perfect form" i.e. they have both male and female parts on all trees. not all trees have this property, but it seems that wisterias do. DES (talk) 20:48, 22 September 2005 (UTC)[reply]

Methadone clinics[edit]

How many methadone clinics are there both public and private in the United States?

Jim

ethernet[edit]

Do you have a question about ethernet? AlMac|(talk) 20:22, 20 September 2005 (UTC)[reply]

How to build a TV station[edit]

I am interested in building educational TV Station in Afghanistan. From the technical point of view what kinds of equpments I need.

  • What types of cameras do I need.
  • What types of camera and equpments are required inside the TV Station.
  • What types of equpments I need to broadcast. (The type of equpments and
  • How far they could reach.)
Unfortunately, I think this topic is far too complex to give a useful answer in a paragraph or two. You will need quite a lot of equipment and a considerable amount of expertise to set up your station, and the necessary equipment will cost lot of money, even if you buy it second-hand. To figure out what you require, you will need to consult extensively with an expert, or spend some time working with TV people yourself to learn what's required (Foriegn press reports have mentioned that there is a TV station in Kabul, for instance).
If I were you, I'd also go and speak to some of the foriegn aid agencies working in Afghanistan. Not only could they put you in touch with people who have the expertise you need, they may be able to help out with sourcing of the right equipment.
In terms of the transmission range, if you use terrestrial TV transmission it will basically be line of sight. Any mountains between your transmitter and the recipients will mean they won't be able to see your signal. So if you want good reception range make sure your transmitter is at the top of a mountain to maximise the area that it can be viewed from!
Good luck with your project, which sounds very admirable. I wish I could be more helpful. --Robert Merkel 00:20, 21 September 2005 (UTC)[reply]
I am no expert on this either. Perhaps the organizations helping the rebuilding effort can put you in touch with relevant experts in their home nations. You'll need transmitting of a kind that will match the reception for your audience.
There are also international organizations that help developing nations in general with such things. Sometimes it is more economical to use satelite dishes in remote villages. An advantage of this is that they can access educational TV from many nations, not just what you are putting out.
  • for example Trinet is a European effort, mainly in Africa to aid developing nations inexpensive communications. This Trinet link also has info about a discussion group precisely to help people in developing nations get at resources for projects such as yours.

AlMac|(talk) 00:25, 21 September 2005 (UTC)[reply]

One more thing to consider; community television organizations run on very low budgets, often with volunteer staff, so they know a thing or two about running a minimalist TV station. It's another area to look into which might provide some helpful contacts. --Robert Merkel 06:02, 21 September 2005 (UTC)[reply]
Also many public television stations in the USA that deliver via cable television, they are able to have several cable channels, at least one devoted to public interest topics, in which the general public is invited to attend classes in how to operate the cameras, can rent them (leave a deposit sufficient for insurance), to do community coverage of general interest.
You might also check with the Universities of Journalism around the world. Some of them have their students publish inexpensive newsletters, with faculty advisors. This might help you see what is the state of art in other nations, expectations of journalism excellence standards.
Columbia School of Journalism publishes CJR Columbia Journalism Review
I tinkered slightly with your question above to make it more obvious what you asking about, because my areas of know-how not quite on point with what you need.

AlMac|(talk) 07:12, 21 September 2005 (UTC)[reply]

A Variable Speed of Light[edit]

The particle theory of light states that the main particle in light is a photon and it has been proven that photons of light must have a mass (even if extremely low) because they cannot escape a black hole. If, theoretically, you shone light directly into a blackhole this would certainly speed up the light photons as by Newton's law; acceleration = mass x force. Therefore how is it that the speed of light is thought of as a constant?

Furthermore in the special theory of relativity as you get close to the speed of light, time itself slows down until it comes to a stand still at the speed of light. If light was to go faster than the speed of light (assuming the 'speed of light' is the speed of light throughout space) then wouldn't that mean the light entering a blackhole was going back in time?

Photons do not have mass, not even an extremely low one. They are affected by gravity because gravity warps the entire spacetime. However, although photons are massless, they do have momentum, and this value describes their behavior when influenced by gravity. Note though that Newton's laws don't hold for these calculations; we must use general relativity instead. In particular, it is wrong that acceleration = mass × force. This is only true for very small speeds; when the speed begins to approach the speed of light, infinitely much more energy is needed to accelerate further. Only massless particles can travel at the speed of light. See the articles on photons, the speed of light, and the theory of relativity for more information. Fredrik | talk 20:56, 20 September 2005 (UTC)[reply]
Well, of course it's wrong that "acceleration = mass × force". However, it's at least approximately correct that force = mass × acceleration. And that is the usual modern statement of Newton's Second Law. However, what Newton actually said was that force = d(momentum)/d(time). And that is exactly right. Even for light. — Nowhither 23:14, 20 September 2005 (UTC)[reply]
I can't believe I missed that one... - Fredrik | talk 10:50, 21 September 2005 (UTC)[reply]
The speed of light is based upon the medium it travels through. Light travels faster through space than through the atmosphere, and faster through air than glass. Physics demands than when a particle falls toward a black hole it has to gain energy. Particles with mass go faster to do this, massless particles instead gain energy, so you see a blue-shift as the photon's frequency increases.
Many thanks for the answer i've always thought particles affected by space-time must have a mass but of course they only have to be affected by space-time e.g everything. --Mr A Pinder 21:44, 20 September 2005 (UTC)[reply]
This touches on something I've been wondering about concerning black holes. As I understand it, all the strange effects one sees when something enters a black hole only really hold from the perspective of an outside observer, but for something (someone?) entering the black hole everything is 'as normal' (ie the normal laws of physics still hold from that perspective?). So is it that the light really doesn't change frequency but it only appears so for the 'outsider' who sees the distortion of space/time (which the 'insider' doesn't)? DirkvdM 12:34, 21 September 2005 (UTC)[reply]
Good question. But you have it backwards. The outsider would not see light approaching a black hole, but light which is moving away from a black hole would be red shifted. It is not possible for light to escape a black hole, but a light emitting object moving close to a black hole, or any superdense body, would show a red shift. An observer would need to be between the black hole and the light emitting object to see a blue shift. As far as whether the light 'really' changes frequency, that is a difficult question, almost in the realm of metaphysical debate. Physics says the light would have to gain energy to move into a gravitational well, and the only way to do that is to increase frequency. If that is due to a space distortion, well maybe so.
I was thinking theoretically, not empirically. Of course you can't 'see' a photon moving towards a black hole because it would have to 'emit something' we can detect. But indeed without the possibility of empirical testing it's little more than metaphysics. But isn't much of modern day physics then not really metaphysics? I anticipate two possible reactions to this. One is irritation (suggesting I'm on to something). The other is that I've just exposed myself as the layman (read 'philosopher') I am :) . DirkvdM 06:58, 23 September 2005 (UTC)[reply]

By the way, there is an interesting and somewhat related issue here that no one has brought up. Yes, the speed of light varies depending on the medium it is passing through. When we say "nothing can go faster than the speed of light" we really mean "the speed of light in a vacuum". That means that if we are not in a vacuum, then it is theoretically possible for an object to go faster than light.

This has interesting implications. What happens when an object in the air goes faster than sound? Answer: a shockwave, whose sound is often called a "sonic boom". An object traveling faster than the speed of light in some medium can produce similar electromagnetic shockwaves. These go by the name "Cherenkov radiation". For more information, refer to that article.

Nowhither 23:35, 21 September 2005 (UTC)[reply]

Bacteria[edit]

What are the differences of Gram-positive and Gram-negative bacteria?

See our articles on Gram-positive and Gram-negative. Chuck 21:58, 20 September 2005 (UTC)[reply]

tadpoles[edit]

how do tadpoles get to be full breed forgs i mean how does the process work?

As you might expect, it's a very complex process, and there's no quick answer to the question. Research into questions like that is a very active part of cutting-edge research in biology (which is another way to say "there's a lot we still don't know"). That area of biology is called developmental biology, and the article I've linked to might be a good starting point. Chuck 23:02, 20 September 2005 (UTC)[reply]
It might also be fun to check out axolotl; some creatures stay tadpole-like for their entire natural life-cycle, and there's even more research to be done on them.--Joel 14:49, 29 September 2005 (UTC)[reply]

"Find the zeros of each function"[edit]

How do I solve problems that ask "find the zeros of each function" and give an (f)x equation, such as:

f(x) = 3x - 8
f(x) = (x/2) + 5
f(x) = 19

--69.163.150.70 00:18, 21 September 2005 (UTC)[reply]

If you want to find the zeroes of a function, you should replace the f(x) with 0 and solve the resulting equation, for instance, the first one will become

If you need help solving equations like that, might I suggest you have a look at Doctor Math and his very extensive explanation of problems like these? --Robert Merkel 00:32, 21 September 2005 (UTC)[reply]

In the first two examples you provide, there is only one zero per function. As you can see from your third example, it is not always possible to find any zeros of a function:

(replacing with 0 results in a falsehood ())

The other two require a little algebra:

Add 8 to each side.
Divide each side by 3
There is one zero: When
Subtract 5 from each side
Multiply each side by 2


Another interpretation of the phrase "zero of the function" could include the function evaluated at x=0, in which case there's an additional zero in each problem...but fairly trivial to solve for.--Joel 14:53, 29 September 2005 (UTC)[reply]

graph theory[edit]

what is a weakly connected dominating set?

See Glossary of graph theory. Chuck 04:05, 21 September 2005 (UTC)[reply]

Volcanoes and the hydrosphere(seawater, and underwater volcanoes)[edit]

  1. ) How do volcanoes at mid-ocean ridges affect the temperature of seawater?
  2. )How do volcanoes change the chemistry of seawater?
  3. )How does seawater affect the composition of the volcanic rock that is formed at the mid-ocean ridge?
  4. )Would volcanoes affect a small body of seawater, such as the Red Sea, the same way as a large ocean like the Atlantic?
  5. )Can a change in the volume of volcanic rock formed at mi-ocean ridges change sea level?
I tinkered with your question just a little bit to make it easier to see what your questions are. I am not an expert at this, and I am a novice at using reference works, but hope what little I know can help you navigate what is here. I looked at several articles here, such as Volcanology and concluded that there is a lot of room here for upgrading the content of these articles, given recent news such as the 2004 Indian Ocean earthquake disaster.
  1. Do you suppose there might be an article on volcano in this on-line encyclopaedia?
    1. Yes, and how about Submarine volcanos on the ocean floor? However, seems to me it is somewhat of a stub article that does not go into much detail on your questions of the chemistry of lava in sea water.
      1. Bogoslof Island is the summit of a mainly underwater stratovolcano in the Bering Sea, 31 miles from the main Aleutian volcanic arc (That's off Alaska USA).
      2. There are several parts of Hawaii that came from the sea, thanks to volcanic eruption, that started totally underwater, such as Loihi Seamount.
      3. Monowai Seamount is another, in the vicinity of New Zealand. It is still totally below sea level.
    2. The Wiki article on Hydrosphere is also a stub so we may have to go outside Wiki to get you a good answer.
  2. You might find some info of relevance in the Wiki articles on geology, hydrogeology and other effects of underwater volcanoes, such as tsunami (giant tidal waves).

AlMac|(talk) 04:15, 21 September 2005 (UTC)[reply]

computer networking[edit]

list five different network devices

Another case of "Do your own homework", but because it's so simple: Network Interface Card, Hub, Switch, Bridge, Router. Thank you, thank you, I'm here all week. Try the veal.--inks 03:23, 21 September 2005 (UTC)[reply]
You missed "wire" :-) Veal's off Tonywalton  | Talk 22:06, 21 September 2005 (UTC)[reply]
A telephone is a network device.
A fax machine
Their value is related to how many others of the same kind of device they can talk to.
Is a "wire" a "device"? AlMac|(talk) 07:25, 22 September 2005 (UTC)[reply]

Does hot or cold water freeze faster?[edit]

I have heard many arguments for the question, does hot water or cold water freeze faster, and was wondering if someone could fully answer this for me, and explain why there is such a debate on this question. --confused

See Mpemba effect. ᓛᖁ♀ 03:46, 21 September 2005 (UTC)[reply]
  • If you had 6 or 7000 atm of preasure, you could try and play around with the properties of super critical OH2, that would warp your brain like a pretzel ;), or if you're really interested, go see your dry-cleaner, he/she might let you play with the supercritical CO2 --205.188.117.71 04:15, 29 September 2005 (UTC)[reply]
Amazing! And puzzling. My first reaction was that the warmer water (or whatever) would have to cool down to the temperature of the cooler water, from which point the situation is equal for both, so the warm water should always take longer to freeze. But as I understand the article (which I don't entirely :) ), the difference is not so much the present temperature but the fact that the liquid has been warmer. In other words, if two vessels of water have previously had a higher temperature and one is given a headstart by letting it cool down a bit before the 'contest' then it will 'win'. Right? DirkvdM 12:48, 21 September 2005 (UTC)[reply]
My understanding is hot water, due to the heat driving out dissolved gas, will freeze faster.
Which fits in with what I said. The difference then lies in the fact that the water was previously heated. Not in the actual temperature. DirkvdM 07:57, 22 September 2005 (UTC)[reply]
More info from the Straight Dope. - Bloodshedder 03:00, 23 September 2005 (UTC)[reply]

how many light years is in a minute[edit]

Your question, as written, makes no sense. A light year is a measure of distance; it is how far light travels through vacuum in a year. A minute is a measure of time. People occasionally use the term "light minute" and "light second", as described in the light year article. --Robert Merkel 05:44, 21 September 2005 (UTC)[reply]
A light minute is 11160000 miles or 186000 * 60.
Don't forget that the Millenium Falcon made the Kessel run in less than 12 parsecs. :-) — Nowhither 23:55, 21 September 2005 (UTC)[reply]

how many miles does the earth travel around the sun?[edit]

The Earth article lists the details of the Earth's orbit; the orbital circumference is 0.940 Terametres, which is another way of say 940 million kilometres. --Robert Merkel 05:50, 21 September 2005 (UTC)[reply]

940 million kilometers is about 580 million miles --WhiteDragon 13:32, 28 September 2005 (UTC)[reply]

Water Weight[edit]

Is hot water heavier than cold water?

Generally not. Our article on water says that "Fresh water has its greatest density at 4°C. It becomes less dense as it freezes or heats up." --Robert Merkel 05:52, 21 September 2005 (UTC)[reply]
However, if we take your question literally, from my limited understanding of special relativity the same number of molecules of hot water will be ever so slightly heavier than the same number of molecules of cold water. According to special relativity, the rest energy of an object is directly proportional to its mass. The rest energy of the hot water is greater than the cold water. Therefore, it must have greater mass. In practice, this effect will be almost undetectable, though, because the amount of energy "contained" in a mass that the extra energy from a little bit of heat is a tiny fraction of the object's total energy. However, we usually don't count molecules to measure quantities of water, we measure volume. --Robert Merkel 05:59, 21 September 2005 (UTC)[reply]

It depends how much of each you have. A litre of hot water is a lot lighter than a swimming pool full of cold. for example. Tonywalton  | Talk 22:08, 21 September 2005 (UTC)[reply]

elevators[edit]

(no question)

  • They're usually used to get up and down in building without the use of stairs. See elevator. - Mgm|(talk) 10:28, 21 September 2005 (UTC)[reply]
  • They are also aircraft control surfaces and grain storage facilities, among other things. See elevator (disambiguation). — Lomn | Talk / RfC 13:22, 21 September 2005 (UTC)[reply]

achievements of world nations in remote sensing technology[edit]

(no question asked)

Did you have a question? You could try the Wikipedia remote sensing article. Or you could post again and ask a question. — Nowhither 01:01, 23 September 2005 (UTC)[reply]

Yellow bell[edit]

What is the phylum or division name of a Yellow bell?

Magnoliophyta. See the article Allamanda. ("Yellow bell" should also take you there now that I have created a redirect page.) - Fredrik | talk 11:16, 21 September 2005 (UTC)[reply]

Electricity[edit]

What is static electricity?

Does the article, static electricity, explain it for you? - Fredrik | talk 11:08, 21 September 2005 (UTC)[reply]

Browser IDs[edit]

How can I change the user id string of Mozilla Firefox without using an extension? --anon

You go about it in the about:config menu (accessed via the address bar), full instructions available here [20]Lomn | Talk / RfC 13:19, 21 September 2005 (UTC)[reply]

Circulatory system[edit]

Are there any really interesting facts about the circulatory system? It's for a project about the human body that I am making here.

I would suggest poking through our article circulatory system to see if anything jumps out at you. Some of the historical ideas about circulation are quite interesting. The articles linked in Template:Cardiovascular system are also quite extensive. You'll have to judge for yourself what counts as 'really interesting'. TenOfAllTrades(talk) 17:53, 21 September 2005 (UTC)[reply]

Can someone point me to the latest fully functional (not a trial version or security patch) of WinZip. Somehow, this particular system I'm working on hasn't got one installed... :( - 82.172.23.66 18:06, 21 September 2005 (UTC)[reply]

The free (trial) version at winzip.com is functionally the same as the full version. — Lomn | Talk / RfC 18:49, 21 September 2005 (UTC)[reply]
  • I know, but I'd prefer a version that lasts beyond a 21-day trial. - Mgm|(talk) 21:13, 21 September 2005 (UTC)[reply]
There's no actual limit on its usage (at least not with older versions, not sure about 9.0), you just have to live with the nag message.
Failing that, there are plenty of free alternatives. 7-Zip is a free archiver with support for many formats, including (as you might guess) the .7z format, which tends to offer much better compression ratios than ZIP. --Pidgeot (t) (c) (e) 22:03, 21 September 2005 (UTC)[reply]
  • Original poster: The way you get the non-nagging version of WinZip is to get the trial version, and, when it nags you, click the button to pay for it. It seem to remember the button is labeled "Buy Now". In any case, you can't miss it. — Nowhither 01:04, 23 September 2005 (UTC)[reply]

Bipolar Medically speaking.[edit]

Would it be safe to say that those who've listened to D.Leppard and Queen and Pink Floyd would be considered having a trait of Bipolar tendisies??? How often should these tendicies be seen??Why isn't the Bipolar medication today worth taking without side effects??? Shouldn't doctors use herbs and other natural substances instead??? why make things worse?? thanks for seeing my email

christie lofton, christielofton@hidden.com
I've obscured your email address, as it's probably a bad idea to leave it out in the open for spam harvesting. Anyway, to attempt to answer:
  1. No. Debatable taste in music is not a medical condition.
  2. Bipolar tendancies vary by individual.
  3. Most medicines have side effects in one form or another. Generally, as a drug type remains on the market, continuing research reduces side effects.
    • Actually, it's the opposite. The longer a drug remains on the market, the more side effects become recognized. Use a drug when first released-- it's safest then. Also, it stays the same drug as long as it is marketed. They don't tinker with it to make it safer. alteripse 08:07, 27 September 2005 (UTC)[reply]
  1. That depends primarily (in my personal opinion, I would hope completely) on the established effectiveness of such remedies.
Hope this helps. — Lomn | Talk / RfC 18:45, 21 September 2005 (UTC)[reply]
You could try reading An Unquiet Mind by Kay Redfield Jamison, Published: Picador (UK) ISBN 0330346512 Vintage (US) ISBN 0679763309. Jamison is a practising psychiatrist and a sufferer from severe bipolar disorder; the book's a good insight into the disorder itself, the effects it has, and into its treatment. She omits to mention any general connection with musical taste, though. Tonywalton  | Talk 22:19, 21 September 2005 (UTC)[reply]
People who listen to the dull cock rock of Def Leppard are far too mentally stilted to exhibit the outre signifiers of bipolar disorder. Similarly Queen appeals to the unsophisticated good time sensibility that may engage a bipolar person in an up phase but lacks (Bohemian Rhapsody excepted) the poetic spirit we romantically like to think as being part and parcel of tragic melancholy. Once Syd Barrett left, Pink Floyd appealed only to architects. This answer may be deemed controversial by other wikipedians, but I wouldn't listen to anyone else. Stick with me: I know what I'm talking about. --bodnotbod 15:11, 22 September 2005 (UTC)[reply]
Oh, I don't know. Who wants to live forever is pretty much down-phase. Tonywalton  | Talk 23:38, 24 September 2005 (UTC)[reply]
Shock horror! Pink Floyd produced some of the definite progressive rock of our times. So there. Yeah, you've been told. :P Rob Church Talk | Desk 21:00, 25 September 2005 (UTC)[reply]

what hypothesis did beadle and tatum test in their experiments on neurospora[edit]

Typing neurospora into the search box on the left will allow you to find articles which will probably help you to do your homework. In future, please refer to the instructions at the top of the page and note that Wikipedia isn't here to do your homework for you. TenOfAllTrades(talk) 18:56, 21 September 2005 (UTC)[reply]

new technology[edit]

Latest technology[edit]

What are some of the latest technologies available in the classroom and/or to teachers/students?

We already know someof the things available, but what are the newest forms of technologies to aid in education and learning?

Are there any "cutting edge" ideas, computers, computer applications, websites, downloads, etc... that can help teachers and education majors to integrate technology into their classrooms?

If possible, could you please email a response to Susan Wurth at Kaskaskia College(Re: Crystal's Question for EDUC 116)? Her email address is <REMOVED> Thank you!

Well interactive whiteboards are gaining in popularity. Theresa Knott (a tenth stroke) 20:13, 21 September 2005 (UTC)[reply]

To the original poster: You have hit on a topic that Wikipedia does not cover very well. We have an article on Educational technology, but it is a lousy article. Perhaps I'll go clean it up. Or you can, if you want. Meanwhile, there is plenty on the web. See, for example, the "Encyclopedia of Educational Technology" at http://coe.sdsu.edu/eet/ . And here is a blog about the topic: http://people.uis.edu/rschr1/et/blogger.html . Also, I have removed Susan's e-mail address, so that every spammer in the world doesn't start deluging her with nonsense. May I suggest that you do not post e-mail addresses (especially if they belong to other people??) in a public forum. — Nowhither 00:27, 22 September 2005 (UTC)[reply]
Educators are learning how to use the power of the Internet to help them network with each other and do their job more effectively.
  • There are discussion groups for particular kinds of teachers, school librarians, etc.
  • Homework Hotline is a deal where a student having trouble with subject X, phones the service and ends up speaking with a teacher of subject X at the student's grade or age level, but who is at some different school system ... different city, public parochial, whatever. Result is the student gets immediate help, and the teachers record the kinds of questions that re-occur so they can improve the curriculum.
  • Parents can go to a web site, select which school, grade level, subject, to see what the homework assignments were.
  • There's article in today's USA Today Newspaper about mental health professionals watching pre-schoolers to catch those that have disabilities that can be helped early.

AlMac|(talk) 07:32, 22 September 2005 (UTC)[reply]

Here's an interesting way to use the Internet for educational purposes. Give students the task of adding to a Wikipedia article. That would be less boring to the student, they have to research a subject for themselves, interpreting and rewording the findings, in stead of just memorising what some textbook says (a direction society is going in I'd say) and it will give them more satisfaction because they actually do useful work (for all the world to see!). I suppose this means more work for the teacher, who has to read through the edits, but it would certainly be less boring for them too. DirkvdM 08:07, 22 September 2005 (UTC)[reply]
An interesting idea indeed, but please can the teacher him or herself first research the Wikipedia criteria and impress them on the students before they start? Quite a few establishments do do this (or say "contribute an article" rather than "add to an article") , but sadly it can end up with a number of two line articles on things like "My school dining room" or "Mr Hockenheimer my Teacher, which are deleted, sometimes speedily, which must be a bit discouraging for the students who have contributed them in good faith. Tonywalton  | Talk 23:49, 24 September 2005 (UTC)[reply]

cost of touch screen feature[edit]

Why do Tablet PCs cost much more than laptops. Are they costly because they have touch screens and pen? How much does it cost to add a touch screen to a device? Or simply, how much does the touch screen facility alone would cost? --Tom

Touch screens are indeed very expensive (1.5-4x price?) compared to normal LCD screens, but any prices that I might be able to quote for desktop monitors won't be relevant to laptops. I imagine the small market for tablet PCs compared to the much larger market for laptops has an effect on price too. Interestingly, touchscreen LCDs seem to need constant (sometimes daily) recalibration -- do tablet PCs suffer the same problem? Ojw 20:34, 21 September 2005 (UTC)[reply]
I'd hazard that prices are also slightly higher to help Microsoft recoup OS development costs since they're not able to sell retail or upgrade versions of XP Tablet Edition. Re: Ojw, mine only needs recalibration when I flip between landscape and portrait mode, owing to the reorientation of my hand when near the screen edges. — Lomn | Talk / RfC 21:15, 21 September 2005 (UTC)[reply]

what trees do acorns come from[edit]

Oak trees (see acorn). — Lomn | Talk / RfC 20:09, 21 September 2005 (UTC)[reply]

History of Science examples from Kuhn's SSR[edit]

Hi all - I am putting together a website for a class taught by the department of Logic and Philosophy of Science at UC Irvine. Among other things, the students will be reading Thomas Kuhn's The Structure of Scientific Revolutions. For those of you who haven't read it, it is packed with history of science examples that are, to put it bluntly, never explained. Since the students will mostly be sophomores in college, I am endeavoring to put together a glossary of Kuhn's examples. This glossary has a few sentences on each examples, and links to Wikipedia for a more detailed description. Unfortunately, I have been unable to track down many of these examples, and so I'm turning to you for help! Anything you know about any of these examples would be a great help. If this stuff is somewhere on wikipedia, please tell me where so that in composing my entry I can link there. Here are the examples I cannot locate:

  • Some method for measuring electrical attraction using a pan balance (I have an idea how this would go, but I'd like to know who did this and to what use it was put)
  • Some problem in applying Newton's laws to the moon, and the solution proposed by Alexis Clairault.
  • He makes a reference to the "Pneumatic Chemists"
  • Kepler and the motion of mars
  • "The proliferation of gases" surrounding Priestly. That's really all he says.
  • The effluvium theory (some theory of electricity)
  • Hauksbee's apparatus (something to do with electrostatic repulsion)
  • Musschenbroek and his observations regarding a charge filled bottle (I assume this is a leyden jar)
  • The theory of elective affinity (some chemical theory that had to do with acids, I think)
  • Rayleigh's paradoxes of electrodynamics (I don't think this is supposed to be Rayleigh scattering)
  • Kepler's Rudolphine tables. (I assume this is the Tabulae Rudolphinae mentioned in the article, but I don't know what the tables are for)
  • Poisson and the a "reductio" of the wave theory of light (this refers to the prediction of a bright spot at the center of a shadow, which was latter observed by Fresnel. Do have anything on this anywhere?)

  • The Joule-Lenz law H=RI^3 (does this go by a different name now? I can't find it under that name)
  • Do we really not have an article on Christian Huyghens? (okay, maybe this is just rhetorical)

Phew... that's a lot, I know. Believe me its only a small number of the total examples. Anything at all would be incredibly helpful! Thanks a million! --best, kevin ···Kzollman | Talk··· 21:22, 21 September 2005 (UTC)[reply]

No... I meant the mispelled one. *bows head in shame* In my defense (and I double checked) Kuhn refers to him as "Huyghens" and there are websites using that spelling. I've created redirects. Thanks, I'll see if I can find something on effluvium. --best, kevin ···Kzollman | Talk··· 22:40, 21 September 2005 (UTC)[reply]
  • Joseph Priestley did indeed have a grand proliferation of gases - he identified eight, according to our article, when "air" was previously thought to be the only one to exist. He was a co-discoverer of oxygen, among others. I believe there's a recent biography of him; skimming that might be useful since our article is pretty sparse. Shimgray 22:43, 21 September 2005 (UTC)[reply]
Yeah, the case for oxygen is hard, since he called it "dephlogisticated air", right? Anyway, Kuhn says that the proliferation of gases was considered a problem in Priestley's day, any idea why? Kuhn seems to say even Priestley thought this was a problem. --best, kevin ···Kzollman | Talk··· 22:48, 21 September 2005 (UTC)[reply]
  • for the motion of mars see Kepler#Kepler's_laws. In short, Kepler was trying to reduce complex and error-fileld observatrional data to simple laws of motion. Mars presents a particualrly tricky case, becasuse its orbit is outside earth's orbit, which as earth overtakes mars leads to apparent "retrograde motion". the diagram at the above link shows a simple version of this, but a good bio of Kepler is probaly worth finding and skimming as our article is a bit brief here. DES (talk) 22:57, 21 September 2005 (UTC)[reply]
Great! Thanks, that article will be a lot of help! --best, kevin ···Kzollman | Talk··· 00:43, 22 September 2005 (UTC)[reply]
Poisson's spot: Poisson thought he'd killed Fresnel's wave theory of light (which he was a big critic of) by showing that it predicted a (seemingly absurd) spot of light that should be visible at the centre of a circular shadow. However, François Arago experimentally found the spot almost immediately. A Fresnel zone plate is essentially an expansion of this idea. -- DrBob 01:15, 22 September 2005 (UTC)[reply]
  • More: The Joule-Lenz law is also just called Joule's law.
  • I'd just point out that most of Wikipedia's articles on scientists and history of science are unfortunately not very informed by critical historical literature, and are instead what Kuhn calls "textbook science" in the introduction to SSR. So beware in using it for your examples! The best book (quick, straightforward, and accurate) for anything relating to early modern physics or astronomy is J.L. Heilbron's Elements of Early Modern Physics which the Irvine library probably has a copy of, if you are still stumped after all of this. (Heilbron, incidently, was Kuhn's research assistant while he was writing SSR; now he's the grand old man of history of physics, but still very active) --Fastfission 02:56, 22 September 2005 (UTC)[reply]

How weight system originated?[edit]

How weight system originated?

e.g 1Kg., How first unit of Kg. came in to existance?

Axay

Axay,

Our Units of measurement article has a good section on the history of various forms of weights and measures including the SI or metric system. You may also wish to check out the U.S. customary units for the history of the imperial system now used mainly the US. Capitalistroadster 01:09, 22 September 2005 (UTC)[reply]

Project 1947[edit]

What was Project 1947? Has to do with UFO's in Bayreuth, Germany in 1947.

It appears that Project 1947 was an effort beginning in the 1990's by UFO enthusiasts/researchers, to examine lots of local newspapers from a period in which UFOs were reported to be observed. If the page isn't working for you, try this page from the Google Cache. One of the links is to a page selling copies of their preliminary report. --Robert Merkel 02:46, 22 September 2005 (UTC)[reply]

remembering amino acids[edit]

Are there useful mnemonics, rules, shortcuts et cetera, to help one remember all 20 common amino acids, whether they're polar, nonpolar, acidic or basic, that sort of thing? --Miborovsky 01:45, 22 September 2005 (UTC)[reply]

astronomy[edit]

what is our comic address in order of increasing size? Where is the Sun in the Milky Way?

  • The sun is located on the outer edge of the milky way, roughly 28,000 lightyears from the galactic center.
  • In addition: We know that the Sun is on the edge of what is probably a 'spur' called the Orion Spur which seems to merge with the Perseus Spiral arm towards the constellation Cygnus. The Perseus Arm is the one just outward from the Sun's location in the Galaxy about 700 to 1000 parsecs from us. Beyond the Perseus Arm, there may be a more distant one, but the arms become less distinct in the outer galaxy zone. from [22]
  • I don't quite understand the first part of your question. Please elaborate. - Cobra Ky (talk, contribs) 04:33, 22 September 2005 (UTC)[reply]
I believe the OP meant cosmic address in increasing size. So, for me, Fukiage district, Chikusa ward, Nagoya city, Aichi prefecture, Honshu island, Japan, Asia, Earth, the Solar System, then apparently the Orion Spur, the Perseus Spiral arm, the Milky Way galaxy, then…? Garrett Albright 04:53, 22 September 2005 (UTC)[reply]
Then the Local Group, Virgo Supercluster, etc... peruse the large-scale structure of the cosmos for more. — Lomn | Talk / RfC 04:55, 22 September 2005 (UTC)[reply]
Saying that we live near The Sun in the Local Group is a bit like writing "My house/My Street/My Town" as an address. Unfortunately, we don't know anyone who lives elsewhere to standardise our addresses with... Ojw 11:23, 22 September 2005 (UTC)[reply]
We live on a blue-green planet that orbits an unregarded yellow star far out in the uncharted backwaters of the unfashionable Western spiral arm of the galaxy...Brian Schlosser42 20:22, 26 September 2005 (UTC)[reply]
By the same token, according to The Hitchhiker's Guide to the Galaxy, the Earth is in sector ZZ9 Plural Z Alpha. See also: [23] --WhiteDragon 13:52, 28 September 2005 (UTC)[reply]

Cyclones v's Hurricanes[edit]

Could someone explain the difference between a Cyclone and a Hurricane?

Thanks.

A

It's solely where they form, see tropical cyclone for cyclones, typhoons, and hurricanes. — Lomn | Talk / RfC 04:53, 22 September 2005 (UTC)[reply]

Diesel Engine[edit]

It is practically difficult to obtain more than 2500 rpm in a 4-stroke diesel engine. Why?

This is a good question, and it's not actually covered by the diesel engine article. According to this page on the Perkins diesel corporate site it seems it's because it's more difficult to get the fuel-air mix into the cylinder in the right quantities and distribution for combustion than in a petrol engine, though the limiting factor is different for large diesels than small ones. Note that modern automotive diesels can generally do up to about 4500 rpm, and the very latest diesels (for instance, one in the upcoming Mercedes-Benz S-class) can do up to about 5000. --Robert Merkel 06:17, 22 September 2005 (UTC)[reply]
My explaination is due to the compression ratio in a diesel the engine components need to be more robust than those in a gasoline engine and so are more massive. Also, I think diesel burns more slowly so it is harder to get more engine RPM.
Also you need to take into consideration the stroke length. Diesel engines tend to have a longer stroke than many gasoline engines in order to get high compression ratios as well as take advantage of that slow burning fuel. A longer stroke length means that the rods, piston heads, and crank arms are all moving a greater distance per revolution. Take a look at the largest of the old muscle car engines that have relatively high compression and relatively long stroke lengths aand you should find that they redline around 5000ish (I've ranges from about 4400 to 5900).

Water[edit]

why is water clear

  • Our Water article is quite detailed but doesn't explain this. However, this article seeks to explain the colour of liquid water [24]. Capitalistroadster 06:53, 22 September 2005 (UTC)[reply]
A biological explanation is that we descended from creatures who only saw light that had passed through water, and therefore had no evolutionary need to sense frequencies that water is able to absorb. At the moment, any light you see passes through your aqueous humor, which has similar optical properties to pure water, so it would still be difficult to see anything that water tends to block, no matter how your retina were constructed.--Joel 15:31, 29 September 2005 (UTC)[reply]

aluminium[edit]

Aluminium dross.[edit]

Aluminium Dross is a mass of solid impurities floating on a molten aluminium bath. It is caused due to oxidation of the metal. It can easily be skimmed off the surface before pouring the metal into a mold. What are the constituents of Aluminium dross ?


Ranjan

  • I would say that the result of oxidising aluminium in air would be primarily aluminium oxide, but I'm not a chemist. Bovlb 07:12, 22 September 2005 (UTC)[reply]
  • I am and I don't see any reason to disagree with you if aluminium gets oxidized you get aluminium oxide. - 131.211.210.12 07:44, 22 September 2005 (UTC)[reply]
  • If the dross is formed during the extraction of aluminium, it is quite impure, with significant amounts of magnesium and silicon oxides. If it is formed on the surface of pure molten aluminium, it should be pretty pure aluminium oxide. Physchim62 14:20, 23 September 2005 (UTC)[reply]
  • There's a chance of some carbides and nitrides, as well.--Joel 15:33, 29 September 2005 (UTC)[reply]

whom should we pay to register a domain[edit]

Why should we pay for registering a domain? To whom does that money go? For example, yahoo provides that service for $4.99. If possible, tell me How much does yahoo take in that and who takes the rest?

Because somebody has to maintain the root servers for the Domain name system, and that costs money. Yahoo's margins on its business is almost certainly a secret, but given the number of companies offering an identical service it's likely to not be very big. Ah, the wonders of the free market. Shop around and find the cheapest price! ---Robert Merkel 07:29, 22 September 2005 (UTC)[reply]
More specifically, a handful of companies are taking millions of pounds per year to run the DNS services that used to be done by one person (Jon Postel?) on his spare PC. See Verisign, ICANN and IANA for some of the companies involved, and a bit more about where the money goes. Ojw 11:19, 22 September 2005 (UTC)[reply]

Pioneers of Adsense[edit]

Google has a program called Adsense in which they place text ads on other sites. Did they pioneer this idea? or did someone else pioneer this concept? I would also be interested in knowing who pioneered Adwords first. Was it overture?

Msn TV / WEB TV and mouse[edit]

Why does a MSN TV or WEB TV does not have a mouse? Are TVs not cabable of supporting mouse OR is it for some other reason?

In the early days of computing, we used a TV instead of the modern day computer monitor. Since then, both the standard digital devices and the analog TV have evolved dramatically, and the analog TV is being replaced by a digital TV. I have a hard time believing the mouse cannot be supported, since I used it on a variety of standard and non-standard screens of PCs and Ncs, so I figure the reason has got to be related to what they are marketing, and possibly not want to spend much money on tech support for different customers with different kinds of home devices. AlMac|(talk) 07:41, 22 September 2005 (UTC)[reply]
Essentially, a Web TV is a small computer running special software that uses the TV for display and has an internet connection of some kind. Making a computer work with a mouse is a fairly trivial task which can be done for just about *any* computer and could certainly have been done for the little computer in the WebTV box. The reason why they didn't is probably because they thought that the users of WebTV would be sitting on the couch, and therefore the flat surface you need to operate the mouse would not be available. --Robert Merkel 07:39, 22 September 2005 (UTC)[reply]

What cause low pressure and high pressure at an area?[edit]

Can you tell me how the atmosphere pressure change at an area?What would probably cause the atmosphere pressure of an area change?Does the sunlight play the important role on it?Or else?

  • See Atmospheric pressure: "Atmospheric pressure is the pressure above any area in the Earth's atmosphere caused by the weight of air." If the sun gets through that's probably because there's less clouds to obstruct its view (which would have an effect on pressure. That's why barometers are used to predict weather. But the sun does not have a direct influence. - 131.211.210.12 07:48, 22 September 2005 (UTC)[reply]

Density of Water[edit]

Why is the density of water 1 gm/cm^3? I have been under the impression that either the gram or the meter (I don't remember which) had been defined so this would be true at a certain temperature and pressure. Is this the case, or is it just a coincidence? Superm401 | Talk 07:45, 22 September 2005 (UTC)[reply]

  • This is only true for standard conditions. If water freezes it gets a higher density. If it boils it's density goes down. The fact it's 1gram per cubic cm is pure coincidence. It's the weight and volume that were defined. - 131.211.210.12 07:50, 22 September 2005 (UTC)[reply]
    • I mentioned that I understood it was only true in a one case. However, it can't be both defined and a coincidence. Which is it? Superm401 | Talk 08:13, 22 September 2005 (UTC)[reply]
A nit to pick. When water freezes it density is lower, this is why ice floats. Most materials are not like this.
The kg was originally defined as the mass of 1 l of water. So the density of water is 1 kg / dm^3, which is 1 g / cm^3. By the way, gram is abbreviated to 'g', not 'gm'. DirkvdM 08:33, 22 September 2005 (UTC)[reply]
Thank you. So kilogram was originally defined that way, but no longer is? Also, thanks for the reminder about the gram abbreviation. I'm surprised I messed that one up. Superm401 | Talk 08:36, 22 September 2005 (UTC)[reply]
The kilogram (see our article) was originally defined as the mass of 1 L of pure water at 4 degrees Celsius (water reaches its maximum density at this temperature) and under 1 atmosphere of pressure.
Because it was difficult to work with a standard made of liquid water–the stuff sloshes about, evaporates, and leaks, and it's difficult to construct a standard 1 L container–the kilogram was officially defined in 1889 to be exactly equal to the mass of a particular platinum-iridium cylinder currently stored in Paris.
The kilogram is the only SI (metric) unit still defined in terms of an actual physical object. Other units (like the meter or the second) are defined in terms of specific measurable physical constants. (The meter, for instance, is defined as a fraction of the distance light travels in a second in a vacuum. The second is based on the frequency of a particular type of radiation.) This has generated problems—over the last hundred years, the standard kilogram has apparently gained a small amount of mass: about 50 micrograms. For highly precise measurements, such a change–about fifty parts per billion–is significant, and decidedly inconvenient. Scientists are now attempting to develop a new standard for the kilogram that doesn't depend on a physical artifact.
Our article says it lost that amount(50 micrograms). Which is right(cite sources if possible)? Superm401 | Talk 14:26, 22 September 2005 (UTC)[reply]
Also, note for future readers: Light travels far more than a meter in a second. According to the definition of the meter, it travels 299,792,458. Still, I understood the point of the anon. Superm401 | Talk 14:28, 22 September 2005 (UTC)[reply]
And just in case anyone cares, the meter started out as one ten-millionth of the distance from the North Pole to the Equator, along a meridian passing through Paris. So if someone asks you the circumference of the Earth, say "40,000 km". Okay, that's not exactly right, and it varies according to the great circle chosen, but it's darned close, as well as easy to remember. Similarly, as originally intended, the density of water is 1 gm/cm3. Again, not exactly right, and varying by temperature and pressure, but very close, and easily remembered. One of these that seems to be pretty much accidental, but has the same features of accuracy and memorability, is that the speed of light in a vacuum is 300,000 km/sec. — Nowhither 21:00, 22 September 2005 (UTC)[reply]
Still easier to remember is that it's 1 billion km/h. That's about 8% off (it's actually 1,079,252,848.8 km/h) but a whole lot easier to remember (and it's in a more usual unit, km/h, which makes that part of it easier to remember - "well, it's 300,000 ehm ... something"). And for those who need such an easy way to remember it it doesn't matter much how accurate it is. As long as they're not off by a factor 100 or such :) . To stretch the point, it's better to know that the speed of light is about 1000 km/h than not to have a clue (even though it's over 20% off - it's 1225 km/h). And while I'm at it, for converting Celsius to Fahrenheit, in everyday situations don't bother with the x 9/5 + 32. Make that x 2 + 30. Much faster and fairly accurate for everyday temperatures. (it's precise for 10 C and 2 F off for 20 C (70 in stead of 68)). You could also remember that for every 5 C over 10 C you have to substract 2 F, but then it gets complicated again :) . DirkvdM 12:25, 23 September 2005 (UTC)[reply]
1 billion km/h. That's nice; thanks. — Nowhither 11:06, 24 September 2005 (UTC)[reply]

About hurricans[edit]

How does each and every hurrican get its own name, say Katrina, Rita? Is it given randomly by some metrologist? S 07:56, 22 September 2005 (UTC)[reply]

are forces such as centrifugal and g forces, forms of energy[edit]

like the subject matter/heading says

Are forces such as centrifugal or G forces also forms of energy

Energy and force are fundamentally different. Thank you. 204.38.191.99 11:34, 22 September 2005 (UTC)[reply]
Forces produce work (energy); work done over a period of time is power. Force may be measured in newtons (N), energy is commonly measured in joules (J), and power is measured in watts (W) or horsepower (hp).
However, centrifugal force is not actually a force per se. It is a fictitious force, the effect of an object having a tendency to move in a straight line (inertia), but not being allowed to do so. The force applied to an object in circular motion is the centripetal force, which pulls the object inward; however, the object has enough velocity to travel at a right angle (tangential) to this force, and travels "around" the inward force instead. If there was no centripetal force, the object would travel outward in a straight line along the tangent, but no force causes this outward motion.
G forces are also not forces. Again, they are caused by inertia — an object at rest will tend to stay at rest. So the answer to your question is yes: inertia is mass, and mass is equivalent to energy. ᓛᖁ♀ 14:32, 22 September 2005 (UTC)[reply]

BBE[edit]

Hi everyone! ^^

I would like to ...advise...(something like that ^^') that I am from Quebec so my english is not perfect...

I'll begin by presenting myself if you want to understand why I'm searching for that.

I'm 17 yeras old and i'm in a School ..I dont know how to say that but I'm doing electronics in this school ^^'

A teacher gave me an homework and one of the question is : What is the definition of : BBE

Anyone are able to find what is the signification of this abreviation. Even the teacher is'nt able to find it, I cnat find it on the internet And all the information that I have is that:

-It's about electronics -It's about the sound -Apparently that in the defenition, there's a "Bradley" somewhere, but not sure.

It's all what I have founded... if someone have the definition this question...please write it here! ^^'

Thanks in advance

And sorry again for my...poor english ^^'

Second[edit]

Why is the standard definition of the second based on the cesium atom? ᓛᖁ♀ 14:37, 22 September 2005 (UTC)[reply]

Based on atomic clock, it appears to simply be that cesium was the element used in the first accurate atomic clock and became the de facto standard for such clocks. Atomic clocks themselves are used for their fine resolution and precision. — Lomn | Talk / RfC 15:04, 22 September 2005 (UTC)[reply]
Isn't it true that there are now more accurate clocks than the original cesium atomic clock? [25] seems to indicate that some clocks are more accurate. If that is the case, why hasn't the standard been updated? --WhiteDragon 14:06, 28 September 2005 (UTC)[reply]
Computation is now easier, and using physics it's fairly straightforward to convert the official standard to a working calibration standard. Much easier than it would be to redefine the second: read The Mood is a Harsh Mistress for a novelist's take on the difficulties involved, which are imaginative, but probably not comprehensive.--Joel 15:41, 29 September 2005 (UTC)[reply]

define a body of water called a straight[edit]

It's actually spelled strait in this context. — Lomn | Talk / RfC 15:00, 22 September 2005 (UTC)[reply]
And here's your definition: wikt:straitCryptic (talk) 15:06, 22 September 2005 (UTC)[reply]
Straits are straight channels straitened as they are straught between land. ᓛᖁ♀ 15:10, 22 September 2005 (UTC)[reply]

What is the meaning of 'synthetic human' or a 'synthetic model' ?[edit]

What is the meaning of 'synthetic human' or a 'synthetic model' ? Is it something like robots ? I recently found this word in a website which says about future technologies.

I have also occasionally seen this concept, but it is not defined in my encyclopaedia of Science Fiction Concepts ISBN 0-7867-0887-0 Parameter error in {{ISBN}}: checksum
Do you watch Star Trek enough to be familiar with the character called Data, who is like an android. There's all kinds of artificial life forms, some humanoid, in science fiction and I think the synthetic one pre-dates Isaac Asimov popularizing the concept of robots. AlMac|(talk) 18:49, 22 September 2005 (UTC)[reply]
You might also want to check out our entry on cyborg. --Fastfission 02:08, 27 October 2005 (UTC)[reply]

Maths: Vectors[edit]

Hi everyone...

I'm currently working on a maths assignment (yr 12) and I am so stuck. This is the last question. I haven't got much clue.

C,OAB is a regular tetrahedron as shown where OA = a, OB = b, and OC = c.

G and K are the cenroids of faces OAB and OAC respectively.

1. Prove, using a vector method, that CG and BK intersect.

2. If H is the point of intersection of CG and BK find OH.

3. What ratio does H divide CG?

4. Prove that the altitudes of a tetrahedron are concurrent.

  • Bold = vectors.

If anyone could help me that will be extremely helpful. Thank you so much.

- Cindy

  • Well, you could start by noting that:
    • In a regular tetrahedron, the line joining the centroid of a face to the opposite vertex is perpendicular to that face (an altitude), and passes through the centroid of the tetrahedron.
    • A centroid can typically be determined by taking the mean of the defining points.
    • Two lines intersect if their four defining points are coplanar.
    • Four points are coplanar precisely when the parallelopiped they define has volume zero.
    • The voume of a parallelopiped can be calculated as the scalar triple product of its three edge vectors.
  • Some or all of those may be useful to you. Good luck with your homework. Bovlb 20:03, 22 September 2005 (UTC)[reply]

What should I do with my Internet venture idea?[edit]

I am having a new idea for an Internet Venture. But dont have a business plan. Just having the idea, thats all. What should I do so right now?

One first step you might consider is reading some books on starting your own business. You will find lots of them at your local library or in a bookstore. You might also enquire at your local community college whether they have a suitable short course on this topic; a friend of mine is in the process of doing just such a course. There may also be some government programs that assist people setting up small businesses. Good luck! --Robert Merkel 00:21, 23 September 2005 (UTC)[reply]

Moles[edit]

How many atoms are contained in 1 mole of Si (silicon) and how many molecules in 1 mole SiO2?

  • The answer is the same in both cases: Avogadro's number, about . -- DrBob 15:58, 22 September 2005 (UTC)[reply]
The reason for which is that a mole is a measure for the number of molecules (be they atoms or not). So no matter what substance you have, one mole of it has by definition the same amount of molecules. Of course the number of atoms in one mole of SiO2 is three times as high because one such molecule contains 3 atoms. But I'm diverging now. DirkvdM 18:56, 22 September 2005 (UTC)[reply]
But I'm diverging now. I hate it when that happens! — Nowhither 21:03, 22 September 2005 (UTC)[reply]
This is a somewhat murky field, though, since a crystal of Si is covalently bonded throughout, making it a single molecule. There might not be a mole of solid Si on earth, unless the little crystallites of it in cast aluminum and transformer cores add up faster than I would expect. The number of atoms would vary inversely with crystal size, but would be extremely large. From a bookkeeping point of view, though, it's simpler and univerally acceptable to assume that each atom of Si is its own molecule, as Nowhither did, ignoring the four bonds it has formed. I'm certain that's what your teacher assumed in asking the question; otherwise, the question loses any real meaning.--Joel 15:57, 29 September 2005 (UTC)[reply]

fractal[edit]

See fractal. --R.Koot 18:57, 22 September 2005 (UTC)[reply]

Where have all the dolphins gone?[edit]

What does the acquatic wildlife do during a hurricane? Do they swim deeper? Is there catastrophic loss of life?


Thank you.

Most oceanic life just doesn't care; hurricanes and other weather patterns significantly affect the surface only. — Lomn | Talk / RfC 18:35, 22 September 2005 (UTC)[reply]
Dolphins, on the other hand, being air breathers, probably have a harder time of it. Interesting question, I don't know the answer. -- Jmabel | Talk 05:33, 23 September 2005 (UTC)[reply]
Why would dolphins care either? There's still plenty of air around in the usual place. Unless they get sucked up by a tornado... Mark1 07:52, 23 September 2005 (UTC)[reply]

Steel[edit]

my question is 'What is steel' i no it is a metal but what is it!

Please answer my question a.s.a.p

Thankyou :D Anon (please)

Have you read our article on steel? — Lomn | Talk / RfC 18:33, 22 September 2005 (UTC)[reply]
(Also, rather than posting 3 similar questions, you can scroll to the bottom of the page and edit the first one) — Lomn | Talk / RfC 18:37, 22 September 2005 (UTC)[reply]
You could also try our article on alloys. Physchim62 14:24, 23 September 2005 (UTC)[reply]

how often do eclipses happen?[edit]

See eclipse, lunar eclipse and solar eclipse. --R.Koot 18:56, 22 September 2005 (UTC)[reply]

how old are the great redwoods of california?[edit]

See Redwood_National_and_State_Parks. --R.Koot 18:55, 22 September 2005 (UTC)[reply]

geology question[edit]

an adjective describing a liquid in which ions, atoms or molecules dissolved in it are so numerous they get close enough to chemically bond is called?

...a homework problem. --jpgordon∇∆∇∆ 21:04, 22 September 2005 (UTC)[reply]

No idea, but it strikes me as being in the field of chemistry, not geology. — Nowhither 21:05, 22 September 2005 (UTC)[reply]
I'd go with solvent, because ions dissolved in any solution are close enough to bond. That is how precipitation caused by Single displacement reactions occurs, for a start. Superm401 | Talk 21:44, 23 September 2005 (UTC)[reply]

Volume of the Proton[edit]

Matter is defined as occupying space and having mass. A proton must be matter since tangible matter is composed of protons in addition to other subatomic particles, and moreover the proton has mass (1.6726 × 10−27 kg, according to your article about the proton). So, a proton must occupy space. What, then, is the volume of a proton? Furthermore, what is the volume of the neutron, the electron, and so on?

[26] has a writeup on neutrons:
the volume of a neutron is approximately 5.6 x 10-54 km3
I would expect protons are similar; however, it increasingly appears that protons and neutrons aren't "solid" objects per se, but rather a collection of quarks which themselves can move about somewhat. [27] Electrons are even less suited to discussions of volume as their positions are discussed in probabilistic terms only (I believe the Heisenberg Uncertainty Principle is the reason for this). — Lomn | Talk / RfC 20:41, 22 September 2005 (UTC)[reply]
Sounds fine, although I must say that, when measuring the volume of a subatomic particle, the cubic kilometer is probably not the most appropriate unit. — Nowhither 21:08, 22 September 2005 (UTC)[reply]
I suspect these all have densities similar to neutronium, or 1014 to 1015g/cm³. This would give:
Particle Mass (g) Volume (cm³)
Electron 9.1094 × 10−28 9.1094 × 10-42-3
Proton 1.6726 × 10−24 1.6726 × 10-38-9
Neutron 1.6749 × 10-24 1.6749 × 10-38-9
...which at least agrees with NASA's neutron volume (though one wonders why anyone would describe particles in cubic kilometers). However, except for the neutron, it's probably impossible to find the true densities, due to the extreme difficulty of accumulating enough charged particles in one location to measure.
The electron seems like it may be somewhat denser, since it is only one particle while the proton and neutron are composed of three quarks. Also, it is sometimes claimed to be a charged black hole (see black hole electron), which, if correct, would presumably give it a Schwarzschild radius of 1.3526 × 10-57m, a volume of 1.0366 × 10-170m³, and a density of 8.7879 × 10136g/cm³, assuming objects smaller than the Planck length can exist. ᓛᖁ♀ 20:58, 22 September 2005 (UTC)[reply]

It would be interesting to compare the density of quark stars. Does anyone know what the Tolman-Oppenheimer-Volkoff limit is? ᓛᖁ♀ 21:40, 22 September 2005 (UTC)[reply]

printing[edit]

How would i make the handwrite on a new written postcard look old?

Use sepia ink and a copperplate hand. Then let the thing sit in the sun for a few weeks. --jpgordon∇∆∇∆ 21:34, 22 September 2005 (UTC)[reply]
You could also try soaking it in tea for that brown, aged look. --Sum0 20:37, 23 September 2005 (UTC)[reply]
Baking it carefully at a reasonably low temperature works very well. Superm401 | Talk 20:44, 23 September 2005 (UTC)[reply]